PSYCH HESI**

Pataasin ang iyong marka sa homework at exams ngayon gamit ang Quizwiz!

Which statement demonstrates that a psychiatric nurse has fostered the most therapeutic nurse-client relationship?

"My clients and I are partners in the planning that helps meet their physical and mental health needs."

Which client statement supports the diagnosis of somatic delusions?

"My heart stopped beating 3 days ago, and now my lungs are rotting away."

A nurse is interviewing a client in the mental health clinic. Which statement by the client indicates an irreversible adverse response to long-term therapy with an antipsychotic medication?

"My tongue seems to move by itself."

A client tells the nurse in the mental health clinic that the practitioner said that the cornerstone of therapy used in the clinic is cognitive therapy. The client asks what this therapy entails. What concept should the nurse explain as the basis of cognitive therapy?

"Negative thoughts can precipitate anxiety."

A client tells a mental health nurse about hearing a man speaking from the corner of the room. The client asks whether the nurse hears him, too. What is the nurse's best response?

"No, I don't hear him, but it probably upsets you to hear him."

One day the nurse and a young adult client sit together and draw. The client draws a face with horns and says, "This is me. I'm a devil." What is the best response by the nurse?

"When I look at you I see a person, not a devil."

A patient in a hospital has dementia, what is a goal?

she will perform ADLs for 3 consecutive days

A health care provider prescribes haloperidol (Haldol) for a client. What should the nurse teach the client to avoid while taking this medication?

Staying in the sun

A withdrawn client refuses to get out of bed and becomes upset when asked to do so. What nursing action is most therapeutic?

Staying with the client until the client calms down

A nurse at the crisis intervention center asks a new female client, who has come because her husband is planning a divorce, her reasons for seeking help. The client responds by describing her first meeting with her husband, when they were both teenagers. What is the most therapeutic response by the nurse?

"And now your husband is asking for a divorce."

A nurse is teaching a client and family about the characteristics of dementia of the Alzheimer type. What physiological characteristic should the nurse include?

Areas of brain destruction called senile plaques

Certain questions are applicable in determining nursing negligence. (Select all that apply.)

- "Was reasonable care provided?" - "Was there a breach of nursing duty?" - "Was there an act of omission that resulted in harm? - "Except for the nurse's action, would the injury have occurred?"

A client with long-term alcohol addiction is admitted to the emergency department. Which medications should the nurse anticipate the healthcare provider will prescribe for this client? (Select all that apply)

- Diazepam. - Multivitamins. - Thiamine (vitamin B1).

A nurse, planning care for a client who is an alcoholic, knows that the most serious life-threatening effects of alcohol withdrawal usually begin after a specific time interval. How many hours after the last drink do they occur? 1 8 to 12 2 12 to 24 3 72 to 96 4 24 to 72

4

. A newly admitted client quietly listens to a nurse's explanation of the services and activities available on the mental health unit. When the nurse is finished, the client looks around and says, "So this is where they keep the crazies." What is the most appropriate initial response by the nurse?

"Are you feeling that a person has to be crazy to need mental health services?"

A client is admitted to the psychiatric hospital after many self-inflicted nonlethal injuries over the preceding month. Of which level of suicidal behavior is the client's behavior reflective?

Gestures

During a group therapy session some members accuse another client of intellectualizing to avoid discussing feelings. The client asks whether the nurse agrees with the others. What is the best response by the nurse?

"Are you uncomfortable with what you were told?"

An older client with a diagnosis of dementia is living in a long-term care facility. The client's daughter, who lives 300 miles away, calls the unit to speak to the nurse about her upcoming visit. What should the nurse say in response to her question about the best time of day to visit?

"Around 2:30 in the afternoon is the best time to visit."

A client is admitted to a psychiatric hospital because of a recurrent mental health problem. During admission the nurse determines the expected client outcomes. The nurse concludes that these outcomes are:

Measurable objectives

The psychiatric clinical nurse specialist decides to use cognitive behavioral therapy (CBT) techniques as she works with a bulimic client. Which statement by the nurse is an example of the application of cognitive behavioral therapy principles?

"Being thin does not seem to solve your problems; you are thin now and still unhappy."

A nurse manager notices that a previously effective nurse appears to be distracted, at times forgets to document changes in clients' status, and rarely completes the required workload without help from another nurse. What should the nurse manager say to the nurse?

"You seem to be having difficulty completing your assignments. What can I do to help?"

One morning a client with the diagnosis of acute depression says, "God is punishing me for my past sins." What is the best response by the nurse?

"You sound very upset about this.

A female client undergoing presurgical testing before a possible colon resection and colostomy says to the nurse, "If I have to have this surgery, I know that my husband will never come near me again." What is the most therapeutic response by the nurse?

"You're concerned about how your husband will respond to your surgery."

A nurse is caring for a client with the diagnosis of schizophrenia. During assessment the nurse identifies both positive (type I) and negative (type II) signs and symptoms. Which clinical findings should the nurse document as positive? (Select all that apply.)

- Disorganized thoughts - Auditory hallucinations

A client is found to have a mood disorder, hypomanic episode. To support the diagnosis, the nurse should identify the signs and symptoms associated with this disorder. (Select all that apply.)

- Distractibility - Flight of ideas

A depressed client is given sertraline (Zoloft) 50 mg at bedtime. For what drug-related side effects should the nurse monitor the client? (Select all that apply.)

- Dry mouth - Constipation

The nurse is interacting with the spouse of a client being treated for substance abuse. During the interview, the spouse makes excuses and takes the blame for the client's actions. Which terms best describe the spouse's behavior? (Select all that apply)

- Enabling. - Co-dependency.

A client states, "I get down on myself when I make a mistake." In a cognitive therapy approach, which nursing interventions are most appropriate? (Select all that apply.)

- Encouraging the client to replace these negative thoughts with positive thoughts - Helping the client modify the belief that anything less than perfection is unacceptable

A client is started on chlorpromazine (Thorazine). To prevent life-threatening complications from the administration of this medication to an anxious, restless client, it is important that the nurse:

Monitor the client's vital signs.

A nurse is implementing interventions to assist an aggressive client in deescalating the agitated behavior. (Select all that apply.)

- Encouraging the client to express his perceived needs - Avoiding verbal struggles in an attempt to demonstrate authority - Providing the client with clear options to the unacceptable behavior - Explaining the expected outcomes if the client is unable to control the unacceptable behavior

A nurse leads an assertiveness training program for a group of clients. Which statement by a client indicates that the treatment has been effective?

"It annoys me when people call me 'sweetie,' so I told him not to do it anymore."

The nurse is teaching a group parents and teachers about preventing substance abuse in children. What practices during childhood are known to minimize substance abuse later in life? (Select all that apply)

- Adults who teach and display self-control behaviors in difficult situations. - Encouraging and praising academic achievement. - A strong sense of community and neighborhood attachment. - Parents who are involved and monitor their children's daily lives.

A client is prescribed a monoamine oxidase inhibitor. The nurse teaches the client about what foods to avoid when taking this medication. (Select all that apply.)

- Aged cheese - Ripe avocados - Delicatessen meats

A client asks the nurse, "Because I'm so comfortable talking with you, can we go out for coffee and movie after I get discharged?" To maintain the boundaries of a therapeutic relationship, how will the nurse respond?

"A social life is important so as your nurse let's talk about how you can form friendships."

A nurse in the mental health clinic has been working regularly with a client. What client behaviors indicate that trust in the nurse has developed? (Select all that apply.)

- Demonstrates decreased muscle tension - Maintains direct eye contact when talking with the nurse

Pt teaching for Buspirone

-2-4 weeks for full effect -No withdrawal symptoms -No sedation -NOT for acute attacks - for CHRONIC anxiety

Creatine levels over ____ means a BAD kidney!

1.3 -Remember - THREE rhymes with kidNEY

Urine ___ ml/hr or less means kidney in DISTRESS

30 ml

A practitioner prescribes routine checks of the client's lithium level to be performed. How many hours after the last dose of lithium should the nurse plan to obtain the blood specimen?

8 to 12

Which part of the client's plan of care is the practical nurse (PN) implementing when plans are used to increase a male client's participation in his own care and social environment?

B. The therapeutic community.

Bupropion (Wellbutrin) has a unique side effect not shared by most other drugs of its class. The nurse should assess the client for which unique possible side effect of this drug?

Generalized seizures

According to Erikson, a person's adjustment to the period of senescence will depend largely on the adjustment the individual made to the earlier developmental stage of:

Generativity versus stagnation

A nurse is counseling a client who has had an angry episode that subsided after several minutes. What is the most important short-term objective for the client?

Talking about situations that cause angry outbursts

When working with a client who is in an alcohol detoxification program, it is most important for the nurse to:

Address the client's holistic needs.

When planning care for an older client, the nurse remembers that aging has little effect on a client's:

Capacity to handle life's stresses

SNRI med names

Duloxetine (Cymbalta) Venlafaxine (Effexor) Levo-minalcipram (Fetzima)

A client is undergoing treatment for anxiety after being physically assaulted. What statement indicates the development of post traumatic stress disorder?

I keep on reliving the moment he grabbed me

Within a few hours of alcohol withdrawal, the nurse should assess the client for the presence of:

Irritability and tremors

Which ego-defense mechanisms are exhibited by a phobic client who refuses to leave her home?

Symbolization

What is the most important information for a nurse to teach to prevent relapse in a client with a psychiatric illness?

The need to follow the prescribed medication regimen

inability to name an object

anomia

An obese woman going through alcohol detox wants a non-Diet Coke, what should you say?

caffeine should be avoided during a detox

How does ziprisadone work?

helps you think clearly

When a girl from car accident comes in with high anxiety what should you do?

make her feel safe

Lithium drugs

treat bipolar disorder; mood stabilizing

The psychiatric clinical nurse specialist decides to use cognitive behavioral therapy (CBT) techniques as she works with a bulimic client. Which statement by the nurse is an example of the application of cognitive behavioral therapy principles? "Being thin does not seem to solve your problems; you are thin now and still unhappy." "What are your feelings about not eating the food that you prepare for others?" "You seem to feel much better about yourself when you eat something." "Is it difficult to talk about your feelings and private matters with someone you've just met?"

"Being thin does not seem to solve your problems; you are thin now and still unhappy."

The spouse of a client who was admitted to an alcohol detox center is attending an AL-ANON meeting. The spouse asks the group leader if their children could inherit the tendency for alcohol addiction. Which is a correct response by the group leader?

"Children of alcoholics are three times more likely to inherit the disease than children of non-alcoholic parents."

The spouse of a client who was admitted to an alcohol detox center is attending an AL-ANON meeting. The spouse asks the group leader if their children could inherit the tendency for alcohol addiction. Which is a correct response by the group leader? "Children of alcoholics are three times more likely to inherit the disease than children of non-alcoholic parents." "Current studies are being conducted to investigate heredity as a factor, but unfortunately there has been no progress." "There are no genetics involved; alcohol abuse and addictions are learned behaviors." "Genetics have a minor effect, but if you educate your children about the dangers of drinking, they should be OK."

"Children of alcoholics are three times more likely to inherit the disease than children of non-alcoholic parents."

Which assessment question will provide the best information regarding a client's risk for waking in the night and interrupted sleep related to lifestyle choices?

"Do you smoke cigarettes, cigars, or a pipe?"

A client in the outpatient clinic is denying that he is addicted to alcohol. He tells the nurse that he is not an alcoholic and that it is his nagging wife who causes him to drink. What is the most therapeutic response by the nurse?

"Everyone is responsible for his own actions."

The nurse is caring for a client with depression who is frequently unkempt and uninterested in daily grooming. The nurse later overhears the client's spouse comment: "Why don't you try to shower, do your hair and makeup and pamper yourself a little bit? It might make you feel better." The client later tells the nurse "My husband doesn't love me anymore." Based on these interactions, what is the best response by the nurse?

"First, take a deep breath. Then take a moment to describe the situation and your moods, then write down all of the automatic thoughts you are experiencing."

A client with a history of obsessive-compulsive behaviors has a marked decrease in symptoms and expresses a wish to obtain a part-time job. On the day of a job interview the client arrives at the mental health center with signs of anxiety. What is the most therapeutic response to the client's behavior by the nurse?

"Going for your interview triggered some feelings in you. Perhaps you could call a friend to drive you there."

A nurse uses the CAGE screening test for alcoholism to determine an individual's potential for a drinking problem. What is one of the four questions included on this test?

"Have you ever felt bad or guilty about your drinking?"

A client undergoing alcohol detoxification asks about attending Alcoholics Anonymous (AA) meetings after discharge. What is the nurse's best initial reply?

"How do you feel about going to those meetings?"

A male nurse is caring for a client. The client states, "You know, I've never had a male nurse before." What is the best reply by the nurse?

"How do you feel about having a male nurse?"

While walking down the hall a female client with schizophrenia stops and stands rigidly. When asked what she is doing, she replies slowly, "I'm standing here because Jesus told me to." What is the best response by the nurse?

"I didn't hear anyone talking; come with me to the community room."

What statement by a male client during a yearly physical examination indicates to a nurse that the client may have a sexual arousal disorder?

"I don't get hard during sex anymore."

A nurse is caring for a client who is experiencing auditory hallucinations. What is the most therapeutic response by the nurse?

"I don't hear the voices you're hearing."

A client who has been taking the prescribed dose of zolpidem (Ambien) for 5 days returns to the clinic for a follow-up visit. When interviewing the client, the nurse identifies that the medication has been effective when the client says:

"I have been sleeping better."

A universal outcome for victims of rape is grief resolution. Which statement by the victim indicates that progress toward this outcome has been made?

"I have begun to stop blaming myself for being raped."

A universal outcome for victims of rape is grief resolution. Which statement by the victim indicates that progress toward this outcome has been made? "I have begun to stop blaming myself for being raped." "If I had not worn that skirt to the party, the rape would not have happened." "I need to make sure that I do not go out alone anymore." "I feel ashamed that I could not defend myself against my attacker."

"I have begun to stop blaming myself for being raped."

A nurse is performing discharge teaching for a client who has been receiving disulfiram (Antabuse). What statement indicates to the nurse that the client understands the teaching concerning disulfuram?

"I have to be careful to check over-the-counter medications."

The nurse tells a client that talking with the staff members is part of the therapy program. The client responds, "I don't see how talking to you can possibly help." What is the most appropriate response by the nurse?

"I hope I'll be able to help you sort out your thoughts and feelings so you can understand them better."

The nurse is providing counseling to a client about recovery from alcoholism. Which client statement indicates that learning has occurred?

"I realize that recovery is a lifelong process that occurs in steps."

The nurse is providing counseling to a client about recovery from alcoholism. Which client statement indicates that learning has occurred? "Once I have detoxed, my recovery is complete." "I realize that recovery is a lifelong process that occurs in steps." "I understand that the goal of the program is to decrease my drinking." "I will only drink beer because hard liquor has a higher alcohol content."

"I realize that recovery is a lifelong process that occurs in steps."

In response to a question posed during a group meeting, the nurse explains that the superego is that part of the self that says:

"I shouldn't want that."

An extremely anxious client enters a crisis center and asks a nurse for help. Which initial response best reflects the nurse's role in crisis intervention?

"I'll be here for you to help you figure things out."

One day a nurse sits down by a depressed client's bed and says, "I'll be spending some time with you today." The client responds, "Go talk to someone else. They all need you more." What is the most therapeutic response by the nurse?

"I'll be spending the next half hour with you."

A client with schizophrenia is admitted to a psychiatric unit. The client is talking while walking in the hall, is unkempt, and obviously has not washed in several days. What should the nurse say when trying to help this client shower?

"I'll help you take your shower now."

A client with a diagnosis of schizophrenia, undifferentiated type, was admitted to the mental health hospital 3 days ago. The client stays in the bedroom except to eat and has no verbal interaction with other clients. When the nurse approaches, the client walks away and says, "Just leave me alone." What is the best response by the nurse?

"I'll talk to you later."

A client who consented to electroconvulsive therapy (ECT) is being prepared for the second session. The client tells the nurse, "I've decided that I don't want this treatment." What is the best response by the nurse?

"I'll tell your psychiatrist that you don't want the treatment."

A newly admitted client looks at but does not respond to the nurse. What is the most appropriate statement by the nurse?

"I'm here to offer you my help and tell you about the services available to you on the mental health unit."

A physician is admitted to the psychiatric unit of a community hospital. The client, who was restless, loud, aggressive, and resistive during the admission procedure, announces, "I'll take my own blood pressure." What is the most therapeutic response by the nurse?

"I'm sorry, but I can't allow that, because I have to take your blood pressure."

An adolescent with the diagnosis of antisocial personality disorder is admitted to the hospital after ingesting 20 tablets of an anxiolytic. When obtaining the client's history, the nurse learns that there was an arrest for drug use and that the client is out on bail. During visiting hours, the nurse discovers the client and visitors smoking marijuana in the hall. When confronted, the client responds, "I'm celebrating. Didn't you hear? I went to trial today and just got put on probation." What is the best response by the nurse?

"If you can't follow the rules against drug use on the unit, your visiting privileges will be canceled."

A client who has recently been found to be infected with HIV comments to the nurse, "There are so many terrible people around. Why couldn't one of them get HIV instead of me?" What is the best response by the nurse?

"It seems unfair that you should have this disease."

A 20-year-old student comes to the college health clinic reporting increasing anxiety, loss of appetite, and an inability to concentrate. What is the most therapeutic response by the nurse?

"It's been difficult for you. How long has this been going on?"

At a staff meeting, while discussing the return of one of the staff nurses from a drug rehabilitation program, one nurse states, "I don't know why we are wasting time on this. We all know that addicts go back to using drugs as soon as the pressure increases." What is the nurse manager's best response?

"It's important for us to share our feelings about staff members with problems."

A client with schizophrenia plans an activity schedule with the help of the treatment team. A written copy is posted in the client's room. What should the nurse say when it is time for the client to go for a walk?

"It's time for you to go for a walk now."

During a group meeting a client tells everyone, "I'm about to be discharged from the hospital, and I'm afraid." What is the most appropriate response by the nurse facilitator?

"Maybe others in the group have similar feelings that they would share."

A woman arrives in the Emergency Center and tells the nurse she thinks she has been raped. The client is sobbing and expresses disbelief that a rape could happen because the man is her best friend.After acknowledging the client's fear and anxiety, how should the nurse respond?

"Rape is no limited to strangers and frequently occurs by someone who is known to the victim"

The nurse working on the mental health unit finds a depressed client crying. What is the most therapeutic approach to help the client explore feelings?

"Tell me what you're feeling now."

During a meeting with the interdisciplinary treatment team, a client in the acute phase of schizophrenia states that she cannot return to live with her parents because they are trying to kill her. Which statement by the team leader represents a correct therapeutic response?

"That must be very frightening; tell us why you believe you are in danger."

During a meeting with the interdisciplinary treatment team, a client in the acute phase of schizophrenia states that she cannot return to live with her parents because they are trying to kill her. Which statement by the team leader represents a correct therapeutic response? "We met with your parents; they do not appear to be violent people." "You are having a delusion; your parents would never hurt you." "That must be very frightening; tell us why you believe you are in danger." "You are being paranoid; your parents referred you here so that you can be helped."

"That must be very frightening; tell us why you believe you are in danger."

A nurse is writing a plan of care in the medical record of a paranoid male client who has unjustifiably accused his wife of having many extramarital affairs. An intermediate goal for this client is:

"The client will develop feelings of self-worth."

A client with a diagnosis of paranoid schizophrenia tells the nurse, "Foreign agents are talking to me, and they say they're going to get me eventually." What is the most therapeutic response by the nurse?

"These thoughts are frightening, but I can't hear the voices."

A client diagnosed with obsessive-compulsive personality disorder with a comorbid depression has been receiving dialectic behavior therapy (DBT). Which statement by the client indicates that the treatment has been effective?

"Yoga and deep breathing help me when I get anxious about upcoming family visits."

A client diagnosed with obsessive-compulsive personality disorder with a comorbid depression has been receiving dialectic behavior therapy (DBT). Which statement by the client indicates that the treatment has been effective? "Yoga and deep breathing help me when I get anxious about upcoming family visits." "I can feel my heart pounding when I practice deep breathing and it gives me a headache." "Listening to relaxing music reminds me of my ex-husband and how I messed up our marriage." "I feel like I can become a movie star after I finish my meditation and exercise sessions."

"Yoga and deep breathing help me when I get anxious about upcoming family visits."

A nurse speaks with a client who has just experienced a panic attack. Which statement will be the most therapeutic in addressing the client's concerns?

"You are concerned that this might happen again"

A nurse on a mental health unit has developed a therapeutic relationship with a manipulative, acting-out client. One day as the nurse is leaving, the client says, "Please stay. I'm afraid that the evening staff doesn't like me. They're always punishing me." What is the nurse's most therapeutic response?

"You know I leave at this time. We'll talk about this in the morning."

A client who is experiencing tremendous stress and anxiety while being admitted to a drug detoxication facility asks the nurse, "Do you think saying a prayer would help?" Which response by the nurse is best?

"You may find that prayer gives comfort and lowers your stress."

A client who is experiencing tremendous stress and anxiety while being admitted to a drug detoxication facility asks the nurse, "Do you think saying a prayer would help?" Which response by the nurse is best? "To be honest, prayer may be the best way to help you cope." "You may find that prayer gives comfort and lowers your stress." "I could help you feel calmer by teaching you to meditate." "Guided imagery could be an effective alternative."

"You may find that prayer gives comfort and lowers your stress."

4. A 38-year-old female client is admitted with a diagnosis of paranoid schizophrenia. When her tray is brought to her, she refuses to eat and tells the nurse, "I know you are trying to poison me with that food." Which response is most appropriate for the nurse to make? A) I'll leave your tray here. I am available if you need anything else. B) You're not being poisoned. Why do you think someone is trying to poison you? C) No one on this unit has ever died from poisoning. You're safe here. D) I will talk to your healthcare provider about the possibility of changing your diet.

(A) is the best choice cited. The nurse does not argue with the client nor demand that she eat, but offers support by agreeing to "be there if needed", e.g., to warm the food. (B and C) are arguing with the client's delusions, and (B) asks "why" which is usually not a good question for a psychotic client. (D) has nothing to do with the actual problem; i.e., the problem is not the diet (she thinks any food given to her is poisoned.) Correct Answer(s): A

4. A 38-year-old female client is admitted with a diagnosis of paranoid schizophrenia. When her tray is brought to her, she refuses to eat and tells the nurse, "I know you are trying to poison me with that food." Which response is most appropriate for the nurse to make? A) I'll leave your tray here. I am available if you need anything else. B) You're not being poisoned. Why do you think someone is trying to poison you? C) No one on this unit has ever died from poisoning. You're safe here. D) I will talk to your healthcare provider about the possibility of changing your diet.

(A) is the best choice cited. The nurse does not argue with the client nor demand that she eat, but offers support by agreeing to "be there if needed", e.g., to warm the food. (B and C) are arguing with the client's delusions, and (B) asks "why" which is usually not a good question for a psychotic client. (D) has nothing to do with the actual problem; i.e., the problem is not the diet (she thinks any food given to her is poisoned.) Correct Answer(s): A

16. The nurse should include which interventions in the plan of care for a severely depressed client with neurovegetative symptoms? (Select all that apply.) A) Permit rest periods as needed. B) Speaking slowly and simply. C) Place the client on suicide precautions. D) Allow the client extra time to complete tasks. E) Observe and encourage food and fluid intake. F) Encourage mild exercise and short walks on the unit

(A, B, D, E, and F) should be included in this client's plan of care because these measures promote the client's comfort and well-being. Neurovegetative symptoms accompany the mood disorder of depression and include physiological disruptions, such as anorexia, constipation, sleep disturbance, and psychomotor retardation. Suicidal ideation (C) does not usually accompany the neurovegetative state because the client does not have the energy or high level of anxiety associated with a suicide attempt. Correct Answer(s): A, B, D, E, F

37. A 45-year-old male client tells the nurse that he used to believe that he was Jesus Christ, but now he knows he is not. Which response is best for the nurse to make? A) Did you really believe you were Jesus Christ? B) I think you're getting well. C) Others have had similar thoughts when under stress. D) Why did you think you were Jesus Christ?

(C) offers support by assuring the client that others have suffered as he has (also the principle on which Alcoholics Anonymous acts). (A) is belittling. (B) is making an inappropriate judgment. You may have narrowed your choices to (C and D). However, you should eliminate (D) because it is a "why" question, and the client does not know why! Correct Answer(s): C

16. The nurse should include which interventions in the plan of care for a severely depressed client with neurovegetative symptoms? (Select all that apply.) A) Permit rest periods as needed. B) Speaking slowly and simply. C) Place the client on suicide precautions. D) Allow the client extra time to complete tasks. E) Observe and encourage food and fluid intake. F) Encourage mild exercise and short walks on the unit

(A, B, D, E, and F) should be included in this client's plan of care because these measures promote the client's comfort and well-being. Neurovegetative symptoms accompany the mood disorder of depression and include physiological disruptions, such as anorexia, constipation, sleep disturbance, and psychomotor retardation. Suicidal ideation (C) does not usually accompany the neurovegetative state because the client does not have the energy or high level of anxiety associated with a suicide attempt. Correct Answer(s): A, B, D, E, F

68. Physical examination of a 6-year-old reveals several bite marks in various locations on his body. X-ray examination reveals healed fractures of the ribs. The mother tells the nurse that her child is always having accidents. Which initial response by the nurse would be most appropriate? A) I need to inform the healthcare provider about your child's tendency to be accident prone. B) Tell me more specifically about your child's accidents. C) I must report these injuries to the authorities because they do not seem accidental. D) Boys this age always seem to require more supervision and can be quite accident prone.

(B) seeks more information using an open ended, non-threatening statement. (A) could be appropriate, but it is not the best answer because the nurse is being somewhat sarcastic and is also avoiding the situation by referring it to the healthcare provider for resolution. Although it is true that suspected cases of child abuse must be reported, (C) is virtually an attack and is jumping to conclusions before conclusive data has been obtained. (D) is a cliché and dismisses the seriousness of the situation. Correct Answer(s): B

68. Physical examination of a 6-year-old reveals several bite marks in various locations on his body. X-ray examination reveals healed fractures of the ribs. The mother tells the nurse that her child is always having accidents. Which initial response by the nurse would be most appropriate? A) I need to inform the healthcare provider about your child's tendency to be accident prone. B) Tell me more specifically about your child's accidents. C) I must report these injuries to the authorities because they do not seem accidental. D) Boys this age always seem to require more supervision and can be quite accident prone.

(B) seeks more information using an open ended, non-threatening statement. (A) could be appropriate, but it is not the best answer because the nurse is being somewhat sarcastic and is also avoiding the situation by referring it to the healthcare provider for resolution. Although it is true that suspected cases of child abuse must be reported, (C) is virtually an attack and is jumping to conclusions before conclusive data has been obtained. (D) is a cliché and dismisses the seriousness of the situation. Correct Answer(s): B

37. A 45-year-old male client tells the nurse that he used to believe that he was Jesus Christ, but now he knows he is not. Which response is best for the nurse to make? A) Did you really believe you were Jesus Christ? B) I think you're getting well. C) Others have had similar thoughts when under stress. D) Why did you think you were Jesus Christ?

(C) offers support by assuring the client that others have suffered as he has (also the principle on which Alcoholics Anonymous acts). (A) is belittling. (B) is making an inappropriate judgment. You may have narrowed your choices to (C and D). However, you should eliminate (D) because it is a "why" question, and the client does not know why! Correct Answer(s): C

A client who was in an automobile accident is admitted to the hospital with multiple injuries. Approximately 14 hours after admission, the client begins to experience signs and symptoms of withdrawal from alcohol. Which signs and symptoms should the nurse connect to alcohol withdrawal? (Select all that apply.)

- Anxiety - Diaphoresis - Psychomotor agitation

39. A 25-year-old female client has been particularly restless and the nurse finds her trying to leave the psychiatric unit. She tells the nurse, "Please let me go! I must leave because the secret police are after me." Which response is best for the nurse to make? A) No one is after you, you're safe here. B) You'll feel better after you have rested. C) I know you must feel lonely and frightened. D) Come with me to your room and I will sit with you.

(D) is the best response because it offers support without judgment or demands. (A) is arguing with the client's delusion. (B) is offering false reassurance. (C) is a violation of therapeutic communication in that the nurse is telling the client how she feels (frightened and lonely), rather than allowing the client to describe her own feelings. Hallucinating and/or delusional clients are not capable of discussing their feelings, particularly when they perceive a crisis. Correct Answer(s): D

39. A 25-year-old female client has been particularly restless and the nurse finds her trying to leave the psychiatric unit. She tells the nurse, "Please let me go! I must leave because the secret police are after me." Which response is best for the nurse to make? A) No one is after you, you're safe here. B) You'll feel better after you have rested. C) I know you must feel lonely and frightened. D) Come with me to your room and I will sit with you.

(D) is the best response because it offers support without judgment or demands. (A) is arguing with the client's delusion. (B) is offering false reassurance. (C) is a violation of therapeutic communication in that the nurse is telling the client how she feels (frightened and lonely), rather than allowing the client to describe her own feelings. Hallucinating and/or delusional clients are not capable of discussing their feelings, particularly when they perceive a crisis. Correct Answer(s): D

32. The nurse is leading a "current events group" with chronic psychiatric clients. One group member states, "Saddam Hussein was my nurse during my last hospitalization. He was a very mean nurse and wasn't nice to me." Which response is best for the nurse to make? A) Saddam Hussein was not your nurse. B) What did he do to you that was so mean? C) I didn't know that Saddam Hussein was a nurse. D) I agree that Saddam Hussein is not a very nice man.

(D) presents the reality of the situation (the individual is not nice) in relation to American culture. The fact that Saddam Hussein is not a nurse should be addressed on an individual basis. Since this is group therapy, the nurse would be illustrating the concept of universality. (A) is likely to promote defensiveness. (B and C) would support the delusion. Correct Answer(s): D

32. The nurse is leading a "current events group" with chronic psychiatric clients. One group member states, "Saddam Hussein was my nurse during my last hospitalization. He was a very mean nurse and wasn't nice to me." Which response is best for the nurse to make? A) Saddam Hussein was not your nurse. B) What did he do to you that was so mean? C) I didn't know that Saddam Hussein was a nurse. D) I agree that Saddam Hussein is not a very nice man.

(D) presents the reality of the situation (the individual is not nice) in relation to American culture. The fact that Saddam Hussein is not a nurse should be addressed on an individual basis. Since this is group therapy, the nurse would be illustrating the concept of universality. (A) is likely to promote defensiveness. (B and C) would support the delusion. Correct Answer(s): D

52. The nurse plans to help an 18-year-old female mentally retarded client ambulate the first postoperative day after an appendectomy. When the nurse tells the client it is time to get out of bed, the client becomes angry and tells the nurse, "Get out of here! I'll get up when I'm ready!" Which response is best for the nurse to make? A) Your healthcare provider has prescribed ambulation on the first postoperative day. B) You must ambulate to avoid complications which could cause more discomfort than ambulating. C) I know how you feel. You're angry about having to ambulate, but this will help you get well. D) I'll be back in 30 minutes to help you get out of bed and walk around the room.

(D) provides a "cooling off" period, is firm, direct, non-threatening, and avoids arguing with the client. (A) is avoiding responsibility by referring to the healthcare provider. (B) is trying to reason with a mentally retarded client and is threatening the client with "complications." (C) is telling the client how she feels (angry), and the nurse does not really "know" how this client feels, unless the nurse is mentally retarded and has just had an appendectomy! Correct Answer(s): D

A 45-year-old male client tells the nurse that he used to believe that he was Jesus Christ, but now he knows he is not. Which response is best for the nurse to make? - "Did you really believe you were Jesus Christ?" - "I think you're getting well." - "Others have had similar thoughts when under stress." - "Why did you think you were Jesus Christ?"

- "Others have had similar thoughts when under stress." (C) offers support by assuring the client that others have suffered as he has (also the principle on which Alcoholics Anonymous acts). (A) is belittling. (B) is making an inappropriate judgment. You may have narrowed your choices to (C and D). However, you should eliminate (D) because it is a "why" question, and the client does not know why!

52. The nurse plans to help an 18-year-old female mentally retarded client ambulate the first postoperative day after an appendectomy. When the nurse tells the client it is time to get out of bed, the client becomes angry and tells the nurse, "Get out of here! I'll get up when I'm ready!" Which response is best for the nurse to make? A) Your healthcare provider has prescribed ambulation on the first postoperative day. B) You must ambulate to avoid complications which could cause more discomfort than ambulating. C) I know how you feel. You're angry about having to ambulate, but this will help you get well. D) I'll be back in 30 minutes to help you get out of bed and walk around the room.

(D) provides a "cooling off" period, is firm, direct, non-threatening, and avoids arguing with the client. (A) is avoiding responsibility by referring to the healthcare provider. (B) is trying to reason with a mentally retarded client and is threatening the client with "complications." (C) is telling the client how she feels (angry), and the nurse does not really "know" how this client feels, unless the nurse is mentally retarded and has just had an appendectomy! Correct Answer(s): D

48. The nurse suspects child abuse when assessing a 3-year-old boy and noticing several small, round burns on his legs and trunk that might be the result of cigarette burns. Which parental behavior provides the greatest validation for such suspicions? A) The parents' explanation of how the burns occurred is different from the child's explanation of how they occurred. B) The parents seem to dismiss the severity of the child's burns, saying they are very small and have not posed any problem. C) The parents become very anxious when the nurse suggests that the child may need to be admitted for further evaluation. D) The parents tell the nurse that the child was burned in a house fire which is incompatible with the nurse's observation of the type of burn.

(D) provides the most validation. The parent's explanation (subjective data) is incompatible with the objective data (small round burns on the legs and trunk). (A) provides only subjective data, and the child's explanation could be influenced by factors such as age, fear, or imagination. The parent's apparent lack of concern (B) is inconclusive, but the nurse's opinion of the parents' reaction is subjective and could be wrong. (C) might provide a clue that child abuse occurred, but the nurse must remember that most parents are anxious about their child being hospitalized. Correct Answer(s): D

48. The nurse suspects child abuse when assessing a 3-year-old boy and noticing several small, round burns on his legs and trunk that might be the result of cigarette burns. Which parental behavior provides the greatest validation for such suspicions? A) The parents' explanation of how the burns occurred is different from the child's explanation of how they occurred. B) The parents seem to dismiss the severity of the child's burns, saying they are very small and have not posed any problem. C) The parents become very anxious when the nurse suggests that the child may need to be admitted for further evaluation. D) The parents tell the nurse that the child was burned in a house fire which is incompatible with the nurse's observation of the type of burn.

(D) provides the most validation. The parent's explanation (subjective data) is incompatible with the objective data (small round burns on the legs and trunk). (A) provides only subjective data, and the child's explanation could be influenced by factors such as age, fear, or imagination. The parent's apparent lack of concern (B) is inconclusive, but the nurse's opinion of the parents' reaction is subjective and could be wrong. (C) might provide a clue that child abuse occurred, but the nurse must remember that most parents are anxious about their child being hospitalized. Correct Answer(s): D

A 65-year-old female client complains to the nurse that recently she has been hearing voices. What question should the nurse ask this client first? - "Do you have problems with hallucinations?" - "Are you ever alone when you hear the voices?" - "Has anyone in your family had hearing problems?" - "Do you see things that others cannot see?"

- "Are you ever alone when you hear the voices?" Determining if the client is alone when she hears voices (B) will assist in differentiating between hallucinations and hearing loss; this is especially important in the aging population. If the client is experiencing hallucinations, the voices will be real to her, and it is unlikely that (A) would provide accurate information. (C and D) might be good follow-up questions, but would not have the priority of (B).

Which characteristics have been associated with substance abuse? (Select all that apply)

- Grandiosity. - Manipulation. - Dysfunctional anger.

The nurse is taking a history for a female client who is requesting a routine female exam. Which assessment finding requires follow-up? - Menstruation onset at age 9. - Contraceptive method includes condoms only. - Menstrual cycle occurs every 35 days. - "Black-out" after one drink last night on a date.

- "Black-out" after one drink last night on a date. A "black-out" typically occurs after ingestion of alcohol beverages that the client has no recall of the experiences or one's behavior and is indicative of high blood alcohol levels, but the client's experience of a "black-out" after one drink (D) is suspicious of the client receiving a "date rape" drug (Flunitrazepam) and needs additional follow-up. Although (A and C) occur on the outer ranges of "average," both are within acceptable or "normal" ranges. (B) is an individual preference, but using condoms as the only contraceptive method carries a higher chance of conception.

The nurse is leading a "current events group" with chronic psychiatric clients. One group member states, "Clara Barton was my nurse during my last hospitalization. She was a very mean nurse and wasn't nice to me." Which response would be best for the nurse to make? - "Clara Barton was not your nurse." - "What did she do to you that was so mean?" - "I didn't know that Clara Barton was a nurse." - "Clara Barton started the American Red Cross."

- "Clara Barton started the American Red Cross." (D) presents the reality of the situation in relation to American culture. The fact that Clara Barton was a nurse during the Civil War should be addressed on an individual basis. Since this is group therapy, the nurse would be illustrating the concept of universality. (A) is likely to promote defensiveness. (B and C) would support the delusion.

A 25-year-old female client has been particularly restless and the nurse finds her trying to leave the psychiatric unit. She tells the nurse, "Please let me go! I must leave because the secret police are after me." Which response is best for the nurse to make? - "No one is after you, you're safe here." - "You'll feel better after you have rested." - "I know you must feel lonely and frightened." - "Come with me to your room and I will sit with you."

- "Come with me to your room and I will sit with you." (D) is the best response because it offers support without judgment or demands. (A) is arguing with the client's delusion. (B) is offering false reassurance. (C) is a violation of therapeutic communication in that the nurse is telling the client how she feels (frightened and lonely), rather than allowing the client to describe her own feelings. Hallucinating and/or delusional clients are not capable of discussing their feelings, particularly when they perceive a crisis.

An adult male client who was admitted to the mental health unit yesterday tells the nurse that microchips were planted in his head for military surveillance of his every move. Which response is best for the nurse to provide? - "You are in the hospital, and I am the nurse caring for you." - "It must be difficult for you to control your anxious feelings." - "Go to occupational therapy and start a project." - "You are not in a war area now; this is the United States."

- "Go to occupational therapy and start a project." Delusions often generate fear and isolation, so the nurse should help the client participate in activities that avoid focusing on the false belief and encourage interaction with others (C). Delusions are often well-fixed, and though (A) reinforces reality, it is argumentative and dismisses the client's fears. It is often difficult for the client to recognize the relationship between delusions and anxiety (B), and the nurse should reassure the client that he is in a safe place. Dismissing delusional thinking (D) is unrealistic because neurochemical imbalances that cause positive symptoms of schizophrenia require antipsychotic drug therapy.

A client who has been admitted to the psychiatric unit tells the nurse, "My problems are so bad that no one can help me." Which response is best for the nurse to make? - "How can I help?" - "Things probably aren't as bad as they seem right now." - "Let's talk about what is right with your life." - "I hear how miserable you are, but things will get better soon."

- "How can I help?" Offering self shows empathy and caring (A), and is the best of the choices provided. Combining the first part of (D) with (A) would be the best response, but this is not a fill-in-the-blank or an essay test! Choose the best of those choices provided and move on. (B) dismisses the client, things are bad as far as this client is concerned. (C) avoids the client's problems and promotes denial. "I hear how miserable you are" is an example of reflective dialogue and would be the best choice if it were not for the rest of the sentence--"but things will get better" which is offering false reassurance.

A 72-year-old female client is admitted to the psychiatric unit with a diagnosis of major depression. Which statement by the client should be of greatest concern to the nurse and require further assessment? - "I will die if my cat dies." - "I don't feel like eating this morning." - "I just went to my friend's funeral." - "Don't you have more important things to do?"

- "I will die if my cat dies." Sometimes a client will use an analogy to describe themselves, and (A) would be an indication for conducting a suicide assessment. (B) could have a variety of etiologies, and while further assessment is indicated, this statement does not indicate potential suicide. Normal grief process differs from depression, and at this client's age peer/cohort deaths are more frequent, so (C) would be within normal limits. (D) is an expression of low self-esteem typical of depression. (B, C, and D) are examples of decreased energy and mood levels which would negate suicide ideation at this time.

A family member brings a relative to the local community hospital because the relative "has been acting strange." Which statements meet involuntary hospitalization criteria? (Select all that apply.)

- "I'd like to end it all with sleeping pills." - "The voices say I should kill all prostitutes."

The nurse is conducting discharge teaching for a client with schizophrenia who plans to live in a group home. Which statement is most indicative of the need for careful follow-up after discharge? - "Crickets are a good source of protein." - "I have not heard any voices for a week." - "Only my belief in God can help me." - "Sometimes I have a hard time sitting still."

- "Only my belief in God can help me." The most frequent cause of increased symptoms in psychotic clients is non-compliance with the medication regimen. If clients believe that "God alone" is going to heal them (C), then they may discontinue their medication, so (C) would pose the greatest threat to this client's prognosis. (A) would require further teaching, but is not as significant a statement as (C). (B) indicates an improvement in the client's condition. (D) may be a sign of anxiety that could improve with treatment, but does not have the priority of (C).

The nurse plans to help an 18-year-old female intellectually disabled client ambulate the first postoperative day after an appendectomy. When the nurse tells the client it is time to get out of bed, the client becomes angry and tells the nurse, "Get out of here! I'll get up when I'm ready!" Which response is best for the nurse to make? - "Your healthcare provider has prescribed ambulation on the first postoperative day." - "You must ambulate to avoid complications which could cause more discomfort than ambulating." - "I know how you feel. You're angry about having to ambulate, but this will help you get well." - "I'll be back in 30 minutes to help you get out of bed and walk around the room."

- "I'll be back in 30 minutes to help you get out of bed and walk around the room." (D) provides a "cooling off" period, is firm, direct, non-threatening, and avoids arguing with the client. (A) is avoiding responsibility by referring to the healthcare provider. (B) is trying to reason with an intellectually disabled client and is threatening the client with "complications." (C) is telling the client how she feels (angry), and the nurse does not really "know" how this client feels, unless the nurse is also intellectually disabled and has also just had an appendectomy.

A 38-year-old female client is admitted with a diagnosis of paranoid schizophrenia. When her tray is brought to her, she refuses to eat and tells the nurse, "I know you are trying to poison me with that food." Which response would be most appropriate for the nurse to make? - "I'll leave your tray here. I am available if you need anything else." - "You're not being poisoned. Why do you think someone is trying to poison you?" - "No one on this unit has ever died from poisoning. You're safe here." - "I will talk to your healthcare provider about the possibility of changing your diet."

- "I'll leave your tray here. I am available if you need anything else." (A) is the best choice cited. The nurse does not argue with the client nor demand that she eat, but offers support by agreeing to "be there if needed", e. g., to warm the food. (B and C) are arguing with the client's delusions, and (B) asks "why" which is usually not a good question for a psychotic client. (D) has nothing to do with the actual problem; i. e., the problem is not the diet (she thinks any food given to her is poisoned).

The wife of a male client recently diagnosed with schizophrenia asks the nurse, "What exactly is schizophrenia? Is my husband all right?" Which response is best for the nurse to provide to this family member? - "It sounds like you're worried about your husband. Let's sit down and talk." - "It is a chemical imbalance in the brain that causes disorganized thinking." - "Your husband will be just fine if he takes his medications regularly." - "I think you should talk to your husband's psychologist about this question."

- "It is a chemical imbalance in the brain that causes disorganized thinking." The nurse should answer the client's question with factual information and explain that schizophrenia is a chemical imbalance in the brain (B). (A) is a therapeutic response but does not answer the question, and may be an appropriate response after the nurse answers the question asked. Although (C) is likely true to some degree, it is also true that some clients continue to have disorganized thinking even with antipsychotic medications. Referring the spouse to the psychologist (D) is avoiding the issue; the nurse can and should answer the question.

When preparing a teaching plan for a client who is to be discharged with a prescription for lithium carbonate (Lithonate), it is most important for the nurse to include which instruction? - "It may take 3 to 4 weeks to achieve therapeutic effects." - "Keep your dietary salt intake consistent." - "Avoid eating aged cheese and chicken liver." - "Eat foods high in fiber such as whole grain breads."

- "Keep your dietary salt intake consistent." Lithium's effectiveness is influenced by salt intake (B). Too much salt causes more lithium to be excreted, thereby decreasing the effectiveness of the drug. Too little salt causes less lithium to be excreted, potentially resulting in toxicity. (A, C, and D) are not specific instructions pertinent to teaching about lithium carbonate (Lithonate).

A male client is admitted to a mental health unit on Friday afternoon and is very upset on Sunday because he has not had the opportunity to talk with the healthcare provider. Which response is best for the nurse to provide this client? - "Let me call and leave a message for your healthcare provider." - "The healthcare provider should be here on Monday morning." - "How can I help answer your questions?" - "What concerns do you have at this time?"

- "Let me call and leave a message for your healthcare provider." It is best for the nurse to call the healthcare provider (A) because clients have the right to information about their treatment. Suggesting that the healthcare provider will be available the following day (B) does not provide immediate reassurance to the client. The nurse can also implement offer to assist the client (C and D), but the highest priority intervention is contacting the healthcare provider.

An 86-year-old female client with Alzheimer's disease is wandering the busy halls of the extended care facility and asks the nurse, "Where should I stand for the parade?" Which response is best for the nurse to provide? - "Anywhere you want to stand as long as you do not get hurt by those in the parade." - "You are confused because of all the activity in the hall. There is no parade." - "Let's go back to the activity room and see what is going on in there." - "Remember I told you that this is a nursing home and I am your nurse."

- "Let's go back to the activity room and see what is going on in there." It is common for those with Alzheimer's disease to use the wrong words. Redirecting the client (using an accepting non-judgmental dialogue) to a safer place and familiar activities (C) is most helpful because clients experience short-term memory loss. (A) dismisses the client's attempt to find order and does not help her relate to her surroundings. (B) dismisses the client and may increase her anxiety level because it merely labels the client's behavior and offers no solution. It is very frustrating for those with Alzheimer's disease to "remember," and scolding them (D) may hurt their feelings.

The nurse is preparing to administer phenelzine sulfate (Nardil) to a client on the psychiatric unit. Which complaint related to administration of this drug would the nurse expect this client to make? - "My mouth feels like cotton." - "That stuff gives me indigestion." - "This pill gives me diarrhea." - "My urine looks pink."

- "My mouth feels like cotton." A dry mouth (A) is an anticholinergic effect that is an expected side effect of MAO inhibitors such as phenelzine sulfate (Nardil). (B, C, and D) are not expected side effects of this medication.

team approach is used to help a 6-year-old boy with attention deficit-hyperactivity disorder (ADHD). What behaviors indicate that the interventions have been effective? (Select all that apply.)

- Has an increased attention span in school - Is able to wait his turn when in line with others

A woman arrives in the Emergency Center and tells the nurse she thinks she has been raped. The client is sobbing and expresses disbelief that a rape could happen because the man is her best friend. After acknowledging the client's fear and anxiety, how should the nurse respond? - "I would be very upset and mad if my best friend did that to me." - "You must feel betrayed, but maybe you might have led him on?" - "Rape is not limited to strangers and frequently occurs by someone who is known to the victim." - "This does not sound like rape. Did you change your mind about having sex after the fact?"

- "Rape is not limited to strangers and frequently occurs by someone who is known to the victim." A victim of date rape or acquaintance rape is less prone to recognize what is happening because the incident usually involves persons who know each other and the dynamics are different than rape by a stranger. (C) provides confrontation for the client's denial because the victim frequently knows and trusts the perpetrator. Nurses should not express personal feelings (A) when dealing with victims. Suggesting that the client led on the rapist (B) indicates that the sexual assault was somehow the victim’s fault. (D) is judgmental and does not display compassion or establish trust between the nurse and the client.

A hospitalized client with a mood disorder begins to be less hyperactive and acts calmer. One day the client says to the nurse, "My partner and I have problems getting along. Sometimes we don't see eye to eye." What are the nurse's most therapeutic responses? (Select all that apply.)

- "Tell me more about how you see things differently." - "It can be very upsetting to be at odds with your partner." - "Let's talk about a specific time when you didn't see eye to eye."

Physical examination of a 6-year-old reveals several bite marks in various locations on his body. X-ray examination reveals healed fractures of the ribs. The mother tells the nurse that her child is always having accidents. Which initial response by the nurse is most appropriate? - "I need to inform the healthcare provider about your child's tendency to be accident prone." - "Tell me more specifically about your child's accidents." - "I must report these injuries to the authorities because they do not seem accidental." - "Boys this age always seem to require more supervision and can be quite accident prone."

- "Tell me more specifically about your child's accidents." (B) seeks more information using an open ended, non-threatening statement. (A) could be appropriate, but it is not the best answer because the nurse is being somewhat sarcastic and is also avoiding the situation by referring it to the healthcare provider for resolution. Although it is true that suspected cases of child abuse must be reported, (C) is virtually an attack and is jumping to conclusions before conclusive data has been obtained. (D) is a clich and dismisses the seriousness of the situation.

A male client with schizophrenia tells the nurse that the voices he hears are saying, "You must kill yourself." To assist the client in coping with these thoughts, which response is best for the nurse to provide? - "Tell yourself that the voices are unreasonable." - "Exercise when you hear the voices." - "Talk to someone when you hear the voices." - "The voices aren't real, so ignore them."

- "Tell yourself that the voices are unreasonable." The nurse should teach the client to use self-talk to disprove the voices (A). Although (B) may be helpful, the client's concrete thinking may make it difficult to understand this suggestion. Clients with schizophrenia have difficulty initiating interaction with others (C). Auditory hallucinations are often relentless, so it is difficult to ignore them (D).

A female client with depression attends group and states that she sometimes misses her medication appointments because she feels very anxious about riding the bus. Which statement is the nurse's best response? - "Can your case manager take you to your appointments?" - "Take your medication for anxiety before you ride the bus." - "Let's talk about what happens when you feel very anxious." - "What are some ways that you can cope with your anxiety?"

- "What are some ways that you can cope with your anxiety?" The best response is to explore ways for the client to cope with anxiety (D). The nurse should encourage problem-solving rather than dependence on the case manager (A) for transportation. Strategies for coping with anxiety should be encouraged before suggesting (B). (C) is therapeutic, but the best response is an open-ended question to explore ways to cope with the anxiety.

A newly immigrated older Chinese adult is brought to a mental health clinic when family members become concerned that their parent is depressed. In an attempt to conduct a culturally competent assessment interview, the nurse asks certain questions. (Select all that apply.)

- "What brought you here for treatment today?" - "What do you believe is the cause of your depression?" - "Does religion have a role in your perception of health and wellness?" -"Have you ever sought treatment for a mental health problem previously?"

The nurse observes a female client with schizophrenia watching the news on TV. She begins to laugh softly and says, "Yes, my love, I'll do it." When the nurse questions the client about her comment she states, "The news commentator is my lover and he speaks to me each evening. Only I can understand what he says." What is the best response for the nurse to make? - "What do you believe the news commentator said to you?" - "Let's watch news on a different television channel." - "Does the news commentator have plans to harm you or others?" - "The news commentator is not talking to you."

- "What do you believe the news commentator said to you?" It is imperative that the nurse determine what the client believes she heard (A). The idea of reference may be to hurt herself or someone else, and the main function of a psychiatric nurse is to maintain safety. (B) is acceptable, but it is best to determine the client's beliefs. (C) is validating the idea of reference, while (D) is challenging the client.

Within several days of hospitalization, a client is repeatedly washing the top of the same table. Which initial intervention is best for the nurse to implement to help the client cope with anxiety related to this behavior? - Administer a prescribed PRN antianxiety medication. - Assist the client to identify stimuli that precipitates the ritualistic activity. - Allow time for the ritualistic behavior, then redirect the client to other activities. - Teach the client relaxation and thought stopping techniques.

- Allow time for the ritualistic behavior, then redirect the client to other activities. Initially, the nurse should allow time for the ritual (C) to prevent anxiety. (A) may help reduce the client's anxiety, but will not prevent ritualistic behavior resulting from the client's ineffective coping ability. (B) is a long-term goal of individual therapy, but is not directly related to controlling the behavior at this time. (D) lists techniques that can be used to assist the client in learning new ways of interrupting obsessive thoughts and resulting ritualistic behavior as treatment progresses.

A male client with schizophrenia who is taking fluphenazine decanoate (Prolixin decanoate) is being discharged in the morning. A repeat dose of medication is scheduled for 20 days after discharge. The client tells the nurse that he is going on vacation in the Bahamas and will return in 18 days. Which statement by the client indicates a need for health teaching? - "When I return from my tropical island vacation, I will go to the clinic to get my Prolixin injection." - "While I am on vacation and when I return, I will not eat or drink anything that contains alcohol." - "I will notify the healthcare provider if I have a sore throat or flu-like symptoms." - "I will continue to take my benztropine mesylate (Cogentin) every day."

- "When I return from my tropical island vacation, I will go to the clinic to get my Prolixin injection." Photosensitivity is a side effect of Prolixin and a vacation in the Bahamas (with its tropical island climate) increases the client's chance of experiencing this side effect. He should be instructed to avoid direct sun (A) and wear sunscreen. (B, C, and D) indicate accurate knowledge. Alcohol acts synergistically with Prolixin (B). (C) lists signs of agranulocytosis, which is also a side effect of Prolixin. In order to avoid extrapyramidal symptoms (EPS), anticholinergic drugs, such as Cogentin, are often prescribed prophylactically with Prolixin.

At the first meeting of a group of older adults at a daycare center for the elderly, the nurse asks one of the members what kinds of things she would like to do with the group. The older woman shrugs her shoulders and says, "You tell me, you're the leader." What is the best response for the nurse to make? - "Yes, I am the leader today. Would you like to be the leader tomorrow?" - "Yes, I will be leading this group. What would you like to accomplish during this time?" - "Yes, I have been assigned to be the leader of this group. I will be here for the next six weeks." - "Yes, I am the leader. You seem angry about not being the leader yourself."

- "Yes, I will be leading this group. What would you like to accomplish during this time?" Anxiety over participation in a group and testing of the leader characteristically occur in the initial phase of group dynamics. (B) provides information and focuses the group back to defining its function. (A) is manipulative bargaining. Although (C) provides information, it does not focus the group on its purpose or task. (D) is interpreting the client's feelings and is almost challenging.

. A client is receiving carbamazepine (Tegretol) for the treatment of a manic episode of bipolar disorder. What should the nurse include when planning client teaching about this medication? (Select all that apply.)

- "You may want to suck on hard candy when you get a dry mouth." - "We'll need to test your blood often during the first few weeks of therapy."

A client is started on fluphenazine. What should the nurse emphasize in the teaching about this drug? (Select all that apply.)

- "You should increase your fluid intake to help prevent constipation." - "You need to use sunscreen for any outdoor activity, no matter what time of year."

A client is admitted with a diagnosis of depression. The nurse knows that which characteristic is most indicative of depression? - Grandiose ideation. - Self-destructive thoughts. - Suspiciousness of others. - A negative view of self and the future.

- A negative view of self and the future. Negative self-image and feelings of hopelessness about the future (D) are specific indicators for depression. (A and/or C) occurs with paranoia or paranoid ideation. (B) may be seen in depressed clients, but are not always present, so (D) is a better answer than (B).

An anxious client expressing a fear of people and open places is admitted to the psychiatric unit. What is the most effective way for the nurse to assist this client? - Plan an outing within the first week of admission. - Distract her whenever she expresses her discomfort about being with others. - Confront her fears and discuss the possible causes of these fears. - Accompany her outside for an increasing amount of time each day.

- Accompany her outside for an increasing amount of time each day. The process of gradual desensitization by controlled exposure to the situation which is feared (D), is the treatment of choice in phobic reactions. (A and C) are far too aggressive for the initial treatment period and could even be considered hostile. (B) promotes denial of the problem, and gives the client the message that discussion of the phobia is not permitted.

A 45-year-old female client is admitted to the psychiatric unit for evaluation. Her husband states that she has been reluctant to leave home for the last six months. The client has not gone to work for a month and has been terminated from her job. She has not left the house since that time. This client is displaying symptoms of what condition? - Claustrophobia. - Acrophobia. - Agoraphobia. - Post-traumatic stress disorder.

- Agoraphobia. Agoraphobia (C) is the fear of crowds or being in an open place. (A) is the fear of being in closed places. (B) is the fear of high places. Remember, a phobia is an unrealistic fear which is associated with severe anxiety. (D) consists of the development of anxiety symptoms following a life event that is particularly serious and stressful (war, witnessing a child killed, etc.) and is experienced with terror, fear, and helplessness--a phobia is different.

Over a period of several weeks, one male participant of a socialization group at a community day care center for the elderly monopolizes most of the group's time and interrupts others when they are talking. What is the best action for the nurse to take in this situation? - Talk to the client outside the group about his behavior during group meetings. - Remind the client to allow others in the group a chance to talk. - Allow the group to handle the problem. - Ask the client to join another group.

- Allow the group to handle the problem. After several weeks, the group is in the working phase and the group members should be allowed to determine the direction of the group. The nurse should ignore the client's comments and allow the group to handle the situation (C). A good leader should not have separate meetings with group members (A), as such behavior is manipulative on the part of the leader. (B) is dictatorial and is not in keeping with good leadership skills. (D) is avoiding the problem. Remember, identify what phase the group is in--initial, working, or termination--this will help determine communication style.

Which have highest risk for injury? (Select all that apply)

- Amitriptyline to treat fibromyalgia pain - Amy trips on things - med can cause injury - Headache while on phenelzine - headache while on this med could indicate hypertension crisis - Taking St. John's wort with sertraline - this med is an SSRI and you CAN NOT mix with St. John's wort - Discontinuing escitalopram, the day before taking Isocarboxazid - SSRI and MAOI can't be mixed before 2 week wash out period of tapering off. - Sore throat and fatigue while on clozapine - risk for infection and injury taking this med

A young adult male client, diagnosed with paranoid schizophrenia, believes that world is trying poison him. What intervention should the nurse include in this client's plan of care? - Remind the client that his suspicions are not true. - Ask one nurse to spend time with the client daily. - Encourage the client to participate in group activities. - Assign the client to a room closest to the activity room.

- Ask one nurse to spend time with the client daily. A client with paranoid schizophrenia has difficulty with trust and developing a trusting relationship with one nurse (B) is likely to be therapeutic for this client. (A) is argumentative. Stress increases anxiety, and anxiety increases paranoid ideation; (C) would be too stressful and anxiety-promoting for a client who is experiencing pathological suspicions. (D) also might increase anxiety and stress.

The nurse is planning care for a 32-year-old male client diagnosed with HIV infection who has a history of chronic depression. Recently, the client's viral load has begun to increase rather than decrease despite his adherence to the HIV drug regimen. What should the nurse do first while taking the client's history upon admission to the hospital? - Determine if the client attends a support group weekly. - Hold all antidepressant medications until further notice. - Ask the client if he takes St. John's Wort routinely. - Have the client describe any recent changes in mood.

- Ask the client if he takes St. John's Wort routinely. St. John's Wort, an herbal preparation, is an alternative (nonconventional) therapy for depression, but it may adversely interact with medications used to treat HIV infection (C). The nurse's top priority upon admission is to determine if the client has been taking this herb concurrently with HIV antiviral drugs, which may explain the rise in the viral load. Asking about (A or D) may be helpful in gathering more data about the client's depressive state, but these issues do not have the priority of (C). (B) may be harmful to the client.

A male client is admitted to the psychiatric unit with a medical diagnosis of paranoid schizophrenia. During the admission procedure, the client looks up and states, "No, it's not MY fault. You can't blame me. I didn't kill him, you did." What action is best for the nurse to take? - Reassure the client by telling him that his fear of the admission procedure is to be expected. - Tell the client that no one is accusing him of murder and remind him that the hospital is a safe place. - Assess the content of the hallucinations by asking the client what he is hearing. - Ignore the behavior and make no response at all to his delusional statements.

- Assess the content of the hallucinations by asking the client what he is hearing. Further assessment is indicated (C). The nurse should obtain information about what the client believes the voices are telling him--they may be telling him to kill the nurse! (A) is telling the client how he feels (fearful). The nurse should leave communications open and seek more information. (B) is arguing with the client's delusion, and the nurse should never argue with a client's hallucinations or delusions, also (B) is possibly offering false reassurance. (D) is avoiding the situation and the client's needs.

The nurse is planning the care for a 32-year-old male client with acute depression. Which nursing intervention would be best in helping this client deal with his depression? - Ensure that the client's day is filled with group activities. - Assist the client in exploring feelings of shame, anger, and guilt. - Allow the client to initiate and determine activities of daily living. - Encourage the client to explore the rationale for his depression.

- Assist the client in exploring feelings of shame, anger, and guilt. Depression is associated with feelings of shame, anger, and guilt. Exploring such feelings is an important nursing intervention for the depressed client (B). If the client's day is filled with group activities (A) he might not have the opportunity to explore these feelings. (C) is a good intervention for the chronically depressed client who exhibits vegetative signs of depression. (D) is essentially asking the client "why" he is depressed--avoid "why’s" disguised as "rationale."

Patient teaching for diazepam: select all that apply:

- Avoid valerian root - Avoid muscle relaxants - Report history of reaction to midazolam

parent of four is remanded to the psychiatric unit by the court for observation. The client was arrested and charged with abusing a 2-year-old son, who is in the pediatric intensive care unit in critical condition. The nurse approaches the client for the first time. How should the nurse anticipate that the client will likely respond? (Select all that apply.)

- By denying beating the son - By avoiding talking about the situation - By exhibiting an emotional response that is inconsistent with degree of injury

When determining whether a client has anorexia nervosa or bulimia nervosa, the nurse should identify those characteristics that relate only to anorexia nervosa. (Select all that apply.)

- Cachexia - Delayed psychosexual development

At a support meeting of parents of a teenager with polysubstance dependency, a parent states, "Each time my son tries to quit taking drugs, he gets so depressed that I'm afraid he will commit suicide." The nurse's response should be based on which information? - Addiction is a chronic, incurable disease. - Tolerance to the effects of drugs causes feelings of depression. - Feelings of depression frequently lead to drug abuse and addiction. - Careful monitoring should be provided during withdrawal from the drugs.

- Careful monitoring should be provided during withdrawal from the drugs. The priority is to teach the parents that their son will need monitoring and support during withdrawal (D) to ensure that he does not attempt suicide. Although (A and C) are true, they are not as relevant to the parent's expressed concern. There is no information to support (B).

In which age groups is depression often underdiagnosed? (Select all that apply)

- Children. - Adolescents. - Older adults.

A client who is known to abuse drugs is admitted to the psychiatric unit. Which medication should the nurse anticipate administering to a client who is exhibiting benzodiazepine withdrawal symptoms? - Perphenazine (Trilafon). - Diphenhydramine (Benadryl). - Chlordiazepoxide (Librium). - Isocarboxazid (Marplan).

- Chlordiazepoxide (Librium). Librium (C), an antianxiety drug, as well as other benzodiazepines, is used in titrated doses to reduce the severity of abrupt benzodiazepine withdrawal. (A) is an antipsychotic agent. (B) is an antihistamine and antianxiety drug. (D) is an MAO inhibitor.

A new employee orientation on the policies and procedures related to the therapeutic use of seclusion stresses that seclusion is contraindicated for certain clients. (Select all that apply.)

- Suicidal - Cognitively impaired - Prone to unstable changes in cardiac status

A 52-year-old male client in the intensive care unit who has been oriented suddenly becomes disoriented and fearful. Assessment of vital signs and other physical parameters reveal no significant change and the nurse formulates the diagnosis, "Confusion related to ICU psychosis." Which intervention would be best to implement? - Move all machines away from the client's immediate area. - Attempt to allay the client's fears by explaining the etiology of his condition. - Cluster care so that brief periods of rest can be scheduled during the day. - Extend visitation times for family and friends.

- Cluster care so that brief periods of rest can be scheduled during the day. The critical care environment confronts clients with an environment which provides stressors heightened by treatment modalities that may prove to be lifesaving. These stressors can result in isolation and confusion. The best intervention is to provide the client with rest periods (C). (A) is not practical--the machinery is often lifesaving. The client is not ready for (B). Although family and friends (D) can provide a support system to the client, visits should be limited because of the critical care that must be provided.

A female client with obsessive-compulsive disorder (OCD) is describing her obsessions and compulsions and asks the nurse why these make her feel safer. What information should the nurse include in this client's teaching plan? (Select all that apply.) - Compulsions relieve anxiety. - Anxiety is the key reason for OCD. - Obsessions cause compulsions. - Obsessive thoughts are linked to levels of neurochemicals. - Antidepressant medications increase serotonin levels.

- Compulsions relieve anxiety. - Anxiety is the key reason for OCD. - Obsessive thoughts are linked to levels of neurochemicals. - Antidepressant medications increase serotonin levels. Correct choices are (A, B, D, and E). To promote client understanding and compliance, the teaching plan should include explanations about the origin and treatment options of OCD symptomology. Compulsions are behaviors that help relieve anxiety (A), which is a vague feeling related to unknown fears, that motivate behavior (B) to help the client cope and feel secure. All obsessions (C) do not result in compulsive behavior. OCD is supported by the neurophysiology theory, which attributes a diminished level of neurochemicals (D), particularly serotonin, and responds to selective serotonin reuptake inhibitors (SSRI).

A homeless person who is in the manic phase of bipolar disorder is admitted to the mental health unit. Which laboratory finding obtained on admission is most important for the nurse to report to the healthcare provider? - Decreased thyroid stimulating hormone level. - Elevated liver function profile. - Increased white blood cell count. - Decreased hematocrit and hemoglobin levels.

- Decreased thyroid stimulating hormone level. Hyperthyroidism causes an increased level of serum thyroid hormones (T3 and T4), which inhibit the release of TSH (A), so the client's manic behavior may be related to an endocrine disorder. (B, C, and D) are abnormal findings that are commonly found in the homeless population because of poor sanitation, poor nutrition, and the prevalence of substance abuse.

A nurse is making a home visit to a young male client manifesting chronic symptoms of AIDS. The nurse assesses the client for signs of altered mental health function associated with AIDS. (Select all that apply.)

- Delusions - Memory loss - Hopelessness - Paranoid thinking

People who are involved in a bioterrorism attack exhibit immediate reactions to the traumatic event. Which responses can a nurse expect in survivors during the period immediately following a traumatic event? (Select all that apply.)

- Denial - Confusion - Helplessness

Serotonin syndrome signs and symptoms

- Diarrhea * -Fever - sweaty and hot - Agitation - Palpitation - Restlessness - Anxiety - Delirium - Disorientation - Muscle twitches * Note - vitamins or other drugs can cause an increase in serotonin levels

A client is receiving substitution therapy during withdrawal from benzodiazepines. Which expected outcome statement has the highest priority when planning nursing care? - Client will not demonstrate cross-addiction. - Co-dependent behaviors will be decreased. - Excessive CNS stimulation will be reduced. - Client's level of consciousness will increase.

- Excessive CNS stimulation will be reduced. Substitution therapy with another CNS depressant is intended to decrease the excessive CNS stimulation that can occur during benzodiazepine withdrawal (C). (A, B, and D) are all appropriate outcome statements for the client described, but do not have the priority of (C).

A client with a diagnosis of schizophrenia, undifferentiated type, is being admitted to the psychiatric unit. What clinical manifestations does the nurse expect when assessing this client? (Select all that apply.)

- Excited behaviors - Loose associations - Inappropriate affect

Which elements have been shown to be beneficial when incorporated into a stress management plan? (Select all that apply)

- Exercise. - Mindfulness practices. - Balanced diet.

nurse who suspects that a newly admitted infant is the victim of child abuse assesses the parents' interaction with their baby. What parental behaviors might support the diagnosis of child abuse? (Select all that apply.)

- Exhibiting difficulty in showing concern for their child. - Procrastinating in obtaining treatment for their child's injuries

The nurse should hold the next scheduled dose of a client's haloperidol (Haldol) based on which assessment finding(s)? - Dizziness when standing. - Shuffling gait and hand tremors. - Urinary retention. - Fever of 102 F.

- Fever of 102 F. A fever (D) may indicate neuroleptic malignant syndrome (NMS), a potentially fatal complication of antipsychotics. The healthcare provider should be contacted before administering the next dose of Haldol. (A, B, and C) are all adverse effects of Haldol which can be managed.

A client who is diagnosed with schizophrenia is admitted to the hospital. The nurse assesses the client's mental status. Which assessment finding is most characteristic of a client with schizophrenia? - Mood swings. - Extreme sadness. - Manipulative behavior. - Flat affect.

- Flat affect. Disinterest, and diminished or lack of facial expression is characteristic of schizophrenia and is referred to as a flat affect (D). (A) is associated with bipolar disorder. (B) is associated with depression. (C) is usually associated with personality disorders and is often seen in clients who abuse substances.

When a nurse is admitting an older client to the mental health unit, it is important to identify any signs of dementia. What signs and symptoms denote the presence of dementia of the Alzheimer type? (Select all that apply.)

- Forgetfulness - Expressive aphasia

A 22-year-old male client is admitted to the emergency center following a suicide attempt. His records reveal that this is his third suicide attempt in the past two years. He is conscious, but does not respond to verbal commands for treatment. Which assessment finding should prompt the nurse to prepare the client for gastric lavage? - He ingested the drug 3 hours prior to admission to the emergency center. - The family reports that he took an entire bottle of acetaminophen (Tylenol). - He is unresponsive to instructions and is unable to cooperate with emetic therapy. - Those with repeated suicide attempts desire punishment to relieve their guilt.

- He is unresponsive to instructions and is unable to cooperate with emetic therapy. Because the client is unable to follow instructions, emetic therapy would be very difficult to implement and gastric lavage would be necessary (C). (A and B) should be considered in determining the course of treatment, but they are not the basis for determining if gastric lavage will be implemented. Medical treatments should never be used as "punitive" measures (D).

A male client with mental illness and substance dependency tells the mental health nurse that he has started using illegal drugs again and wants to seek treatment. Since he has a dual diagnosis, which person is best for the nurse to refer this client to first? - The emergency room nurse. - His case manager. - The clinic healthcare provider. - His support group sponsor.

- His case manager. The case manager (B) is responsible for coordinating community services, and since this client has a dual diagnosis, this is the best person to describe available treatment options. (A) is unnecessary, unless the client experiences behaviors that threaten his safety or the safety of others. (C and D) might also be useful, but it is most important at this time that a treatment program be coordinated to meet this client's needs.

A client on the psychiatric unit appears to imitate a certain nurse on the unit. The client seeks out this particular nurse and imitates the nurse's mannerisms. The nurse knows that the client is using which defense mechanism? - Sublimation. - Identification. - Introjection. - Repression.

- Identification. Identification (B) is an attempt to be like someone or emulate the personality traits of another. (A) is substituting an unacceptable feeling for one that is more socially acceptable. (C) is incorporating the values or qualities of an admired person or group into one's own ego structure. (D) is the involuntary exclusion of painful thoughts or memories from one's awareness.

An adult who has been in a gay relationship for 3 years arrives at the emergency department in a state of near panic. The client says, "My partner just left me. I'm a wreck." What should the nurse do to help the client cope with this loss? Select all that apply.

- Identify the client's support systems. - Encourage the client to talk about the situation.

The parents of a 14-year-old boy bring their son to the hospital. He is lethargic, but responsive. The mother states, "I think he took some of my pain pills." During initial assessment of the teenager, what information is most important for the nurse to obtain from the parents? - If he has seemed depressed recently. - If a drug overdose has ever occurred before. - If he might have taken any other drugs. - If he has a desire to quit taking drugs.

- If he might have taken any other drugs. Knowledge of all substances taken (C) will guide further treatment, such as administration of antagonists, so obtaining this information has the highest priority. (A and B) are also valuable in planning treatment. (D) is not appropriate during the acute management of a drug overdose.

A client, who is on a 30-day commitment to a drug rehabilitation unit, asks the nurse if he can go for a walk on the grounds of the treatment center. When he is told that his privileges do not include walking on the grounds, the client becomes verbally abusive. Which approach will the nurse take? - Call a staff member to escort the client to his room. - Tell the client to talk to his healthcare provider about his privileges. - Remind the client of the unit rules. - Ignore the client's inappropriate behavior.

- Ignore the client's inappropriate behavior. The client is trying to engage the nurse in a dispute. Ignoring the behavior (D) provides no reinforcement for the inappropriate behavior. (A) is not necessary unless the client becomes a physical threat to the nurse. (B) would be inappropriate, because it is referring the situation to the healthcare provider and is not in keeping with good health team management. Consistent limits must be established and enforced. (C) would subject the nurse to more verbal abuse because the client could use any response as an excuse to attack the nurse once again.

A 46-year-old female client has been on antipsychotic neuroleptic medication for the past three days. She has had a decrease in psychotic behavior and appears to be responding well to the medication. On the fourth day, the client's blood pressure increases, she becomes pale and febrile, and demonstrates muscular rigidity. Which action will the nurse initiate? - Place the client on seizure precautions and monitor carefully. - Immediately transfer the client to ICU. - Describe the symptoms to the charge nurse and record on the client's chart. - No action is required at this time as these are known side effects of such drugs.

- Immediately transfer the client to ICU. These symptoms are descriptive of neuroleptic malignant syndrome (NMS) which is an extremely serious/life threatening reaction to neuroleptic drugs (B). The major symptoms of this syndrome are fever, rigidity, autonomic instability, and encephalopathy. Respiratory failure, cardiovascular collapse, arrhythmias, and/or renal failure can result in death. This is an EMERGENCY situation, and the client requires immediate critical care. Seizure precautions (A) are not indicated in this situation. (C and D) do not consider the seriousness of the situation.

A 25-year-old woman is seeking outpatient counseling after thinking about suicide. The nurse realizes that there some factors place individuals at a higher risk for suicide. Which of these factors increases the risk for suicide? (Select all that apply.)

- Impulsivity - Panic attacks - Unemployment - Substance abuse

The nurse is assessing the parents of a nuclear family who are attending a support group for parents of adolescents. According to Erikson, these parents who are adapting to middle adulthood should exhibit which characteristic? - Loss of independence. - Increased self-understanding. - Isolation from society. - Development of intimate relationships.

- Increased self-understanding. Middle adulthood is characterized by self-reflection, understanding, and acceptance (B), and generativity or guidance of children. (A and C) are maladaptive behaviors in middle adulthood. Although middle-aged adults may delay or re-establish intimate relationships, (D) is initially developed during young adulthood.

The nurse is planning discharge for a male client with schizophrenia. The client insists that he is returning to his apartment, although the healthcare provider informed him that he will be moving to a boarding home. What is the most important nursing diagnosis for discharge planning? - Ineffective denial related to situational anxiety. - Ineffective coping related to inadequate support. - Social isolation related to difficult interactions. - Self-care deficit related to cognitive impairment.

- Ineffective denial related to situational anxiety. The best nursing diagnosis is (A) because the client is unable to acknowledge the move to a boarding home. (B, C, and D) are potential nursing diagnoses, but denial is most important because it is a defense mechanism that keeps the client from dealing with his feelings about living arrangements.

During a home visit to an older adult, the nurse observes a change in behavior and suspects delirium. The nurse assesses the client for one of several conditions that may have precipitated the delirium. (Select all that apply.)

- Infection - Dehydration - Urine retention

The nurse and client have entered the working phase of a therapeutic relationship. What can the nurse expect the client to do during this phase? (Select all that apply.)

- Initiate topics of discussion. - Accept limits on unacceptable behavior. - Express emotions related to transference.

The nurse is assessing a client's intelligence. Which factor should the nurse remember during this part of the mental status exam? - Acute psychiatric illnesses impair intelligence. - Intelligence is influenced by social and cultural beliefs. - Poor concentration skills suggests limited intelligence. - The inability to think abstractly indicates limited intelligence.

- Intelligence is influenced by social and cultural beliefs. Social and cultural beliefs (B) have significant impact on intelligence. Chronic psychiatric illness may impair intelligence (A), especially if it remains untreated. Limited concentration does not suggest limited intelligence (C). Difficulties with abstractions are suggestive of psychotic thinking (D), not limited intelligence.

A client tells a nurse, "I have been having trouble sleeping and feel wide awake as soon as I get into bed." Which strategies should the nurse teach the client that will promote sleep? (Select all that apply.)

- Leaving the bedroom when he is unable to sleep - Exercising in the afternoon rather than in the evening - Counting backward from 100 to 0 when his mind is racing

nurse is aware that an older adult is at risk for social isolation as a result of certain factors. (Select all that apply.)

- Living alone since a spouse's death 3 years ago - Experiencing progressive macular degeneration

A nurse is interacting with a depressed, suicidal client. What themes in the client's conversation are of most concern to the nurse? (Select all that apply.)

- Loneliness - Hopelessness

The charge nurse is collaborating with the nursing staff about the plan of care for a client who is very depressed. What is the most important intervention to implement during the first 48 hours after the client's admission to the unit? - Monitor appetite and observe intake at meals. - Maintain safety in the client's milieu. - Provide ongoing, supportive contact. - Encourage participation in activities.

- Maintain safety in the client's milieu. The most important reason for closely observing a depressed client immediately after admission is to maintain safety (B), since suicide is a risk with depression. (A, C, and D) are all important interventions, but safety is the priority.

The community health nurse talks to a male client who has bipolar disorder. The client explains that he sleeps 4 to 5 hours a night and is working with his partner to start two new businesses and build an empire. The client stopped taking his medications several days ago. What nursing problem has the highest priority? - Excessive work activity. - Decreased need for sleep. - Medication management. - Inflated self-esteem.

- Medication management. The most important nursing problem is medication management (C) because compliance with the medication regimen will help prevent hospitalization. The client is also exhibiting signs of (A, B, and C); however, these problems do not have the priority of medication management.

What are some of the family risk factors the nurse should look for when interviewing a client who is suspected of being in an abusive relationship?

- Mental health problems in the nuclear family. - Substance abuse by household members. - Family relationships that appear dysfunctional.

A nurse working on a mental health unit receives a community call from a person who is tearful and states, "I just feel so nervous all of the time. I don't know what to do about my problems. I haven't been able to sleep at night and have hardly eaten for the past 3 or 4 days." The nurse should initiate a referral based on which assessment? - Altered thought processes. - Moderate levels of anxiety. - Inadequate social support. - Altered health maintenance.

- Moderate levels of anxiety. The nurse should initiate a referral based on anxiety levels (B) and feelings of nervousness that interfere with sleep, appetite, and the inability to solve problems. The client does not report symptoms of (A) or evidence of (C). There is not enough information to initiate a referral based on (D).

An older client has been prescribed an atypical antipsychotic medication. Which nursing interventions demonstrate that the nurse has determined the client's risk for injury? (Select all that apply.)

- Monitoring the pulse for an irregular rhythm - Assessing the temperature to determine the possibility of an infection

A client with schizophrenia who has type II (negative) symptoms is prescribed risperidone (Risperdal). Which outcomes indicate that the medication has minimized these symptoms? (Select all that apply.)

- More interest is shown in unit activities. - The client performs activities of daily living independently.

An older client is admitted to the hospital with the diagnosis of dementia of the Alzheimer type and depression. Which signs of depression does the nurse identify? (Select all that apply.)

- Neglect of personal hygiene - Increased appetite - "I don't know" answers to questions - "I can't remember" answers to questions

When presenting a workshop on adolescent suicide, a community health nurse identifies risk factors. (Select all that apply.)

- Victim of family violence - Dependence on alcohol, drugs, or both - Uncertainty related to sexual orientation - Repeated demonstration of poor impulse control

A client who is being treated with lithium carbonate for bipolar disorder develops diarrhea, vomiting, and drowsiness. What action should the nurse take? - Notify the healthcare provider immediately and prepare for administration of an antidote. - Notify the healthcare provider of the symptoms prior to the next administration of the drug. - Record the symptoms as normal side effects and continue administration of the prescribed dosage. - Hold the medication and refuse to administer additional amounts of the drug.

- Notify the healthcare provider of the symptoms prior to the next administration of the drug. Early side effects of lithium carbonate (occurring with serum lithium levels below 2.0 mEq per liter) generally follow a progressive pattern beginning with diarrhea, vomiting, drowsiness, and muscular weakness. At higher levels, ataxia, tinnitus, blurred vision, and large dilute urine output may occur. (B) is the best choice. Although these are expected symptoms, the healthcare provider should be notified prior to the next administration of the drug. (A, C, and D) would not reflect good nursing judgment.

On admission to a residential care facility, an elderly female client tells the nurse that she enjoys cooking, quilting, and watching television. Twenty-fours after admission, the nurse notes that the client is withdrawn and isolated. It is best for the nurse to encourage this client to become involved in which activity? - Clean the unit kitchen cabinets. - Participate in a group quilting project. - Watch television in the activity room. - Bake a cake for a resident's birthday.

- Participate in a group quilting project. Peer interaction in a group activity (B) will help to prevent social isolation and withdrawal. (A, C, and D) are activities that can be accomplished alone, without peer interaction.

A nurse teaches dietary guidelines to a client who will be receiving tranylcypromine sulfate (Parnate), a monoamine oxidase inhibitor (MAOI). The client compiles a list of foods to avoid. Which foods included on the list indicate that the teaching has been effective? (Select all that apply.)

- Pepperoni pizza - Bologna sandwich

The nurse should include which interventions in the plan of care for a severely depressed client with neurovegetative symptoms? (Select all that apply.) - Permit rest periods as needed. - Speaking slowly and simply. - Place the client on suicide precautions. - Allow the client extra time to complete tasks. - Observe and encourage food and fluid intake. - Encourage mild exercise and short walks on the unit

- Permit rest periods as needed. - Speaking slowly and simply. - Allow the client extra time to complete tasks. - Observe and encourage food and fluid intake. - Encourage mild exercise and short walks on the unit (A, B, D, E, and F) should be included in this client's plan of care because these measures promote the client's comfort and well-being. Neurovegetative symptoms accompany the mood disorder of depression and include physiological disruptions, such as anorexia, constipation, sleep disturbance, and psychomotor retardation. Suicidal ideation (C) does not usually accompany the neurovegetative state because the client does not have the energy or high level of anxiety associated with a suicide attempt.

The nurse should include which interventions in the plan of care for a severely depressed client with neurovegetative symptoms? (Select all that apply.)

- Permit rest periods as needed. - Speaking slowly and simply. - Allow the client extra time to complete tasks. - Observe and encourage food and fluid intake. - Encourage mild exercise and short walks on the unit

When intimate partner violence (IPV) is suspected, the nurse plays an important role as an advocate for the victim. The advocate role includes several important components. (Select all that apply.)

- Planning for future safety - Validating the experiences - Promoting access to community services

When talking with a client who has alcoholism, the nurse notes that the client becomes irritable, makes excuses, and blames family and friends for the drinking problem. Which defense mechanisms does the nurse conclude that the client is using? (Select all that apply.)

- Projection - Rationalization

A 27-year-old female client is admitted to the psychiatric hospital with a diagnosis of bipolar disorder, manic phase. She is demanding and active. Which intervention should the nurse include in this client's plan of care? - Schedule her to attend various group activities. - Reinforce her ability to make her own decisions. - Encourage her to identify feelings of anger. - Provide a structured environment with little stimuli.

- Provide a structured environment with little stimuli. Clients in the manic phase of a bipolar disorder require decreased stimuli and a structured environment (D). Plan noncompetitive activities that can be carried out alone. (A) is contraindicated; stimuli should be reduced as much as possible. Impulsive decision-making is characteristic of clients with bipolar disorder. To prevent future complications, the nurse should monitor these clients' decisions and assist them in the decision-making process (B). (C) is more often associated with depression than with bipolar disorder.

Which nursing activities are specifically focused on achieving Healthy People 2010's mental health objectives? (Select all that apply.)

- Providing suicide screening for a senior citizens group - Initiating outpatient services for homeless schizophrenic adults - Teaching stress-management techniques to those housed in the local jail - Advocating for culturally competent mental health care within each state

On admission, a highly anxious client is described as delusional. The nurse understands that delusions are most likely to occur with which class of disorder? - Neurotic. - Personality. - Anxiety. - Psychotic.

- Psychotic. Delusions are false beliefs associated with psychotic behavior, and psychotic persons are not in touch with reality (D). (A, B, and C) are mental health disorders which are not associated with a break in reality, nor with hallucinations (false sensations such as hearing, or seeing) or delusions (false beliefs).

A female client refuses to take an oral hypoglycemic agent because she believes that the drug is being administered as part of an elaborate plan by the Mafia to harm her. Which nursing intervention is most important to include in this client's plan of care? - Reassure the client that no one will harm her while she is in the hospital. - Ask the healthcare provider to give the client the medication. - Explain that the diabetic medication is important to take. - Reassess client's mental status for thought processes and content.

- Reassess client's mental status for thought processes and content. The most important intervention is to reassess the client's mental status (D) and to take further action based on the findings of this assessment. Attempting to reassure the client (A) is in effect arguing with the client's delusions and could escalate an already anxious situation. Collaborating about diabetic care (B and C) is not likely to help change the client's false beliefs.

A client with bipolar disorder on the mental health unit becomes loud, and shouts at one of the nurses, "You fat tub of lard! Get something done around here!" What is the best initial action for the nurse to take? - Have the orderly escort the client to his room. - Tell the client his healthcare provider will be notified if he continues to be verbally abusive. - Redirect the client's energy by asking him to tidy the recreation room. - Call the healthcare provider to obtain a prescription for a sedative.

- Redirect the client's energy by asking him to tidy the recreation room. Distracting the client, or redirecting his energy (C), prevents further escalation of the inappropriate behavior. (A) could result in escalating the abuse and unnecessarily involve another staff member in the abusive situation. (B) is a threat and is using a health team member (healthcare provider) as the threat. (D) may be indicated if the behavior escalates, but, at this time, the best initial action is (C).

Which of the following interventions will assist in creating and maintaining a therapeutic environment on an acute care mental health unit? (Select all that apply.)

- Reorienting clients to the rules of the unit whenever necessary - Providing a posted schedule of unit activities - Monitoring each client for the potential of aggressive behavior - Encouraging the clients to take an active role in planning the unit's activities

A male client is admitted to the mental health unit because he was feeling depressed about the loss of his wife and job. The client has a history of alcohol dependency and admits that he was drinking alcohol 12 hours ago. Vital signs are: temperature, 100 F, pulse 100, and BP 142/100. The nurse plans to give the client lorazepam (Ativan) based on which priority nursing diagnosis? - Risk for injury related to suicidal ideation. - Risk for injury related to alcohol detoxification. - Knowledge deficit related to ineffective coping. - Health seeking behaviors related to personal crisis.

- Risk for injury related to alcohol detoxification. The most important nursing diagnosis is related to alcohol detoxification (B) because the client has elevated vital signs, a sign of alcohol detoxification. Maintaining client safety related to (A) should be addressed after giving the client Ativan for elevated vital signs secondary to alcohol withdrawal. (C and D) can be addressed when immediate needs for safety are met.

Which diet selection by a client who is depressed and taking the MAO inhibitor tranylcypromine sulfate (Parnate) indicates to the nurse that the client understands the dietary restrictions imposed by this medication regimen? - Hamburger, French fries, and chocolate milkshake. - Liver and onions, broccoli, and decaffeinated coffee. - Pepperoni and cheese pizza, tossed salad, and a soft drink. - Roast beef, baked potato with butter, and iced tea.

- Roast beef, baked potato with butter, and iced tea. Only (D) contains no tyramine. Tyramine in foods interacts with MAOI in the body causing a hypertensive crisis which is life-threatening, and Parnate is classified as an MAOI antidepressant. Some items in (A, B, and C) contain tyramine and would not be permitted for a client taking Parnate.

The nurse observes a client who is admitted to the mental health unit and identifies that the client is talking continuously, using words that rhyme but that have no context or relationship with one topic to the next in the conversation. This client's behavior and thought processes are consistent with which syndrome? - Dementia. - Depression. - Schizophrenia. - Chronic brain syndrome.

- Schizophrenia. The client is demonstrating symptoms of schizophrenia (C), such as disorganized speech that may include word salad (communication that includes both real and imaginary words in no logical order), incoherent speech, and clanging (rhyming). Dementia (A) is a global impairment of intellectual (cognitive) functions that may be progressive, such as Alzheimer's or organic brain syndrome (D). Depression (C) is typified by psychomotor retardation, and the client appears to be slowed down in movement, in speech, and would appear listless and disheveled.

A 30-year-old sales manager tells the nurse, "I am thinking about a job change. I don't feel like I am living up to my potential." Which of Maslow's developmental stages is the sales manager attempting to achieve? - Self-Actualization. - Loving and Belonging. - Basic Needs. - Safety and Security.

- Self-Actualization. Self-actualization is the highest level of Maslow's development stages, which is an attempt to fulfill one's full potential (C). (B) is identifying support systems. (C) is the first level of Maslow's developmental stages and is the foundation upon which higher needs rest. Individuals who feel safe and secure (D) in their environment perceive themselves as having physical safety and lack fear of harm.

A depressed client is receiving paroxetine (Paxil). The nurse monitors this client for the side effects associated with this drug. (Select all that apply.)

- Sexual dysfunction - Insomnia and restlessness - Hypertension or hypotension

A 40-year-old male client diagnosed with schizophrenia and alcohol dependence has not had any visitors or phone calls since admission. He reports he has no family that cares about him and was living on the streets prior to this admission. According to Erikson's theory of psychosocial development, which stage is the client in at this time? - Isolation. - Stagnation. - Despair. - Role confusion.

- Stagnation. The client is in Erikson's "Generativity vs. Stagnation" stage (age 24 to 45), and meeting the task includes maintaining intimate relationships and moving toward developing a family (B). (A) occurs in young adulthood (age 18 to 25), (C) occurs in maturity (age 45 to death), and (D) occurs in adolescence (age 12 to 20). These are all stages that occur if individuals are not successfully coping with their psychosocial developmental stage.

A 19-year-old female client with a diagnosis of anorexia nervosa wants to help serve dinner trays to other clients on a psychiatric unit. What action should the nurse take? - Encourage the client's self-motivation by asking her to pass trays for the rest of the week. - Provide an additional challenge by asking the client to help feed the older clients. - Suggest another way for this client to participate in the unit's activities. - Tell the client that hospital guidelines allow only staff to pass the trays.

- Suggest another way for this client to participate in the unit's activities. Clients with anorexia should not be allowed to plan or prepare food for unit activities and their desires to do so should be redirected (C). These clients gain pleasure from providing others with food and watching them eat. Such behaviors reinforce their perception of self-control. (A and B) are contraindicated for a client with anorexia nervosa. (D) avoids addressing the problem, so it is best to suggest another activity for the client.

A psychologist tells the nurse that a client is the genital stage of Freudian psychosexual development. Assuming normal development, how old should the nurse expect the client to be?

12 years or up

An elderly female client with advanced dementia is admitted to the hospital with a fractured hip. The client repeatedly tells the staff, "Take me home. I want my Mommy." Which response is best for the nurse to provide? - Orient the client to the time, place, and person. - Tell the client that the nurse is there and will help her. - Remind the client that her mother is no longer living. - Explain the seriousness of her injury and need for hospitalization.

- Tell the client that the nurse is there and will help her. Those with dementia often refer to home or parents when seeking security and comfort. The nurse should use the techniques of "offering self" and "talking to the feelings" to provide reassurance (B). Clients with advanced dementia have permanent physiological changes in the brain (plaques and tangles) that prevent them from comprehending and retaining new information, so (A, C, and D) are likely to be of little use to this client and do not help the client's emotional needs.

Based on non-compliance with the medication regimen, an adult client with a medical diagnosis of substance abuse and schizophrenia was recently switched from oral fluphenazine HCl (Prolixin) to IM fluphenazine decanoate (Prolixin Decanoate). What is most important to teach the client and family about this change in medication regimen? - Signs and symptoms of extrapyramidal effects (EPS). - Information about substance abuse and schizophrenia. - The effects of alcohol and drug interaction. - The availability of support groups for those with dual diagnoses.

- The effects of alcohol and drug interaction. Alcohol enhances the EPS side effects of Prolixin. The half-life of Prolixin PO is 8 hours, whereas the half-life of the Prolixin Decanoate IM is 2 to 4 weeks. That means the side effects of drinking alcohol are far more severe when the client drinks alcohol after taking the long-acting Prolixin Decanoate IM. (A, B, and D) provide valuable information and should be included in the client/family teaching, but they do not have the priority of (C).

A nurse working in the emergency room of a children's hospital admits a child whose injuries could have resulted from abuse. Which statement most accurately describes the nurse's responsibility in cases of suspected child abuse? - The nurse should obtain objective data such as x-rays before reporting suspicions to the authorities. - The nurse should confirm any suspicions of child abuse with the healthcare provider before reporting to the authorities. - The nurse should report any case of suspected child abuse to the nurse in charge. - The nurse should note in the client's record any suspicions of child abuse so that a history of such suspicions can be tracked.

- The nurse should report any case of suspected child abuse to the nurse in charge. It is the nurse's legal responsibility to report all suspected cases of child abuse. Notifying the charge nurse starts the legal reporting process (C).

The nurse suspects child abuse when assessing a 3-year-old boy and noticing several small, round burns on his legs and trunk that might be the result of cigarette burns. Which parental behavior provides the greatest validation for such suspicions? - The parents' explanation of how the burns occurred is different from the child's explanation of how they occurred. - The parents seem to dismiss the severity of the child's burns, saying they are very small and have not posed any problem. - The parents become very anxious when the nurse suggests that the child may need to be admitted for further evaluation. - The parents tell the nurse that the child was burned in a house fire which is incompatible with the nurse's observation of the type of burn.

- The parents tell the nurse that the child was burned in a house fire which is incompatible with the nurse's observation of the type of burn. (D) provides the most validation. The parent's explanation (subjective data) is incompatible with the objective data (small round burns on the legs and trunk). (A) provides only subjective data, and the child's explanation could be influenced by factors such as age, fear, or imagination. The parent's apparent lack of concern (B) is inconclusive, but the nurse's opinion of the parents' reaction is subjective and could be wrong. (C) might provide a clue that child abuse occurred, but the nurse must remember that most parents are anxious about their child being hospitalized.

Which statement about contemporary mental health nursing practice is accurate? - There is one approved theoretical framework for psychiatric nursing practice. - Psychiatric nursing has yet to be recognized as a core mental health discipline. - Contemporary practice of psychiatric nursing is primarily focused on inpatient care. - The psychiatric nursing client may be an individual, family, group, organization, or community.

- The psychiatric nursing client may be an individual, family, group, organization, or community. Mental health nursing is not only concerned with one-on-one interactions. Psychiatric stressors can impact and be reflected in the overall direction, activities, and responses involving families, groups, and entire communities (D). (A, B, and C) are incorrect statements about the status of mental health nursing.

A nurse notes that a client in the detoxification unit is exhibiting early signs of alcohol withdrawal. What clinical manifestations might the nurse have noticed? (Select all that apply.)

- Tremors - Anorexia

A client with a history of chronic alcoholism was admitted to a surgical unit after surgery to repair a severely fractured right ankle. The nurse is concerned that the client is experiencing manifestations of acute alcohol withdrawal when certain documentation and assessment data from the last 6 hours seem to indicate this problem. (Select all that apply.)

- Tremors in both hands make it difficult for the client to hold a cup. - The client's bed linens and pajamas had to be changed during the night as a result of increased diaphoresis. - The usually cooperative client becomes verbally abusive when asked to lower the volume of the television.

A health care provider prescribes carbamazepine (Tegretol) for a client. The nurse teaches the client about effects of the drug that should be reported to the health care provider. Which effects identified by the client as cause to call the provider indicate an understanding of the teaching? (Select all that apply.)

- Unusual bleeding or bruising - Nausea or vomiting

A nurse provides crisis intervention for a client who recently left her husband because of physical abuse. Which client behaviors indicate to the nurse that the therapy has been successful? (Select all that apply.)

- Utilizes healthier coping skills - Describes the current situation realistically

A nurse is in the process of developing a therapeutic relationship with a client who has an addiction problem. What client communication permits the nurse to conclude that they are making progress in the working stage of the relationship? (Select all that apply.)

- Verbalizes difficulty identifying personal strengths - Acknowledges the effects of the addiction on the family - Addresses how the addiction has contributed to family distress

The nurse is assessing a client who expresses feeling overwhelmed with the care of an elderly parent. Which question best uncovers the client's perception of this event as a stressor? (Select all that apply)

- What impact does this stressor have on your life? - What do you believe is causing you stress right now?

A male adolescent is admitted with bipolar disorder after being released from jail for assault with a deadly weapon. When the nurse asks the teen to identify his reason for the assault, he replies, "Because he made me mad!" Which goal is best for the nurse to include in the client's plan of care? The client will - outline methods for managing anger. - control impulsive actions toward self and others. - verbalize feelings when anger occurs. - recognize consequences for behaviors exhibited.

- control impulsive actions toward self and others. Those with bipolar disorder often exhibit poor impulse control, and the most important goal for this client at this time is to learn to control impulsive behavior (B) so that he can avert the social consequences related to such behaviors. (A, C, and D) are important goals, but they do not address the acute issue of impulse control, which is necessary to reduce the likelihood of harming self or others.

A woman brings her 48-year-old husband to the outpatient psychiatric unit and describes his behavior to the admitting nurse. She states that he has been sleepwalking, cannot remember who he is, and exhibits multiple personalities. The nurse knows that these behaviors are often associated with - dissociative disorder. - obsessive-compulsive disorder. - panic disorder. - post-traumatic stress syndrome.

- dissociative disorder. Sleepwalking, amnesia, and multiple personalities are examples of detaching emotional conflict from one's consciousness, which is the definition of a dissociative disorder (A). (B) is characterized by persistent, recurrent intrusive thoughts or urges (obsessions) that are unwilled and cannot be ignored, and provoke impulsive acts (compulsions) such as handwashing. (C) is an acute attack of anxiety characterized by personality disorganization. (D) is re-experiencing a psychologically terrifying or distressing event that is outside the usual range of human experience, such as war, rape, etc.

A 35-year-old male client who has been hospitalized for two weeks for chronic paranoia continues to state that someone is trying to steal his clothing. The most appropriate action for the nurse to take is to - encourage the client to actively participate in assigned activities on the unit. - place a lock on the client's closet. - ignore the client's paranoid ideation to extinguish these behaviors. - explain to the client that his suspicions are false.

- encourage the client to actively participate in assigned activities on the unit. Diverting the client's attention from paranoid ideation and encouraging him to complete assignments can be helpful in assisting him to develop a positive self-image (A). The client's problem is not security, and (B) actually supports his paranoid ideation. (C) is not correct because ignoring the client's symptoms may lower his self-esteem. The nurse should not argue with the client about his delusions (D), and should not try to reason with the client regarding his paranoid ideation.

A 35-year-old male client on the psychiatric unit of a general hospital believes that someone is trying to poison him. The nurse understands that a client's delusions are most likely related to his - early childhood experiences involving authority issues. - anger about being hospitalized. - low self-esteem. - phobic fear of food.

- low self-esteem. Psychotic clients have difficulty with trust and have low self-esteem (C). Nursing care should be directed at building trust and promoting positive self-esteem. Activities with limited concentration and no competition should be encouraged in order to build self-esteem. (A, B, and D) are not specifically related to the development of delusions.

A child is brought to the emergency room with a broken arm. Because of other injuries, the nurse suspects the child may be a victim of abuse. When the nurse tries to give the child an injection, the child's mother becomes very loud and shouts, "I won't leave my son! Don't you touch him! You'll hurt my child!" What is the best interpretation of the mother's statements? The mother is - regressing to an earlier behavior pattern. - sublimating her anger. - projecting her feelings onto the nurse. - suppressing her fear.

- projecting her feelings onto the nurse. Projection is attributing one's own thoughts, impulses, or behaviors onto another--it is the mother who is probably harming the child and she is attributing her actions to the nurse (C). The mother may be immature, but (A) is not the best description of her behavior. (B) is substituting a socially acceptable feeling for an unacceptable one. These are not socially acceptable feelings. The mother may be suppressing her fear (D) by displaying anger, but such an interpretation cannot be concluded from the data presented.

Clozapine can cause

-Agranulocytosis, therefore, WBC count of < 3000 is contraindicated. -Risk for infection goes up while on this med

Buproprion (Wellbutrin, Zyban)

-Atypical Antidepressant -Depression and helps people quit smoking -Side effects - insomnia, headache, weight loss -Teach patient to never miss doses

Trazodone

-Atypical antidepressant -Remember TraZZZadone makes you sleepy -Must avoid ALL alcohol and other sedatives (benzos and antihistamines)

Anxiolytics medications

-Benzodiazepines - sedatives (dangerous) -Barbiturates - sedatives (dangerous) **Making vitals low and slow ^** -Buspirone - you can still drive a BUS on buspirone because its not sedating

Those at highest risk for lithium toxicity are those with

-Decreased renal function -Kidneys are the washing machines of the body - washing the blood out from medications and wastes so any decrease in renal function usually sets the stage for toxicity

What are anxiolytics?

-Drugs that alleviate the symptoms of anxiety. -ANXIolytics

Carbamazepine

-Given for bipolar, trigeminal neuralgia (neuropathic pain) and seizures -Can cause Leukopenia - increased risk for massive infection as it lowers WBC -Report fever and sore throat -Oral contraceptives are ineffective with this med - other forms of birth control are needed

Benzodiazepines

-LAM and PAM -AlprazoLAM, MidazoLAM -TemazePAM, ClonazePAM -Benzos act fast within minutes but highly addictive and hard to come off of. -Remember - two crazy girls pam and lam driving a benz

Lithium (big med for HESI and NCLEX)

-LONG term tx for Bipolar and schizoaffective disorder -Remember - think of a lithium battery that lasts a LONG time

Pt teaching for barbiturates

-Last longer in the body but takes longer to get out of the body -Higher risk for toxicity leading to HYPOtension and respiratory depression -If a patient is on phenoBARbital and has low BP and increased sedation - ASSESS! -Remember - you went to the BAR and now you're sedated and sleepy and now you're going to be locked behind BARS for a long time (lasts in system a long time)

Key points to remember:

-Orthostatic hypotension - slow position changes -Amitriptyline - Amy trips on things - slow position changes -Urinary retention - imipramine - inhibit my peeing -Never take with MAOI - 2 week washout period. -NO MAOI + antidepressants (TCA, SSRI, SNRI)

Lithium lets go of fluid so NEVER give it if

-Patient is dehydrated -Patient is Hyponatremia (below 135 mEq/L)

Be cautious with these patients when watching out for toxicity

-Patients with kidney disease -Elderly patients who naturally have decreased kidney function

MAOIs med names

-Phenelzine (Nardrill) -Selegiline -Isocarboxazid

NEVER mix SSRI with

-St. John's wort -MAOI -Tramadol

Things to teach patients who are taking Benzodiazepine

-Take it at bedtime and DON'T skip doses -Always taper off never abruptly stop -STOP drinking ALL alcohol (test questions will trick you with the word "decrease") -Antidote for benzos is flumazenil

What is Buspirone?

-anxiolytic - non sedating anxiety med -Super easy to quit and SLOW acting -Atypical anxiolytic -Remember - the bus is going so slow (takes a long time to kick in -Remember - you can still drive a BUS when you're taking BUSpirone because its not sedating

What are barbiturates?

-drugs that depress the activity of the central nervous system, reducing anxiety but impairing memory and judgment

TCAs

-tricyclic antidepressants -Helps with depression, anxiety, and pain (neuropathy or neuropathic pain with diabetes and fibromyalgia)

Normal lithium levels

0.6-1.2 mEq/L

A terminally ill client is moving gradually toward resolution of feelings about impending death. In a plan of care based on Elisabeth Kübler-Ross' research, the nurse should use nonverbal interventions after having assessed that the client is in the:

Acceptance stage

A psychiatric nurse understands that a situational crisis usually resolves within:

1 to 2 months

4 rules to remember about antidepressants

1. Increase risk of suicide - antidepressants can increase suicidal thoughts in first few weeks of treatment. (highest in ages 18-24) 2. SLOW onset and always SLOW taper off 3. Never mix with antidepressants: -SSRI and St. John's Wort -MAOI with ANY other antidepressant (TCA, SSRI, SNRI) - could lead to serotonin syndrome. 2 weeks wash out period is needed. NEVER start a new antidepressant while tapering off an MAOI. 4. Remember that all psych drugs: -Decrease BP (slow position changes) -Cause weight changes - usually weight gain

Pt is at a toxic range of lithium levels at

1.5 Normal range - 0.6-1.2 mEq/L

The nurse is admitting a client with a history of bipolar disorder. The nurse determines that the client is in the manic phase. Which signs and symptoms contribute to the nurse's conclusion? Select all that apply. 1 Irritability 2 Grandiosity 3 Pressured speech 4 Thought blocking 5 Psychomotor retardation

123 Irritability and emotional lability, fluctuating between euphoria and anger, are commonly associated with mania. An inflated self-esteem and delusions of grandeur represent mood-congruent psychotic features of mania; clients believe that they possess extraordinary talents, that they are famous, or that they know someone famous. They are extremely talkative and their speech is rapid, with an urgent quality (pressured speech); they rapidly change subjects and have flight of ideas and racing thoughts. Thought blocking occurs most often with schizophrenia; the client loses the train of thinking and is unable to retrieve the previous thought. Psychomotor retardation is related to depression; clients with mania move fast, pace, fidget, and are rarely still.

The nurse is leading a relapse-prevention group for clients who experience bipolar disorder manic episodes. Which strategies should the nurse teach to help prevent or identify impending relapse? Select all that apply. 1 Watch for changes in libido. 2 Keep dietary changes to a minimum. 3 Maintain a regular sleeping schedule. 4 Plan multiple varied activities every day. 5 Monitor yourself for increased irritability or mood instability

1235Increased sex drive often indicates the beginning of a manic episode. Changes in the eating pattern can trigger a manic episode. Changes in the sleeping pattern may increase anxiety and trigger a manic episode. An elevated, expansive, or irritable mood often indicates the beginning of a manic episode. Too many activities may be too stimulating and precipitate a manic episode. Simple, repetitive routines should be followed to limit change or anxiety.

A 37-year-old man has been remanded by the court to the drug rehabilitation unit of a psychiatric facility for treatment of cocaine addiction. When taking his health history, what characteristics should the nurse expect the client to report? Select all that apply. 1 Anxiety 2 Weight loss 3 Palpitations 4 Sedentary habits 5 Difficulties with speech

123Cocaine, an alkaloid stimulant, can precipitate anxiety, hypervigilance, euphoria, agitation, and anger. The loss of appetite and increased metabolic rate associated with cocaine addiction both promote weight loss. Cocaine is a stimulant that has cardiac effects such as tachycardia and dysrhythmias. Sedentary habits are associated with barbiturate addiction. Difficulties with speech are associated with other addictions such as alcohol and methadone.

Sildenafil (Viagra) is prescribed for a man with erectile dysfunction. A nurse teaches the client about common side effects of this drug, which include which of the following? Select all that apply. 1 Flushing 2 Headache 3 Dyspepsia 4 Constipation 5 Hypertension

123Flushing is a common central nervous system response to sildenafil (Viagra). Headache is a common central nervous system response to sildenafil. Dyspepsia is a common gastrointestinal response to sildenafil. Diarrhea, not constipation, is a common gastrointestinal response to sildenafil. Hypotension, not hypertension, is a cardiovascular response to sildenafil. It should not be taken with antihypertensives and nitrates, because drug interactions can precipitate cardiovascular collapse.

A psychiatric nurse is working at a community mental health clinic. Which activity demonstrates that the nurse knows the importance of engaging in effective self-awareness?

Accepting a client's decision to refuse electroconvulsive therapy as a treatment for chronic depression

A client in the early dementia stage of Alzheimer disease is admitted to a long-term care facility. Which activities must the nurse initiate? Select all that apply. 1 Weighing the client once a week 2 Having specialized rehabilitation equipment available 3 Keeping the client in pajamas and robe most of the day 4 Establishing a schedule with periods of rest after activities 5 Reviewing the client's weekly budget and use of community resources 6 Setting up a plan for weekly entertainment through a senior citizens group

124 Monitoring weight is an objective way to assess nutritional status. Specialized equipment can facilitate the client's participation in self-care. Incorporating rest periods into the client's day prevents fatigue and energizes the client for the next period of activity. The client needs to wear clothes to help maintain a positive view of self. It is not appropriate to review budgeting and use of community resources with a client in the early dementia stage of Alzheimer disease; these activities may produce frustration, withdrawal, or self-absorption. A client in the early dementia stage of Alzheimer disease is usually unable to participate in or travel with a senior citizen group.

When a recently hospitalized client has a tentative diagnosis of opioid addiction, the nurse should assess the client for signs and symptoms related to opioid withdrawal. List them in the order that they will occur as the client progresses through withdrawal. 1. Runny nose 2. Severe bone pain 3. Flulike syndromes 4. Return of appetite

1324 When opioids, which are central nervous system depressants, are withdrawn initially, the client will experience a runny nose (rhinorrhea), tearing (lacrimation), diaphoresis, yawning, and irritability. As withdrawal progresses, rebound hyperexcitability precipitates muscle twitching, restlessness, hypertension, tachycardia, temperature irregularities, tremors, and loss of appetite. Finally flulike symptoms, insomnia, and yawning occur. Once withdrawal is complete the appetite returns, vital signs become stable, and other withdrawal signs and symptoms subside and eventually disappear.

A psychologist has been a client on a mental health unit for 3 days. The client has questioned the authority of the treatment team, advised other clients that their treatment plans are wrong, and been disruptive in group therapy. What is the most appropriate nursing intervention?

Accepting that the client is unable to control this behavior and setting appropriate limits

When caring for clients with the diagnosis of anorexia nervosa or bulimia nervosa, it is important that the nurse understand the sociocultural influences related to eating disorders in the United States. What are these influences? Select all that apply. 1 Diet industry 2 Fashion trends 3 Fast food industry 4 Over-the-counter medications 5 Competitive women's athletics

125 Weight management moved into the mainstream in the 1950s and increased its momentum with the fitness industry in the 1980s and 1990s. In the new century, women are constantly bombarded by the media with products and programs that are designed to help them attain the perfect body, which for most women is unrealistic. Since the 1960s the trend in fashion has been toward thinness, with fabrics that cling and styles that reveal the body. Print and movie media, including advertising, are focused on a thin, perfect ideal that is unattainable for most women. Several women's sports, such as gymnastics and figure skating, emphasize low body weights, and so does ballet. These demands may lead to eating disorders in girls and women who wish to compete. Although some people with bulimia nervosa may eat fast food, the fast food industry is unrelated to the origin of anorexia nervosa or bulimia nervosa. Although some people with eating disorders use over-the-counter medications, particularly laxatives, over-the-counter medications are unrelated to the origin of eating disorders.

The nurse interviews a young female client with anorexia nervosa to obtain information for the nursing history. What will the client's history most likely reveal? Select all that apply. 1 Ritualistic behaviors 2 Desire to improve her self-image 3 Supportive mother-daughter relationship 4 Low achievement in school and little concern for grades 5 Satisfaction with and a desire to maintain her current weight

12Clients with anorexia nervosa frequently have a history of ritualistic behaviors, rigidity, and meticulousness, reflecting a need for control. Clients with anorexia nervosa have a disturbed self-image and always see themselves as fat and needing further weight loss. The relationship between mother and daughter is often not supportive but instead conflicted. Usually there is high achievement and great concern about grades. Usually there is dissatisfaction with weight and a desire to lose weight.

What clinical findings may be expected when a nurse assesses an individual with an anxiety disorder? Select all that apply. 1 Worrying about a variety of issues 2 Acting out with antisocial behavior 3 Converting the anxiety into a physical symptom 4 Displacing the anxiety onto a less threatening object 5 Demonstrating behavior common to an earlier stage of development

1345Excessive anxiety and worry about a number of events, topics, or activities for a 6-month duration are the hallmark of generalized anxiety disorder. Converting anxiety into a physical symptom is an example of a conversion disorder, which eases anxiety. Displacing the anxiety onto a less threatening object, which eases anxiety, is typical of a phobic disorder. Regression is an attempt during periods of stress to return to behavior that has been satisfying and is appropriate at an earlier stage of development. Acting out anxiety with antisocial behavior is most commonly found in individuals with personality rather than anxiety disorders.

A nurse is working with an adolescent client with conduct disorder. Which strategies should the nurse implement while working on the goal of increasing the client's ability to meet personal needs without manipulating others? Select all that apply. 1 Provide physical outlets for aggressive feelings. 2 Discuss how others can precipitate anxiety. 3 Establish a contract regarding manipulative behavior. 4 Develop activities that provide opportunities for success. 5 Encourage the client to verbalize negative feelings to others.

134Channeling energy to healthy physical activities can decrease violent behavior. A behavioral contract is used to reinforce problem solving and encourage the use of social skills. Successful experiences improve the client's self-esteem and should decrease the manipulative behavior. Clients with conduct disorders tend to generate stress for others, not the other way around. Verbalization of negative feelings to others can often escalate and result in antisocial or acting out behavior.

A nurse is assigned to care for a group of clients who have been found to have depression. Which clinical manifestations does the nurse anticipate? Select all that apply. 1 Neglect of personal hygiene 2 Labiality of affect 3 Specific food cravings 4 "I don't know" answers to questions 5 Apathetic response to the environment

145Clients with depression are uninterested in their appearance because of low self-esteem. "I don't know" answers to questions type response requires little thought or decision-making, typical of depression. These clients' sense of futility leads to a lack of response to the environment. With depression there is little or no emotional involvement and therefore little alteration in affect. Clients with depression are uninterested in food of any kind.

A nurse is assessing a client with a diagnosis of primary insomnia. Which findings from the client's history may be the cause of this disorder? Select all that apply. 1 Chronic stress 2 Severe anxiety 3 Generalized pain 4 Excessive caffeine 5 Chronic depression 6 Environmental noise/distractor

146Acute or primary insomnia is caused by emotional or physical stress not related to the direct physiologic effects of a substance or illness. Excessive caffeine intake can cause disruptive sleep hygiene; caffeine is a stimulant that inhibits sleep. Environmental noise causes physical and emotional discomfort and is therefore related to primary insomnia. Severe anxiety is usually related to a psychiatric disorder and therefore causes secondary insomnia. Generalized pain is usually related to a medical or neurologic problem and therefore causes secondary insomnia. Chronic depression is usually related to a psychiatric disorder and therefore causes secondary insomnia.

A nurse who works in a mental health facility determines that the priority nursing intervention for a newly admitted client with bulimia nervosa is to: 1 Check on the client continually. 2 Observe the client during meals. 3 Teach the client to measure intake and output. 4 Involve the client in developing a daily meal plan.

1Bulimic clients often hide food or force vomiting; therefore they must be carefully observed. Observing the client during meals is insufficient because these clients may induce vomiting after eating. Fluid and electrolyte balance can become a problem for these clients and monitoring is required, but at this time it is the responsibility of the nurse, not the client, to measure intake and output. These clients will not become involved in planning meals; this is a long-term goal.

A man has completed an alcohol detoxification program and is setting goals for rehabilitation. When setting goals it is important for this client to understand the need to: 1 Restructure his life without alcohol. 2 Plan to avoid people who drink. 3 Accept that he is a fragile person. 4 Develop new social drinking skills.

1Clients must learn new lifestyles and coping skills to maintain sobriety. Planning to avoid people who drink is an unrealistic, unattainable plan. Accepting that he is a fragile person is judgmental, negative thinking that will lower self-esteem. Abstinence is essential; social drinking is not an option.

A client who has a diagnosis of paranoid schizophrenia and has been violent in the past is admitted to the psychiatric unit. What should the nurse do before conducting an admission interview? 1 Enter the room with another staff member while remaining between the client and the door. 2 Have two other staff members present when talking with the client. 3 Alert the assault response team about the client's history. 4 Move to the client's side and sit down.

1Making sure to stay between the client and the door provides safety for the nurse and the other staff member because it will enable them to make a rapid exit. Moving to the client and sitting down invades the client's territory and may precipitate an aggressive client response. Alerting the assault response team is premature; the team is alerted when a client is out of control, harming self or others, and cannot be managed by the staff on the unit. Having two other staff members present may be viewed by the client as confrontational and may precipitate an aggressive response.

A client receiving risperidone (Risperdal) is going on an all-day fishing outing with family members. It is important that the nurse: 1 Encourage the client to use sunscreen. 2 Caution the client to avoid excessive activity. 3 Advise the client to bring an additional dose of medication to take after lunch. 4 Have the client take a blood pressure reading before leaving for the fishing outing.

1Risperidone (Risperdal) causes photosensitivity, which can be controlled with the use of sunscreens and protective clothing. Cautioning the client to avoid excessive activity is not a necessary precaution with this atypical antipsychotic drug; the client should be allowed to participate fully. The medication should be administered as prescribed; additional doses should not be administered. Participating in an outing should not affect the client's blood pressure.

A depressed client cries when the family does not visit. What is the most therapeutic response by the nurse? 1 "Your family didn't visit, and now you're feeling rejected." 2 "It's difficult to realize that no one cares about you." 3 "It's terrible to have such negative thoughts about yourself." 4 "Your family members work—that's why they don't visit you."

1The statement "Your family didn't visit, and now you're feeling rejected" accurately reflects the client's emotions and may encourage exploration of feelings. The nurse does not know that no one cares about the client, and the statement may increase the client's unhappiness. The client is upset about the lack of visitors; discussing negative thoughts about self changes the subject. The defensive statement "Your family members work—that's why they don't visit you" may worsen the client's self-derogatory feelings.

A young woman who has just lost her first job comes to the mental health clinic very upset and says, "I just start crying without any reason and without any warning." How should the nurse respond initially? 1 "Are you having any other problems at this time?" 2 "Crying unexpectedly can be very upsetting." 3 "Do you know what makes you cry?" 4 "Most of us need to cry from time to time."

2 The response "Crying unexpectedly can be very upsetting" identifies the client's feelings. Asking, "Do you know what makes you cry?" is an unrealistic question; the cause of anxiety may not be known. "Most of us need to cry from time to time" moves the focus away from the client. "Are you having any other problems at this time?" disregards the client's comment; it is a direct question that may impede communication.

SSRI and MAOI cant be mixed before a

2 week wash out period and tapering off

The practical nurse (PN) is caring for a male client who is admitted for schizophrenia and observes that his thoughts do not flow logically and he uses invented words. How should the PN document this behavior?

B. Uses neologisms and tangential expressions.

A nurse is caring for depressed older adults. What precipitating factors for depression are most common in the older adult without cognitive problems? Select all that apply. 1 Dementia 2 Multiple losses 3 Declines in health 4 A milestone birthday 5 An injury requiring hospitalization

23Depression in the older adult is most often associated with the loss of family members and friends (e.g., death, relocation) and declines in mobility, health, and income. A decline in health, particularly when associated with a chronic illness, frequently precipitates depression in older adults. Dementia is a cognitive problem. Research does not correlate the onset of depression with a milestone birthday in older adults. A traumatic injury does not precipitate the onset of depression in the older adult as often as does a chronic illness.

Common side effects of TCA

Blurred vision Urinary retention Dry mouth Constipation Sweating Seizures Dizziness

A nurse who is assessing a recently hospitalized client with a diagnosis of opioid addiction should look for signs of withdrawal. What are these signs? Select all that apply. 1 Seizures 2 Yawning 3 Drowsiness 4 Constipation 5 Muscle aches

25Yawning and muscle aches are clinical manifestations of opioid withdrawal, which occurs after cessation or reduction of prolonged moderate or heavy use of opioids. Seizures do not occur with opioid withdrawal. Insomnia, not drowsiness, occurs with opioid withdrawal. Diarrhea, not constipation, occurs with opioid withdrawal.

The laboratory calls to report that a hospitalized client's lithium level is 1.9 mEq/L after 10 days of lithium therapy. How should the nurse respond to this information? 1 By monitoring the client closely because the level of lithium in the blood is slightly high 2 By notifying the practitioner of the findings because the level is dangerously high 3 By continuing the administration of the medication as prescribed because the level is within the therapeutic range 4 By reporting the finding to the practitioner so the dosage can be increased because the level is below the therapeutic range

2Any result above 1.5 mEq/L is approaching or in the toxic range. (The therapeutic range for lithium is 0.6 to 1.2 mEq/L.) Immediate action must be taken. The level is dangerously high, and continued administration of the drug and simple monitoring are unsafe.

An older adult tells the nurse, "I regret so many of the choices I've made during my life." Which of Erikson's developmental conflicts has the client probably failed to accomplish? 1 Autonomy versus shame and doubt 2 Ego integrity versus despair 3 Identity versus role confusion 4 Generativity versus stagnation

2The sense of ego integrity comes from satisfaction with life and acceptance of what has been and what is. Despair reflects guilt or remorse over what might have been. During puberty adolescents attempt to find themselves and integrate their own values with those of society; an inability to solve conflict results in confusion and hinders mastery of future roles. During early and middle adulthood the individual is concerned with the ability to produce and to care for that which is produced or created; failure during this stage leads to self-absorption or stagnation. Autonomy, the ability to control the body and environment, is developed during the toddler period; doubt may result when the child is made to feel ashamed or embarrassed.

A client has recently been admitted for evaluation of sudden onset psychosis. In addition to a medical assessment and serum drug screen, which other tests should the nurse expect the client will undergo to find a cause for this change in mental status?

Brain MRI, PET scan.

Which finding should the practical nurse (PN) identify in a 10-year-old client who is diagnosed with attention deficit hyperactivity disorder (ADHD)?

D. Inability to concentrate long enough to complete school work.

An older male client who has vision and hearing problems is admitted after a combative incident with his caregivers. Which intervention should the practical nurse (PN) implement when providing basic care?

D. Obtain the client's attention and consent before starting care.

A daycare environment is recommended for a client with incapacitating behaviors resulting from an obsessive-compulsive personality disorder. The client's partner asks the nurse why this approach is necessary. What is the best response by the nurse? 1 "A location that requires no decision-making will resolve feelings of anxiety." 2 "The daycare setting allows the staff to exert control over unacceptable behaviors." 3 "A neutral atmosphere facilitates the working through of conflicts." 4 "This environment limits time to carry out the rituals."

3 These clients can better work through their underlying problems when the environment is structured, demands are reduced, and the routine is simple. Preventing these clients from carrying out rituals may precipitate panic reactions. Although eliminating the necessity to make decisions may decrease anxiety, simple decision-making should be encouraged. The intention of therapy should be to help the client gain control, not to enable others to do the controlling.

At what age is a client in Freud's phallic stage of psychosexual development and Erikson's psychosocial phase of initiative versus guilt?

3 to 5 years

A client with vascular dementia (formerly known as multiinfarct dementia) has signs and symptoms that are different from dementia of the Alzheimer type. What characteristics unique to vascular dementia should the nurse expect when assessing a client with this diagnosis? Select all that apply. 1 Memory impairment 2 Failure to identify objects 3 Exaggerated deep tendon reflexes 4 Episodic progression of symptoms 5 Inability to use words to communicate

34The diagnosis of vascular dementia is made when there is evidence of focal neurological signs and symptoms such as exaggerated deep tendon reflexes, extensor plantar response, gait abnormalities, and muscle weakness and computed tomography reveals multiple infarcts involving the cortex and underlying white matter. Usually the signs and symptoms associated with vascular dementia have a steplike progression because of further intermittent occlusion of small arteries or arterioles in the cortex of the brain. Dementia of the Alzheimer type is associated with a gradual, progressive loss of memory and cognitive abilities. Both vascular dementia and dementia of the Alzheimer type are associated with deficits in memory and cognition. Failure to identify objects despite intact sensory function (agnosia) is a cognitive disturbance associated with both vascular dementia and dementia of the Alzheimer type. Both vascular dementia and dementia of the Alzheimer type are associated with language disturbances such as inability to use or understand words (aphasia).

A nurse is counseling a recently widowed client, who says, "His death has complicated my life even more than the hassles he caused when he was alive!" The nurse realizes the client is having difficulty with the grieving process and nurse concludes that the relationship with the husband was probably: 1 Loving 2 Long-term 3 Subservient 4 Ambivalent

3If the relationship was ambivalent, the surviving spouse now has feelings of both anger and guilt to resolve. A loving relationship evokes fewer feelings of guilt and is followed by a less complicated grieving process. The length of the relationship seems to have little to do with the ease or difficulty in completing the grieving process. Individuals in the subservient role usually have learned to accept directions and either find a new director or are relieved to have a chance to express their own feelings.

A client is admitted to a long-term care facility and placed in a semiprivate room. After the second night on the unit the client's roommate reports that the client is masturbating at night and demands another room. What is the most appropriate intervention by the nurse? 1 Telling the roommate that this is acceptable behavior and that the client has the right to engage in it 2 Informing the client who is masturbating that this behavior is inappropriate and should not continue 3 Providing the client who was masturbating with periods of private time 4 Moving the roommate who made the report to another room

3Masturbating is a healthy human sexual behavior. The client should be provided with private time. The client has the right to meet physical needs but should not impose the behavior on others. Moving the roommate to another room could be ineffective because this may happen with the client's future roommate. Telling the roommate that this is acceptable behavior and that the client has the right to engage in it does not address either client's needs.

A nurse interviews a young female client with anorexia nervosa to obtain information for the nursing history. What will the client's history most likely reveal? 1 Close, supportive mother-daughter relationship 2 Satisfaction with and desire to maintain her current weight 3 Low level of achievement in school and little concern for grades 4 Strong desire to improve her body image

4

A nurse notes that haloperidol (Haldol) is most effective for clients who exhibit behavior that is: 1 Depressed 2 Withdrawn 3 Manipulative 4 Overactive

4

A client with a history of substance abuse is brought to the emergency department. The client is having seizures, is hypertensive, and has hyperthermia. What drug should the nurse consider that the client may have been abusing? 1 Fentanyl 2 Alcohol 3 Oxycodone 4 Methamphetamine

4 Methamphetamine is a stimulant that increases the heart rate and blood pressure. It can cause hyperthermia, convulsions, and death. Alcohol is a central nervous system (CNS) depressant. Overdose of alcohol leads to a decreased level of consciousness and respiratory depression. Fentanyl and oxycodone are opioid and CNS depressants. Overdose leads to hypotension, a decreased level of consciousness, and respiratory depression.

The nurse teaches a client methods of coping with anger. The nurse concludes that the client has learned the most effective method when the client states that when she is angry she will: 1 Go for a long jog. 2 Go to the basement to scream. 3 Concentrate on what made her angry. 4 Talk about the anger

4 Talking about angry feelings is better than acting them out; this response indicates that the client has learned a positive coping method. Although taking a long jog or going to the basement to scream may help, it is an isolated activity that does not permit sharing of feelings and may not always be possible. Concentrating on what made the client angry may result in an escalation of angry feelings.

The nurse at the mental health clinic is counseling a client with obsessive-compulsive disorder who spends a lot of time each day engaged in handwashing and has trouble keeping appointments on time as a result. What is the most therapeutic initial intervention by the nurse?

Accepting the ritualistic behavior with a matter-of-fact attitude without displaying criticism

An older client with the diagnosis of dementia of the Alzheimer type is admitted to a long-term care facility. When planning care for this client, the nurse recalls that confusion: 1 Will be unchanged despite reality orientation 2 Is a common finding and is expected with aging 3 Results from brain changes that make interventions futile 4 Occurs with a transfer to new surroundings

4 A change in environment and introduction of unfamiliar stimuli precipitate confusion in clients with dementia-type disorders; with appropriate intervention, including frequent reorientation, confusion can be reduced. Reality orientation can reduce confusion when these clients are confronted with unfamiliar surroundings. The assertions that reality orientation is ineffective, that confusion is an expected finding in aging, and that brain changes in dementia make interventions futile are all untrue.

What is an appropriate behavior modification goal for a client with anorexia nervosa? 1 Eating every meal for a week 2 Attending group therapy every day 3 Talking about food for 1 hour a day 4 Gaining 1 lb of weight a week

4 A goal is focused on where the client should be after certain actions are taken; this client needs to gain weight. Forcing the client to eat every meal for a week may set up a struggle between the client and the nurse; the focus of care should not be on the actual intake of food. Behavior modification techniques work much better than group therapy; anorexic clients lack insight and will focus on food, not eating. Anorexic clients talk freely about food; this is not therapeutic.

An anxious client expressing a fear of people and open places is admitted to the psychiatric unit. What is the most effective way for the nurse to assist this client?

Accompany her outside for an increasing amount of time each day

A client being admitted for alcoholism reports having had alcoholic blackouts. The nurse knows that an alcoholic blackout is best described as: 1 A fugue state resembling absence seizures 2 Fainting spells followed by loss of memory 3 Loss of consciousness lasting less than 10 minutes 4 Absence of memory in relation to drinking episodes

4 Although the exact cause is unclear, alcoholic blackouts appear to result from responses of central nervous system cells to the substance. The individual does not have any type of seizure during the blackout. Fainting is not associated with the blackout. The individual loses memory but not consciousness.

Nortriptyline (Pamelor) three times a day is prescribed for a depressed client. When does the nurse expect a therapeutic response? 1 1 to 3 days 2 12 to 24 hours 3 30 minutes to 2 hours 4 2 to 3 weeks

4 As with other tricyclics, optimal therapeutic effects take 2 to 3 weeks to occur. One to 3 days, 12 to 24 hours, and 30 minutes to 2 hours are all too soon to expect a response to nortriptyline.

A nurse is discussing plans with a client who has decided to withdraw from alcohol. What should the nurse recommend as one of the most effective treatments for alcoholism?

Active membership in Alcoholics Anonymous

When implementing the plan of care for a client who is recovering from an overdose of clomipramine (Anafranil), the practical nurse (PN) recognizes that it is essential to monitor the client for which side effects?

D. Orthostatic hypotension and constipation.

An 18-year-old woman is brought to the emergency department by her two roommates after being found unconscious in the bathroom. Laboratory tests are ordered. The nurse reviewing the findings notes that the urinalysis is positive for flunitrazepam (Rohypnol). The nurse knows that flunitrazepam is often used:

As a date rape drug

While admitting a confused 80-year-old client to the mental health unit, the nurse recalls that one factor associated with the aging process is: 1 Changing of personality 2 Lowering of intelligence 3 Diminution of long-term memory 4 Slowing of responses

4 Neurological responses are slowed because of reduced sensory-receptor sensitivity. Excluding pathological processes, the personality will be consistent with that of earlier years. There is no loss of intellectual ability unless there is a pathological problem. Short-term, not long-term, memory is reduced because of a shortened attention span, delayed transmission of information to the brain, and perceptual deficits.

It is lunchtime at a mental health facility, and a 27-year-old client is unable to open her door because she has run out of the paper towels she uses to avoid touching the doorknob. What is the priority intervention by the nurse? 1 Exploring the feelings that triggered her use of the ritual for opening doors 2 Encouraging her to open the door if she expects to leave for the dining room 3 Opening the door for her and telling her that the staff will try to find more towels 4 Giving her a supply of paper towels and then sending her to the dining room for lunch

4 Providing paper towels takes into consideration the fact that the client's anxiety will increase if the ritual is interrupted; this allows the client to complete the ritual and encourages her to go to the dining room for lunch. Exploring feelings at this time is inappropriate because it will increase anxiety and result in the client's missing a meal. Having the client open the door without performing the ritual denies the ritual, which will result in an increase in anxiety. Opening the door for the client will precipitate a conflict between completing the ritual and going to the dining room; this conflict will increase anxiety.

A client is being treated for daytime somnolence and sleep attacks at work. He reports falling asleep several times a day while being in the middle of something. He also describes short episodes in which he cannot seem to move his limbs, usually after laughing really hard about something. The symptoms disappear within seconds. For which other health problem is this client's medication also prescribed?

ADHD

An older adult living in a long-term care facility has been receiving lithium 600 mg twice a day for 3 weeks to ease manic behavior. The client is experiencing nausea and vomiting, diarrhea, thirst, polyuria, slurred speech, and muscle weakness. What is the most appropriate nursing intervention? 1 Obtaining a prescription for the antidote to lithium and administering it immediately 2 Suggesting that the practitioner replace the lithium for an antiepileptic that will control the mania 3 Assessing the client for coarse hand tremor and, if it is present, giving the daily dose of lithium with a bit of water 4 Withholding the next dose of lithium and drawing blood to test it for toxicity

4 The client is displaying signs and symptoms of early lithium toxicity; older clients should be monitored carefully and given smaller doses of lithium because its excretion from the kidneys is slower than that in younger adults. There is no antidote to lithium. Coarse hand tremor is an indication of advanced lithium toxicity; the lithium should be withheld. Although antiepileptics are effective in 25% to 50% of clients with treatment-resistant bipolar disorder, this is not the appropriate treatment for lithium toxicity.

The nurse manager hears a conversation between a nurse and a client that is focused on the details of their impending divorces. What is the nurse manager's response? 1 Waiting until the conversation ends and then telling the nurse that such topics must be discussed in strict privacy to ensure client confidentiality 2 Immediately explaining to both nurse and client that such conversations are inappropriate and that the nurse's assignment will be changed 3 Waiting until shift report and using that opportunity to discuss appropriate nurse-client boundaries with the attending nursing staff 4 Immediately asking to speak to the nurse privately and stating that sharing such personal information is nontherapeutic and not tolerated

4 The nurse-client relationship should always remain client focused. Discussing personal issues with the client, even in an attempt to share similar experiences, is nontherapeutic and should be discussed immediately by the nurse's supervisor. Although the ease with which this conversation was overheard does raise concerns about the nurse's understanding of the client's right to confidentiality and privacy, there is a greater issue that needs immediate attention and should be addressed immediately. The nurse's management of the nurse-client relationship should be discussed privately. It may not be necessary to change the assignment. Although it may be useful to reinforce information on privacy with the entire staff, the situation requires an immediate private discussion between the nurse and the nurse manager to satisfactorily address the problem for the individual nurse.

A young adult male client, diagnosed with paranoid schizophrenia, believes that world is trying poison him. What intervention should the nurse include in this client's plan of care?

Ask one nurse to spend time with the client daily

When leading the first session of a newly formed group of clients in a mental health clinic, the nurse notes that group members frequently assume self-serving roles. The nurse understands that: 1 Some group members will need to be placed in another group. 2 Certain group members may be emerging to control attention seekers. 3 The group is attempting to reconcile conflicting viewpoints among its members. 4 Early group development involves these behaviors.

4 These behaviors are a necessary phase of group development because they help members discover what they can expect from the leader and other members. It is inappropriate to assume at the first meeting that some clients will need to be switched to another group. Group factions are unlikely to emerge in the first session; moreover, factions seldom emerge to control disruptive group behavior. The group has not yet developed to the phase of reconciling conflict; conflict resolution and management occur only in operating groups.

An adult male client who was admitted to the mental health unit yesterday tells the nurse that microchips were planted in his head for military surveillance of his every move. Which response is best for the nurse to provide?

Go to occupational therapy and start a project.

The nurse is caring for a client experiencing a crisis. What role is most important for the nurse to assume when providing therapeutic crisis intervention? 1 Passive listener 2 Friendly adviser 3 Participant observer 4 Active participan

4 To intervene in a crisis the nurse must assume a direct, active role because the client's ability to cope is lessened and help is needed to problem-solve. Being a passive listener is insufficient to help the client. Being a friendly adviser can blur the boundaries between a professional and a social relationship. The role of the nurse should not include giving advice. Being a participant observer is insufficient to help the client.

A client with a known history of opioid addiction is treated for multiple stab wounds to the abdomen. After surgical repair the nurse notes that the client's pain is not relieved by the prescribed morphine injections. The nurse realizes that the failure to achieve pain relief indicates that the client is probably experiencing the phenomenon of: 1 Habituation 2 Physical addiction 3 Psychological dependence 4 Tolerance

4 Tolerance is a phenomenon that occurs in addicted individuals in which increasing amounts of the drug of addiction are needed to satisfy need; the client should receive adequate analgesia after surgery. Drug habituation is a mild form of psychological dependence; the individual develops a habit of taking the substance. A physical addiction is related to biochemical changes in body tissues, especially the nervous system. The tissues come to require the substance for usual function. Psychological dependence is emotional reliance on the substance to maintain a sense of well-being.

The nurse is planning care for a 32-year-old male client diagnosed with HIV infection who has ahistory of chronic depression. Recently, the client's viral load has begun to increase rather thandecrease despite his adherence to the HIV drug regimen. What should the nurse do first while takingthe client's history upon admission to the hospital?

Ask the client if he takes St. John's Wort routinely

A 23-year-old woman is admitted to a psychiatric unit after several episodes of uncontrolled rage at her parents' home, and borderline personality disorder is diagnosed. While watching a television newscast describing an incident of violence in the home, the client says, "People like that need to be put away before they kill someone." The nurse concludes that the client is using: 1 Denial 2 Introjection 3 Sublimation 4 Projection

444Projection is the process of attributing one's thoughts about one's self to others. Denial involves pushing out of awareness one's own thoughts, wishes, or feelings that are unacceptable to one's own self. Introjection is the process of taking in someone else's values, beliefs, attitudes, or qualities. Sublimation is the channeling of unacceptable thoughts or feelings into acceptable activities.

The wife of a male client recently diagnosed with schizophrenia asks the nurse, "What exactly is schizophrenia? Is my husband all right?" Which response is best for the nurse to provide to this family member?

It is a chemical imbalance in the brain that causes disorganized thinking

A client recently admitted to the psychiatric unit after an attempted suicide is placed in a private room with the curtains, telephone, and call light removed. A student nurse caring for this client on a one-to-one basis identifies that the client has become withdrawn and tends to sleep most of the time. What is the most appropriate nursing intervention?

Asking an open-ended question and sitting quietly by the client

According to Erikson, what will happen to an individual who fails to master the maturational crisis of adolescence? 1 Interpersonal isolation 2 Rebellion against parental orders 3 Feelings of inferiority on comparing the self to others 4 Role confusio

4According to Erikson, adolescents are struggling with identity versus role confusion, struggling to find out who they are. If an adolescent is unsuccessful in this regard, role confusion may result. Industry versus inferiority is the developmental struggle of the school-aged child. This reflects part of the struggle for autonomy; it does not indicate failure to achieve the developmental task of adolescence. Adolescents tend to be group oriented, not isolated; they struggle to belong, not to escape. Developing intimacy is the developmental task for the young adult.

A nurse is counseling the spouse of a client who has a history of alcohol abuse. The nurse explains that people with a long history of alcohol abuse drink alcohol mainly because they: 1 Lack the motivation to stop 2 Have no other coping mechanism 3 Enjoy the associated socialization 4 Are dependent on it

4Alcohol causes both physical and psychological dependence; the individual needs the alcohol to function. Alcoholism is a disorder that entails physical and psychological dependence. The individual is unlikely to have other coping mechanisms; however, the individual needs to learn how to use other coping mechanisms more consistently and effectively. People with alcoholism usually drink alone or feel alone in a crowd; this is not the prime reason for their drinking.

A hyperactive client with bipolar I disorder becomes loud and insulting and says to a staff member, "Get lost, you old buzzard!" The nurse can best handle this situation by:

Asking the client to come with her for a walk

What should the nurse do when an adolescent girl with the diagnosis of anorexia nervosa starts to discuss food and eating? 1 Listen to the client's list of favorite foods and secure these foods for her. 2 Use the client's current interest in food to encourage her to increase her food intake. 3 Let the client talk about food as long as she wants and limit discussion about her eating. 4 Tell the client gently but firmly to direct her discussion of food to the nutritionist.

4All food issues should be discussed with the nutritionist, thereby removing a potential source of conflict between the nurse and client. Listening to the client's list of favorite foods and securing these foods for her will accomplish little because the client's failure to eat is not based on food likes or dislikes. Using the client's current interest in food to encourage her to increase her food intake will increase the conflict between the nurse and client. Letting the client talk about food as long as she wants and limiting discussion about her eating may be self-defeating because a discussion of food will be the major focus of all nurse-client interactions.

A practitioner prescribes alprazolam (Xanax) 0.25 mg by mouth three times a day for a client with anxiety and physical symptoms related to work pressures. For what most common side effect of this drug should the nurse monitor the client? 1 Bradycardia 2 Agranulocytosis 3 Tardive dyskinesia 4 Drowsiness

4Alprazolam (Xanax), a benzodiazepine, potentiates the actions of γ-aminobutyric acid, enhances presympathetic inhibition, and inhibits spinal polysynaptic afferent pathways. Drowsiness, dizziness, and blurred vision are common side effects. Alprazolam may cause tachycardia, not bradycardia. Agranulocytosis is usually a side effect of the antipsychotics in the phenothiazine, not the benzodiazepine, group. Tardive dyskinesia occurs after prolonged therapy with antipsychotic medications; alprazolam is an antianxiety medication, not an antipsychotic.

A person mowing a lawn is badly disfigured by the lawnmower blade. According to Erikson's theory, which age at the time of injury will be associated with the greatest risk of long-term psychological effects? 1 35 years 2 55 years 3 70 years 4 11 years

4An 11-year-old child is generally in Erikson's stage of industry versus inferiority, which involves the mastery of skills; unfortunately, the child did not master the skill of lawnmowing. Also, the child will be entering adolescence (stage of identity versus confusion) when major physical and emotional changes occur in relation to how one is perceived by the self and by others. A 35-year-old adult is generally in Erikson's stage of intimacy versus isolation and therefore is less concerned about proving industriousness; a person of this age has usually moved through the stage of identity versus confusion. A 55-year-old adult is generally in the stage of generativity versus stagnation, is therefore less concerned about being industrious, and has moved through the stage of identity versus confusion. A 70-year-old adult is generally in Erikson's stage of ego integrity versus despair, is less concerned about becoming industrious, and has moved through the stage of identity versus confusion.

In conjunction with which classification of medication are trihexyphenidyl, biperiden (Akineton), and benztropine (Cogentin) often prescribed? 1 Anxiolytics 2 Barbiturates 3 Antidepressants 4 Antipsychotics

4Antipsychotics are used to control the extrapyramidal (parkinsonian) symptoms that often develop as a side effect of antipsychotic therapy. There is no documented use of anxiolytics with antianxiety agents because they do not have extrapyramidal side effects. Barbiturates do not have extrapyramidal side effects that respond to these drugs. Antiparkinsonian drugs usually are not prescribed in conjunction with antidepressants because antidepressants do not cause parkinsonian symptoms.

A nurse manager of a mental health unit has delegated medication administration to the licensed practical nurse (LPN). While supervising the LPN's technique, the nurse manager sees the LPN beginning to dispense an incorrect dose. How should the nurse manager respond initially? 1 By telling the LPN that an error has been made 2 By informing the nursing supervisor that the LPN is unsafe 3 By pointing out the error just before the LPN begins to administer the medication 4 By questioning the dosage in the hope that the LPN will identify the error

4Because the nurse is supervising, not evaluating, the LPN, questioning the dosage rather than pointing out the error is a positive approach that will allow the LPN to grow and help foster a supportive working relationship. Telling the LPN that an error has been made is not the initial intervention; this may become necessary if the LPN does not identify the error on his own. Informing the nursing supervisor is inappropriate and premature. Waiting until just before administration of the incorrect dose puts the client at risk.

A client is admitted voluntarily to a mental health unit. The client arrives on the unit and realizes that smoking is not permitted on the unit. At 2 am the client demands to leave the hospital because of the restriction on smoking. What should the nurse's first intervention be?

Asking the client to submit a formal "72-hour letter" for release

A client with schizophrenia is started on a regimen of chlorpromazine (Thorazine). After 10 days a shuffling gait, tremors, and some rigidity are apparent. Benztropine mesylate (Cogentin) 2 mg by mouth daily is prescribed. What should the nurse remember when administering these medications together? 1 Both medications are cholinesterase inhibitors. 2 The antipsychotic effects of chlorpromazine will be decreased. 3 The synergistic effect of these medications will cause drooling. 4 Both medications have a cholinergic blocking action.

4Both medications block central acetylcholine receptors. Neither medication inhibits cholinesterase; neostigmine (Prostigmin) acts in this manner. Although benztropine mesylate (Cogentin) can cause mental confusion when given in large doses, it does not reduce the antipsychotic effect of chlorpromazine (Thorazine). Both medications cause dry mouth.

A nurse in the mental health clinic is counseling a client with the diagnosis of depression. During the counseling session the client says, "Things always seem the same. They never change." The nurse suspects that the client is feeling hopeless. For what indication of hopelessness should the nurse assess the client? 1 Focused concentration 2 Preoccupation with delusions 3 Intense interpersonal relationships 4 Outbursts of anger

4Clients who are depressed and feeling hopeless also tend to have inappropriate expressions of anger. Depressed clients frequently have a diminished ability to think or concentrate. Preoccupation with delusions is usually associated with clients who have schizophrenia rather than with clients experiencing depression and hopelessness. Clients who are depressed and feeling hopeless tend to be socially withdrawn and do not have the physical or emotional energy for intense interpersonal relationships.

A nurse is counseling a client who is experiencing substance abuse delirium. What communication strategies should be used by the nurse when working with this client? 1 Encouraging the client to practice self-control 2 Using humor when communicating with the client 3 Approaching the client from the side rather than the front 4 Offering an introduction to the client at each meeting

4Clients with delirium have short-term memory loss; therefore it is necessary to reinforce information. A client experiencing delirium is unable to participate in a discussion about self-control. Humor is inappropriate and may cause the client to feel uncomfortable. Approaching the client from the side rather than the front may initiate a startle response, causing the client to become fearful.

A client with mental health problems is given a prescription for fluphenazine (Prolixin). The nurse develops a teaching plan about the medication. What should the nurse caution the client to avoid? 1 Eating cheeses 2 Nighttime driving 3 Taking drugs containing aspirin 4 Staying in the sun

4Fluphenazine causes photosensitivity; severe sunburn may occur with exposure to the sun. There are no known side effects of fluphenazine (Prolixin) that affect the ability to drive at night. The client should avoid eating cheese if she is taking a monoamine oxidase inhibitor, not fluphenazine, which is a phenothiazine. Aspirin is not contraindicated for clients taking fluphenazine.

A 17-year-old client is found to have anorexia nervosa. The psychiatrist, in conjunction with the client and the parents, decides to institute a behavior modification program. The nurse recalls that a major component of behavior modification is: 1 Reducing necessary restrictions 2 Deconditioning fear of weight gain 3 Reducing anxiety-producing situations 4 Rewarding positive behavior

4In behavior modification, positive behavior is reinforced and negative behavior is not reinforced or punished. Reducing the number or complexity of necessary restrictions, deconditioning the fear of weight gain, and reducing the number of anxiety-producing situations may all be part of the program, but none is a major component.

While assessing an older adult in the emergency department the nurse notes that the client is upset. The nurse asks what is wrong, and the client describes the current situation and then offers information that goes further and further off the topic. What pattern of communication does this conversation reflect? 1 Perseveration 2 Thought blocking 3 Overcompensation 4 Tangential thinking

4In tangential thinking the person never answers the question or returns to the central point of the conversation. It often is seen in people with dementia. Perseveration is the repetitive expression of a single idea in response to different questions; it is found most often in clients with cognitive impairments and those experiencing catatonia. Thought blocking is a sudden stoppage of the spontaneous flow of speaking for no apparent external reason; it is seen most often in clients who are experiencing auditory hallucinations. Overcompensation, also known as reaction formation, is a defense mechanism, not a pattern of communication.

A client who retired a year ago tells the nurse in the community health center, "I don't have any hobbies or interests, and since I retired I feel useless and unneeded." According to Erikson's developmental theory, with which developmental conflict is the client faced? 1 Initiative versus guilt 2 Intimacy versus isolation 3 Identity versus role confusion 4 Integrity versus despair

4Integrity versus despair is the task of the older adult; this client has not adapted to triumphs and disappointments, so there is no acceptance of what life is and was; this results in feelings of despair and disgust. Initiative versus guilt is the task of the preschool period. Intimacy versus isolation is the task of the young adult. Identity versus role confusion is the task of the adolescent.

The nurse anticipates that the medication that will be used to prevent symptoms of withdrawal in clients with a long history of alcohol abuse is: 1 Phenobarbital (Luminal) 2 Chlorpromazine (Thorazine) 3 Methadone hydrochloride (Methadone) 4 Lorazepam (Ativan)

4Lorazepam (Ativan) is most effective in preventing the signs and symptoms associated with withdrawal from alcohol. It depresses the central nervous system by potentiating γ-aminobutyric acid, an inhibitory neurotransmitter. Phenobarbital (Luminal) is used to prevent withdrawal symptoms associated with barbiturate use. Chlorpromazine (Thorazine), an antipsychotic medication, is not used for alcohol withdrawal. Methadone hydrochloride (Methadone) is used to prevent withdrawal symptoms associated with opioid use.

59. The nurse is preparing to administer phenelzine sulfate (Nardil) to a client on the psychiatric unit. Which complaint related to administration of this drug should the nurse expect this client to make? A) My mouth feels like cotton. B) That stuff gives me indigestion. C) This pill gives me diarrhea. D) My urine looks pink.

A dry mouth (A) is an anticholinergic effect that is an expected side effect of MAO inhibitors such as phenelzine sulfate (Nardil). (B, C, and D) are not expected side effects of this medication. Correct Answer(s): A

A nurse is caring for a terminally ill client who is considering signing an "allow natural death" (AND) document rather than the traditional do-not-resuscitate (DNR) order. In light of the process of grieving, what feeling associated with end-of-life decisions is the AND advance directive attempting to alleviate? 1 Anger 2 Denial 3 Sadness 4 Guilt

4Many bereaved people blame themselves for not following the correct course of action in preventing the death. By framing the death as part of a natural process rather than the removal of an intervention, the nurse lessens the client's guilt. Anger may occur no matter what course of action is taken. Denial of death is less likely to occur when a DNR or AND is signed. Sadness may occur no matter what course of action is taken.

A nurse is caring for a client who is addicted to opioids and who has undergone major surgery. The client is receiving methadone. What is the purpose of this medication? 1 Allows symptom-free termination of opioid addiction 2 Provides postoperative pain control without causing opioid dependence 3 Counteracts the depressive effects of long-term opioid use on thoracic muscles 4 Switches the user from illicit opioid use to use of a legal drug

4Methadone may legally be dispensed; the strength of this drug is controlled and remains constant from dose to dose, unlike illicit drugs. Methadone is used in the medically supervised withdrawal period to treat physical dependence on opiates; methadone therapy substitutes a legal for an illegal drug. Methadone may be administered over the long term to replace illegal opioid use. If methadone treatment is abruptly stopped, there will be withdrawal symptoms. Methadone is a synthetic opioid and can cause dependence; it is used in the treatment of heroin addiction but may be prescribed for people who have chronic pain syndromes. It is not used for acute postoperative pain. Methadone is not known to counteract the depressive effects of long-term opioid use on thoracic muscles.

A nurse is working with a married woman who has come to the emergency department several times with injuries that appear to be related to domestic violence. While talking with the nurse manager, the nurse expresses disgust that the woman keeps returns to the situation. What is the best response by the nurse manager? 1 "She must not have the financial resources to leave her husband." 2 "There's nothing the staff can do; people are free to choose their own lives." 3 "These women should be told how stupid they are to stay in that kind of situation." 4 "Most women try to leave about six times before they are successful."

4Nurses who work with victims of partner abuse need to be supportive and patient. It takes time and several attempts for most victims to leave abusive relationships. It may or may not be true that the client doesn't have the financial resources to leave her husband; there is not enough information to support this conclusion. The staff can encourage the woman to make plans for addressing various potential events and provide information about social services and telephone help lines. Shaming women in this position will simply make them less likely to seek help.

An older adult is brought to the clinic by a family member because of increasing confusion over the past week. What can the nurse ask the client to do to assess orientation to place? 1 Explain a proverb. 2 Give the state where she was born. 3 Recall what she ate for breakfast. 4 Identify the name of the clinic's town.

4Orientation to place refers to an individual's awareness of the objective world in its relation to the self; orientation to time, place, and person is part of the assessment of cerebral functioning. Explaining a proverb requires abstract thinking, which involves a higher integrative function than does orientation to place. Having the client state where she was born helps the nurse assess remote memory, not orientation. Having the client recalling what she had for breakfast helps assess recent memory, not orientation.

A woman who is frequently physically abused tells the nurse in the emergency department that it is her fault that her husband beats her. What is the most therapeutic response by the nurse? 1 "Maybe it was your husband's fault, too." 2 "I can't agree with that—no one should be beaten." 3 "Tell me why you believe that you deserve to be beaten." 4 "You say that it was your fault—help me understand that."

4Paraphrasing and clarifying are interviewing techniques that promote communication between the nurse and client and help the client hear and explore her words and gain insight into her behavior. Asking a "why" question is generally not therapeutic because most clients cannot respond to these questions with logical explanations. "Maybe it was your husband's fault, too" is a declarative statement that is closed, will limit dialog, and is not therapeutic.

The health care provider is performing a follow-up exam on a 17-year-old-client who was raped six months ago. The client states that she completed counseling and has been feeling like she is starting to put the experience behind her. Which action by the client is a sign of recovery?

Going to her senior prom with her boyfriend.

A client arrives at the mental health clinic complaining about feelings of extreme terror when attempting to ride in an elevator and feelings of uneasiness in large crowds. He reports that these fears are interfering with his concentration at work. What does the nurse identify as the source of these symptoms? 1 Conflict with society, resulting in an obsession 2 Depression about life events, resulting in unreasonable fears 3 Repression of a terrifying incident in an elevator, resulting in an obsession 4 Generalized anxiety about conflicts, resulting in unreasonable fears

4Phobias are specific fears that often serve as a means of coping with generalized anxiety. Conflicts with society do not result in phobias. Although depression is related to phobias, finding a direct connection to life events is difficult.

A client has been receiving oxycodone (OxyContin) for moderate pain associated with multiple injuries sustained in a motor vehicle collision. The client has returned three times for refills of the prescription. What behavior, in addition to the client's slurred speech, leads the nurse to suspect opioid intoxication? 1 Mood lability 2 Hypervigilance 3 Increased respirations 4 Constricted pupils

4Pupil constriction is a physical response to opioid intoxication; the pupils will dilate with opioid overdose. Opioids cause apathy or a depressed, sad mood (dysphoria); lability of mood is associated with the use of anabolic-androgenic steroids. Opioids cause drowsiness and psychomotor retardation; alertness is associated with the use of stimulants such as caffeine and amphetamines. Opioids depress the respiratory center of the brain, causing slow, shallow respirations; increases in temperature, pulse, respirations, and blood pressure are associated with cocaine use.

What is the primary reason that the nurse encourages the family of an alcoholic to become involved in the treatment program? 1 Alcoholics try to hide their drinking from their families. 2 Family members provide insights into the dynamics behind the drinking. 3 Family members have been most successful in providing necessary support. 4 Alcoholism involves the entire family

4Research indicates that alcoholism is a family disease, with its roots in the family of origin. Although alcoholics may try to hide their drinking from their families, this is not the reason for including the family in the treatment program. Family members often have no understanding of the dynamics behind the drinking and often need assistance with coping and counseling. Family members often do not understand the dynamics behind the drinking and often are enablers; they also need assistance with coping and counseling.

What is the primary concern for a nurse caring for a client who is grossly impaired by stimulants? 1 Drowsiness 2 Fluid imbalance 3 Suicidal ideation 4 Seizure activity

4Stimulants increase the excitatory neurotransmitters (e.g., adrenaline and dopamine), lowering the seizure threshold. A person who is under the heavy influence of stimulants will be unable to rest and sleep because of stimulation of the sympathetic nervous system. Although dehydration may occur, it is not the priority concern. Suicidality is of greatest concern during stimulant withdrawal, not when a client is grossly impaired by stimulants.

Channeling energy to healthy physical activities can decrease violent behavior. A behavioral contract is used to reinforce problem solving and encourage the use of social skills. Successful experiences improve the client's self-esteem and should decrease the manipulative behavior. Clients with conduct disorders tend to generate stress for others, not the other way around. Verbalization of negative feelings to others can often escalate and result in antisocial or acting out behavior.

4Stimulating the central nervous system with cocaine most commonly causes these responses, which can progress to fear, hallucinations, paranoid delusions, and violent behavior. Nausea is not a side effect. Euphoria, rather than fatigue, and loss of appetite, rather than hunger, are side effects. Seizures, hoarseness, and electrolyte imbalance are not common side effects of cocaine use. An increase in energy, rather than lethargy, occurs. Some cocaine users believe that the drug maximizes sexual experiences, but there is no documentation of this physiological response. Hormone imbalances are not common side effects.

During a nursing team conference, a mental health worker suggests that a client with schizophrenia, paranoid type, be assigned to group therapy. What should the nurse manager explain about this type of therapy for this client? 1 Individuals with this disorder respond well to small therapeutic groups. 2 Compliance with unit rules and medication regimens increases as therapeutic group involvement increases. 3 Involvement in small therapeutic groups may decrease the regression and dependency associated with institutionalization. 4 Therapeutic group work tends to be threatening to individuals who are suspicious.

4Suspicious individuals do not do well in groups because they are unable to tolerate the give-and-take that is necessary for successful group function. Suspicious individuals do not trust others enough to do well in group therapy. The assertion that compliance with unit rules and medication regimens increases as therapeutic group involvement increases may not be true for acutely ill psychiatric clients, who are not ready to accept reality. Reducing problems related to institutionalization is not the purpose of group therapy. These problems are addressed with remotivation therapy.

A client arrives at the mental health clinic disheveled, agitated, and demanding that the nurse "do something to make these feelings stop." What clinical manifestation is evident? 1 Suicidal tendencies 2 Narcissistic ideation 3 Demanding personality 4 Feelings of panic

4The client can no longer control or tolerate these overwhelming feelings and is seeking help. The client has not indicated plans for self-harm. Narcissistic ideation is not typical of a narcissistic personality. The client's behavior does not indicate a demanding personality.

A disturbed male client, unprovoked, attacks another client. A short-term initial plan for this client should include: 1 Placing the client in restraints or secluding the client 2 Keeping the client actively participating in activities and in contact with reality 3 Getting the client to apologize for the attack to the other client and to show remorse 4 Having the client sit with a staff member in whom he trusts

4The client needs someone with whom he has a working and trusting relationship; this individual must observe, protect, anticipate, and prevent the client from acting out destructive impulses. Restraints or seclusion may eventually be necessary, but they are too restrictive for an initial intervention. Although there is a need to keep clients in touch with reality, this client may not be ready for participation. At this time the client cannot be held responsible for his aggressive behavior.

A male client with cyclothymic disorder with hypomanic symptoms is admitted to the psychiatric unit. He has progressively lost weight and does not take the time to eat his food. How can the nurse best respond to this situation? 1 By providing a tray for him in his room 2 By assuring him that he is deserving of food 3 By pointing out that he must replace the energy that he is burning up by eating 4 By ordering food that he can hold in his hand to eat while moving around

4The client with hypomanic symptoms cannot tolerate sitting still long enough to eat an adequate meal; handheld foods will help meet the client's nutritional needs and do not require the client to sit down. This client will most likely ignore the tray. Unworthy feelings are related to a depressive, not manic, episode. It is unlikely that this client will understand or care about the need to replace energy with food.

At 4 am on a Saturday a client calls the crisis hotline. On what should the nurse focus during the initial assessment? 1 Family constellation 2 Inability to control the situation 3 Previous methods used for crisis resolution 4 Perception of the crisis event

4The client's perception of the event is essential to determining what the situation means to the client and what action can be taken. Family constellation will be important later; however, the current crisis is the immediate focus. The nurse should focus on helping the client gain control of the situation within a workable frame of reference. The focus should be positive and relate to what the client can do, not what the client is unable to do. If previous methods of coping were effective, the client would not be in a crisis. The focus should be on the immediate situation; previous crisis resolution methods may be addressed later.

A client visits the mental health clinic because of an aversion to arachnids. The client reports screaming hysterically when a spider is in the vicinity and indicates that this phobia is interfering with her job performance. What defense mechanisms does the nurse conclude that the client is using? 1 Undoing and sublimation 2 Repression and identification 3 Introjection and reaction formation 4 Displacement and projection

4The defense mechanisms of displacement and projection defense are related to phobias; displacement is the release of pent-up feelings onto something or someone else that is less threatening than the original source of the feelings, and projection is the attribution of one's own unacceptable characteristics or motives to another. Undoing and sublimation are not related to phobias; undoing is an attempt to atone for unacceptable acts or wishes; sublimation is the channeling of unacceptable impulses into constructive acceptable behaviors. Repression is the unconscious process of keeping from the consciousness ideas or impulses that are unacceptable to the individual; suppression is the conscious inhibition of an idea, impulse, or affect. Neither is related to phobias. Introjection is treating something outside the self as if it is actually inside the self; reaction formation is the expression of unacceptable desires by the use of opposite behaviors in an exaggerated way. Neither is related to phobias.

A nurse is caring for a client who has been hospitalized for alcohol withdrawal. The client decides to attend an Alcoholics Anonymous meeting. What is a basic principle of this group? 1 Spouses should attend Al-Anon meetings. 2 A commitment to permanent abstinence must be made. 3 People have the power to overcome alcoholism if they truly want to stop drinking. 4 Amends must be made to each person who has been harmed.

4The eighth step of the 12 steps of Alcoholics Anonymous (AA) is "Made a list of all persons we had harmed, and became willing to make amends to them all." Attendance of Al-Anon meetings by the spouses of alcoholics is not a basic principle of AA; attendance is the decision of the individual exposed to the alcoholic. AA focuses on one day at a time. The program holds that alcoholics are powerless to overcome alcoholism and that their lives have become unmanageable; recovering alcoholics believe that a power greater than themselves will help them recover.

A depressed client often sleeps past the expected time of awakening and spends excessive time resting and sleeping. Which nursing intervention is appropriate for this client? 1 Offering the client a series of relaxation tapes 2 Rescheduling the client's bedtime to an earlier hour 3 Suggesting that the client exercise before going to bed 4 Restricting the client's access to the bedroom

4The goal is 6 to 8 hours of rest at night; too much time spent sleeping in the daytime will defeat the goal of adequate rest at night. Offering the client a series of relaxation tapes will contribute to the client's desire for relaxation and sleep. Rescheduling the client's bedtime to an earlier hour will support the client's hypersomnia; the client already sleeps too much. Suggesting that the client exercise before going to bed will increase the metabolic rate, which is not conducive to rest.

A client has been attending weekly outpatient psychotherapy sessions for several months. The nurse psychotherapist has been working with the client to help lessen obsessive-compulsive behaviors that have interfered with the client's work performance. What information about the client best validates the client's improvement? 1 She states that she spends less time on ritualistic behaviors while at work. 2 She discusses techniques she uses to provide distraction from obsessive thoughts. 3 She reports spending an increased amount of time with friends in pleasurable activities. 4 She receives a letter from a supervisor at work stating that her job performance has improved.

4The letter provides objective validation that the client's work performance has improved. Although spending less time at work on compulsive behavior, coming up with techniques to lessen the need for the behavior, and spending more time with friends in pleasurable activities are all acceptable outcomes of therapy, they all represent subjective information reported by the client.

Although upset by a young client's continual complaints about all aspects of care, the nurse ignores them and attempts to divert the conversation. Immediately after this exchange with the client, the nurse discusses with a friend the various stages of development of young adults. Which defense mechanism is the nurse is using? 1 Sublimation 2 Substitution 3 Identification 4 Intellectualization

4The nurse is using facts and knowledge to detach herself from the emotional impact of the client's problem and to ease the anxiety it is causing. Sublimation is the channeling of unacceptable thoughts or feelings into acceptable activity. Substitution is similar to displacement; anxiety is reduced with a transfer of the emotions associated with an object or person to another, safer object or person. Identification is trying to unconsciously imitate the behavior of another who is considered important in an attempt to incorporate the important aspects of this individual into the self.

While a client is attending an Alcoholics Anonymous (AA) meeting, a nurse talks with the client's spouse about the purpose of AA. What is the priority goal of this self-help group? 1 Developing functional relationships 2 Identifying how people present themselves to others 3 Understanding patterns of interacting within the group 4 Changing destructive behavior

4The purpose of a self-help group is for individuals to develop their strengths and new, constructive patterns of coping. Developing functional relationships, identifying how people present themselves to others, and understanding patterns of interaction within the group are purposes of group therapy.

In the process of development the individual strives to maintain, protect, and enhance the integrity of the self. The nurse determines that this is usually accomplished through the use of: 1 Affective reactions 2 Withdrawal patterns 3 Ritualistic behaviors 4 Defense mechanisms

4When the individual experiences a threat to self-esteem, anxiety increases, and defense mechanisms are used to protect the self. Affective reactions are mood disorders. Withdrawal patterns are deviant ways of coping with stress; if carried to an extreme, behavior may become pathological. Ritualistic behaviors are not an aspect of the developmental process.

23. The nurse is taking a history for a female client who is requesting a routine female exam. Which assessment finding requires follow-up? A) Menstruation onset at age 9. B) Contraceptive method includes condoms only. C) Menstrual cycle occurs every 35 days. D) Black-out after one drink last night on a date.

A "black-out" typically occurs after ingestion of alcohol beverages that the client has no recall of the experiences or one's behavior and is indicative of high blood alcohol levels, but the client's experience of a "black-out" after one drink (D) is suspicious of the client receiving a "date rape" drug (Flunitrazepam) and needs additional follow-up. Although (A and C) occur on the outer ranges of "average," both are within acceptable or "normal" ranges. (B) is an individual preference, but using condoms as the only contraceptive method carries a higher chance of conception. Correct Answer(s): D

A client has recently been admitted for evaluation of sudden onset psychosis. In addition to a medical assessment and serum drug screen, which other tests should the nurse expect the client will undergo to find a cause for this change in mental status? Brain MRI, PET scan. Skull radiographs, free thyroxin. CT angiography, electromyogram. Carotid Doppler, electrocardiogram.

Brain MRI, PET scan.

23. The nurse is taking a history for a female client who is requesting a routine female exam. Which assessment finding requires follow-up? A) Menstruation onset at age 9. B) Contraceptive method includes condoms only. C) Menstrual cycle occurs every 35 days. D) Black-out after one drink last night on a date.

A "black-out" typically occurs after ingestion of alcohol beverages that the client has no recall of the experiences or one's behavior and is indicative of high blood alcohol levels, but the client's experience of a "black-out" after one drink (D) is suspicious of the client receiving a "date rape" drug (Flunitrazepam) and needs additional follow-up. Although (A and C) occur on the outer ranges of "average," both are within acceptable or "normal" ranges. (B) is an individual preference, but using condoms as the only contraceptive method carries a higher chance of conception. Correct Answer(s): D

62. A young adult male client, diagnosed with paranoid schizophrenia, believes that world is trying poison him. What intervention should the nurse include in this client's plan of care? A) Remind the client that his suspicions are not true. B) Ask one nurse to spend time with the client daily. C) Encourage the client to participate in group activities. D) Assign the client to a room closest to the activity room.

A client with paranoid schizophrenia has difficulty with trust and developing a trusting relationship with one nurse (B) is likely to be therapeutic for this client. (A) is argumentative. Stress increases anxiety, and anxiety increases paranoid ideation; (C) would be too stressful and anxiety-promoting for a client who is experiencing pathological suspicions. (D) also might increase anxiety and stress. Correct Answer(s): B

62. A young adult male client, diagnosed with paranoid schizophrenia, believes that world is trying poison him. What intervention should the nurse include in this client's plan of care? A) Remind the client that his suspicions are not true. B) Ask one nurse to spend time with the client daily. C) Encourage the client to participate in group activities. D) Assign the client to a room closest to the activity room.

A client with paranoid schizophrenia has difficulty with trust and developing a trusting relationship with one nurse (B) is likely to be therapeutic for this client. (A) is argumentative. Stress increases anxiety, and anxiety increases paranoid ideation; (C) would be too stressful and anxiety-promoting for a client who is experiencing pathological suspicions. (D) also might increase anxiety and stress. Correct Answer(s): B

A nurse is caring for a client who has been experiencing delusions. According to psychodynamic theory, delusions are:

A defense against anxiety

59. The nurse is preparing to administer phenelzine sulfate (Nardil) to a client on the psychiatric unit. Which complaint related to administration of this drug should the nurse expect this client to make? A) My mouth feels like cotton. B) That stuff gives me indigestion. C) This pill gives me diarrhea. D) My urine looks pink.

A dry mouth (A) is an anticholinergic effect that is an expected side effect of MAO inhibitors such as phenelzine sulfate (Nardil). (B, C, and D) are not expected side effects of this medication. Correct Answer(s): A

63. The nurse should hold the next scheduled dose of a client's haloperidol (Haldol) based on which assessment finding(s)? A) Dizziness when standing. B) Shuffling gait and hand tremors. C) Urinary retention. D) Fever of 102° F.

A fever (D) may indicate neuroleptic malignant syndrome (NMS), a potentially fatal complication of antipsychotics. The healthcare provider should be contacted before administering the next dose of Haldol. (A, B, and C) are all adverse effects of Haldol which can be managed. Correct Answer(s): D

63. The nurse should hold the next scheduled dose of a client's haloperidol (Haldol) based on which assessment finding(s)? A) Dizziness when standing. B) Shuffling gait and hand tremors. C) Urinary retention. D) Fever of 102° F.

A fever (D) may indicate neuroleptic malignant syndrome (NMS), a potentially fatal complication of antipsychotics. The healthcare provider should be contacted before administering the next dose of Haldol. (A, B, and C) are all adverse effects of Haldol which can be managed. Correct Answer(s): D

A patient has tinnitus (ringing of the ears) what can this indicate?

A kidney malfunction

A client is admitted with a diagnosis of depression. The nurse knows that which characteristic is most indicative of depression?

A negative view of self and the future

A client with a history of alcohol abuse says to the nurse, "Drinking is a way out of my depression." Which strategy will probably be most effective for the client at this time?

A self-help group

30. A woman arrives in the Emergency Center and tells the nurse she thinks she has been raped. The client is sobbing and expresses disbelief that a rape could happen because the man is her best friend. After acknowledging the client's fear and anxiety, how should the nurse respond? A) "I would be very upset and mad if my best friend did that to me." B) "You must feel betrayed, but maybe you might have led him on?" C) "Rape is not limited to strangers and frequently occurs by someone who is known to the victim." D) "This does not sound like rape. Did you change your mind about having sex after the fact?"

A victim of date rape or acquaintance rape is less prone to recognize what is happening because the incident usually involves persons who know each other and the dynamics are different than rape by a stranger. (C) provides confrontation for the client's denial because the victim frequently knows and trusts the perpetrator. Nurses should not express personal feelings (A) when dealing with victims. Suggesting that the client led on the rapist (B) indicates that the sexual assault was somehow the victim's fault. (D) is judgmental and does not display compassion or establish trust between the nurse and the client. Correct Answer(s): C

30. A woman arrives in the Emergency Center and tells the nurse she thinks she has been raped. The client is sobbing and expresses disbelief that a rape could happen because the man is her best friend. After acknowledging the client's fear and anxiety, how should the nurse respond? A) "I would be very upset and mad if my best friend did that to me." B) "You must feel betrayed, but maybe you might have led him on?" C) "Rape is not limited to strangers and frequently occurs by someone who is known to the victim." D) "This does not sound like rape. Did you change your mind about having sex after the fact?"

A victim of date rape or acquaintance rape is less prone to recognize what is happening because the incident usually involves persons who know each other and the dynamics are different than rape by a stranger. (C) provides confrontation for the client's denial because the victim frequently knows and trusts the perpetrator. Nurses should not express personal feelings (A) when dealing with victims. Suggesting that the client led on the rapist (B) indicates that the sexual assault was somehow the victim's fault. (D) is judgmental and does not display compassion or establish trust between the nurse and the client. Correct Answer(s): C

A male client with mental illness and substance dependency tells the mental health nurse that he has started using illegal drugs again and wants to seek treatment. Since he has a dual diagnosis, which person is best for the nurse to refer this client to first

His case manager

A female client arrives in the clinic carrying a duffle bag and is wearing torn and dirty clothes. She tells the practical nurse (PN) she has no place to go. The PN takes her vital signs and observes leg ulcers on both lower extremities. What additional information should the practical nurse obtain to determine if she is homeless?

A. Ask the client directly about her living arrangements

The nurse develops a plan of care for a client with symptoms of paranoia and psychosis. The priority nursing diagnosis is impaired social interactions related to inability to trust. Which intervention is most important for the nurse to implement? A. Greet the client by first name during each social interaction. B. Determine if the client is experiencing auditory hallucinations. C. Introduce the client to peers on the unit as soon as possible. D. Assign the client to a group about developing social skills.

A. Greet the client by first name during each social interaction. Rationale: The most important nursing intervention is to greet the client by name (A) and provide short frequent contact to establish trust. The presence of auditory hallucinations can affect social interactions (B), but is not a priority intervention. (C and D) are effective interventions after individual rapport has been established with the client.

A client diagnosed with Stage 3 Alzheimer's disease is experiencing difficulty toileting appropriately. What instruction is best for the practical nurse (PN) to provide the family?

A. Label the client's bathroom door.

The practical nurse (PN) is caring for a male client with schizophrenia who is exhibiting forgetfulness, disinterest in activities, and difficulty completing tasks. Which intervention should the PN implement?

A. Provide a structured schedule of activities on the unit.

A 19-year-old calls the clinic and tells the practical nurse (PN) that since bringing her newborn infant home, she has felt apathetic, fatigued, and helpless. She states, "I don't know what's expected of me." What action is most important for the PN to take?

A. Tell the charge nurse to come to the phone and talk with the client.

A man who has been admitted numerous times for alcohol detoxification is found wandering in the street and is unable to identify himself or his home address. He is manifesting ataxia, nystagmus, and confusion and has a blood alcohol level (BAL) of 0.29%. Which prescribed medication should the practical nurse (PN) administer to prevent Korsakoff's psychosis?

A. Thiamine

Which finding should the practical nurse (PN) report immediately when talking with a new mother who is diagnosed with postpartum depression with psychotic features?

A. Thoughts of harming her infant.

The nurse who is leading a group therapy session is called to manage a unit emergency and assigns the practical nurse (PN) as the leather of the group. During the therapeutic session, a client challenges the PN as the leader. Which response should the practical nurse (PN) communicate?

A. You are saying that I should not be the leader?

A client who has been hospitalized for 2 weeks for paranoia reports continuously to the staff that someone is trying to steal his clothing. What is the correct action for the nurse to take based on the client's complaints? A. Enroll the client in an exercise class to promote positive activities. B. Place a lock on the client's closet to allay the client's concerns. C. Promote extinction of the ideation by ignoring the client. D. Explain to the client that these suspicions are certainly false.

A: Diverting the client's attention from paranoid ideation and encouraging the client to engage in positive activities can be helpful in assisting to develop a positive self-image. Option B actually supports paranoid ideation. Option C may lower self-esteem. The nurse should not argue with the client about the delusions (option D).

A client has recently been admitted for evaluation of sudden onset psychosis. In addition to a medical assessment and serum drug screen, which other tests should the nurse expect the client will undergo to find a cause for this change in mental status? Brain MRI, PET scan. Skull radiographs, free thyroxin. CT angiography, electromyogram. Carotid Doppler, electrocardiogram.

Brain MRI, PET scan.

What clinical manifestation best indicates to the nurse that the mental status of a client with the diagnosis of schizophrenia, paranoid type, is improving?

Ability to function effectively in activities of daily living

The nurse is educating a new staff member about the causes of personality disorders. The nurse is correct to identify which causes?

Abnormal brain structure and disturbances involving serotonin and GABA.

The nurse is educating a new staff member about the causes of personality disorders. The nurse is correct to identify which causes? Abnormal brain structure and disturbances involving serotonin and GABA. Abnormal brain structure and disturbances involving dopamine and GABA. Nutrition al deficiencies and damage to the brain stem. Nutritional deficiencies and damage to the limbic system.

Abnormal brain structure and disturbances involving serotonin and GABA.

Which behaviors indicate that the treatment plan for a client in alcohol rehabilitation has been effective?

Abstinent 10 days; states that sobriety is to be accomplished one day at a time; has spoken with employer about returning to work.

Which behaviors indicate that the treatment plan for a client in alcohol rehabilitation has been effective? Abstinent 10 days; states that sobriety is to be accomplished one day at a time; has spoken with employer about returning to work. Abstinent 15 days; states that the drinking problem has been overcome; plans to find a new line of work with new coworkers. Attends Alcoholics Anonymous daily; states that many of the members are real alcoholics; denies having any problems at work. Attends Alcoholics Anonymous once per week; states a willingness to stop drinking during the day; plans to drink occasionally after work.

Abstinent 10 days; states that sobriety is to be accomplished one day at a time; has spoken with employer about returning to work. Rationale The statement "one day at a time" reflects the Alcoholics Anonymous (AA) philosophy. AA promotes a 12-step program that has been successful in helping individuals who desire to stop drinking and abusing substances. Individuals learn about sobriety and responsibility through the support of other members.

Oral chlordiazepoxide (Librium) 100 mg/ hr is prescribed for a client with a Clinical Institute Withdrawal Assessment (CIWA) score of 25. The client has had 300 mg in 3 hours but is still displaying acute alcohol withdrawal symptoms. What is the next nursing action?

Administering chlordiazepoxide as indicated by the client's CIWA score

On the third day of hospitalization, a client with a history of heavy drinking begins experiencing alcohol withdrawal delirium. What is the most appropriate response by the nurse when the client begins experiencing hallucinations?

Administering the prescribed medication to the client to subdue the agitated behavior

The nurse is teaching a group parents and teachers about preventing substance abuse in children. What practices during childhood are known to minimize substance abuse later in life? Adults who teach and display self-control behaviors in difficult situations. Encouraging and praising academic achievement. A strong sense of community and neighborhood attachment. Parents who are involved and monitor their children's daily lives. Implementing strict rules with severe yet consistent consequences.

Adults who teach and display self-control behaviors in difficult situations. Encouraging and praising academic achievement. A strong sense of community and neighborhood attachment. Parents who are involved and monitor their children's daily lives.

When being admitted to a mental health facility, a young male adult tells the nurse that the voices he hears frighten him. The nurse knows that clients tend to hallucinate more vividly:

After going to bed

57. Over a period of several weeks, one male participant of a socialization group at a community day care center for the elderly monopolizes most of the group's time and interrupts others when they are talking. What is the best action for the nurse to take in this situation? A) Talk to the client outside the group about his behavior during group meetings. B) Remind the client to allow others in the group a chance to talk. C) Allow the group to handle the problem. D) Ask the client to join another group.

After several weeks, the group is in the working phase and the group members should be allowed to determine the direction of the group. The nurse should ignore the client's comments and allow the group to handle the situation (C). A good leader should not have separate meetings with group members (A), as such behavior is manipulative on the part of the leader. (B) is dictatorial and is not in keeping with good leadership skills. (D) is avoiding the problem. Remember, identify what phase the group is in--initial, working, or termination--this will help determine communication style. Correct Answer(s): C

57. Over a period of several weeks, one male participant of a socialization group at a community day care center for the elderly monopolizes most of the group's time and interrupts others when they are talking. What is the best action for the nurse to take in this situation? A) Talk to the client outside the group about his behavior during group meetings. B) Remind the client to allow others in the group a chance to talk. C) Allow the group to handle the problem. D) Ask the client to join another group.

After several weeks, the group is in the working phase and the group members should be allowed to determine the direction of the group. The nurse should ignore the client's comments and allow the group to handle the situation (C). A good leader should not have separate meetings with group members (A), as such behavior is manipulative on the part of the leader. (B) is dictatorial and is not in keeping with good leadership skills. (D) is avoiding the problem. Remember, identify what phase the group is in--initial, working, or termination--this will help determine communication style. Correct Answer(s): C

A client is prescribed sertraline (Zoloft), an antidepressant. What should the nurse include when preparing a teaching plan about the side effects of this drug?

Agitation

A client who is addicted to opioids undergoes emergency surgery. During the postoperative period the health care provider decreases the previously prescribed methadone dosage. For what clinical manifestations should the nurse monitor the client?

Agitation and attempts to escape from the hospital

A 45-year-old female client is admitted to the psychiatric unit for evaluation. Her husband states that she has been reluctant to leave home for the last six months. The client has not gone to work for a month and has been terminated from her job. She has not left the house since that time. This client is displaying symptoms of what condition

Agoraphobia

75. A 45-year-old female client is admitted to the psychiatric unit for evaluation. Her husband states that she has been reluctant to leave home for the last six months. The client has not gone to work for a month and has been terminated from her job. She has not left the house since that time. This client is displaying symptoms of what condition? A) Claustrophobia. B) Acrophobia. C) Agoraphobia. D) Post-traumatic stress disorder.

Agoraphobia (C) is the fear of crowds or being in an open place. (A) is the fear of being in closed places. (B) is the fear of high places. Remember, a phobia is an unrealistic fear which is associated with severe anxiety. (D) consists of the development of anxiety symptoms following a life event that is particularly serious and stressful (war, witnessing a child killed, etc.) and is experienced with terror, fear, and helplessness--a phobia is different. Correct Answer(s): C

75. A 45-year-old female client is admitted to the psychiatric unit for evaluation. Her husband states that she has been reluctant to leave home for the last six months. The client has not gone to work for a month and has been terminated from her job. She has not left the house since that time. This client is displaying symptoms of what condition? A) Claustrophobia. B) Acrophobia. C) Agoraphobia. D) Post-traumatic stress disorder.

Agoraphobia (C) is the fear of crowds or being in an open place. (A) is the fear of being in closed places. (B) is the fear of high places. Remember, a phobia is an unrealistic fear which is associated with severe anxiety. (D) consists of the development of anxiety symptoms following a life event that is particularly serious and stressful (war, witnessing a child killed, etc.) and is experienced with terror, fear, and helplessness--a phobia is different. Correct Answer(s): C

On the psychiatric unit a client has been receiving high doses of haloperidol (Haldol) for 2 weeks. The client says, "I just can't sit still, and I feel jittery." Which side effect does the nurse suspect that the client is experiencing?

Akathisia

18. Based on non-compliance with the medication regimen, an adult client with a medical diagnosis of substance abuse and schizophrenia was recently switched from oral fluphenazine HCl (Prolixin) to IM fluphenazine decanoate (Prolixin Decanoate). What is most important to teach the client and family about this change in medication regimen? A) Signs and symptoms of extrapyramidal effects (EPS). B) Information about substance abuse and schizophrenia. C) The effects of alcohol and drug interaction. D) The availability of support groups for those with dual diagnoses.

Alcohol enhances the EPS side effects of Prolixin. The half-life of Prolixin PO is 8 hours, whereas the half-life of the Prolixin Decanoate IM is 2 to 4 weeks. That means the side effects of drinking alcohol are far more severe when the client drinks alcohol after taking the long-acting Prolixin Decanoate IM. (A, B, and D) provide valuable information and should be included in the client/family teaching, but they do not have the priority of (C). Correct Answer(s): C

18. Based on non-compliance with the medication regimen, an adult client with a medical diagnosis of substance abuse and schizophrenia was recently switched from oral fluphenazine HCl (Prolixin) to IM fluphenazine decanoate (Prolixin Decanoate). What is most important to teach the client and family about this change in medication regimen? A) Signs and symptoms of extrapyramidal effects (EPS). B) Information about substance abuse and schizophrenia. C) The effects of alcohol and drug interaction. D) The availability of support groups for those with dual diagnoses.

Alcohol enhances the EPS side effects of Prolixin. The half-life of Prolixin PO is 8 hours, whereas the half-life of the Prolixin Decanoate IM is 2 to 4 weeks. That means the side effects of drinking alcohol are far more severe when the client drinks alcohol after taking the long-acting Prolixin Decanoate IM. (A, B, and D) provide valuable information and should be included in the client/family teaching, but they do not have the priority of (C). Correct Answer(s): C

A depressed client says, "I'm no good. I'm better off dead." What is the priority nursing intervention?

Alerting the staff to schedule 24-hour observation of the client

Within several days of hospitalization, a client is repeatedly washing the top of the same table. Whichinitial intervention is best for the nurse to implement to help the client cope with anxiety related tothis behavior?

Allow time for the ritualistic behavior, then redirect the client to other activities

A client seeks assistance at a crisis center. The client describes being extremely anxious and unable to sleep since helping with clean-up duties at a school where a student fatally shot a teacher and some classmates. Which intervention is the priority when assisting this client?

Allow ventilation of feelings.

A client seeks assistance at a crisis center. The client describes being extremely anxious and unable to sleep since helping with clean-up duties at a school where a student fatally shot a teacher and some classmates. Which intervention is the priority when assisting this client? Allow ventilation of feelings. Refer the client to a member of the clergy. Advise the client to avoid going near the school for at least six weeks. Send the client to an emergency department for further evaluation.

Allow ventilation of feelings.

A client and the client's spouse are presented with electroconvulsive therapy (ECT) as a treatment option instead of pharmacotherapy after the client experiences adverse effects of medication therapy. The nurse meets with them to discuss the procedure. What should the nurse's first action be?

Allowing the client and family members to voice feelings, myths, and fantasies about ECT

SR or XL meds should never be

Altered - never crush, chew, or cut these kind of meds

TCA drug names

Amitriptyline - remember "Amy trips on things" drug can cause sedation and orthostatic hypotension so you must move slowly. Imipramine - remember inhibits my peeing - drug can cause urinary retention

A delirious client sees a design on the wallpaper and perceives it as an animal. How should a nurse communicate what the client perceived in the change-of-shift report?

An illusion

A young adolescent is found to have anorexia nervosa. The nurse understands that the anorexia nervosa was probably precipitated by:

An inaccurate perception of hunger stimuli and a struggle between dependence and independence

The biggest problem for an older female client, immediately after the sudden death of her husband, will probably be her inability to cope with:

Anger

28. A 19-year-old female client with a diagnosis of anorexia nervosa wants to help serve dinner trays to other clients on a psychiatric unit. What action should the nurse take? A) Encourage the client's participation in unit activities by asking her to pass trays for the rest of the week. B) Provide an additional challenge by asking the client to also help feed the older clients. C) Suggest another way for this client to participate in unit activities. D) Tell the client that hospital policy does not permit her to pass trays.

Anorexics gain pleasure from providing others with food and watching them eat. Such behaviors reinforce their perception of self-control. These clients should not be allowed to plan or prepare food for unit activities and their desires to do so should be redirected (C). (A and B) are contraindicated for a client with anorexia nervosa. (D) avoids addressing the problem and is manipulative in that the nurse is blaming hospital policy for treatment protocol. Correct Answer(s): C

The sibling of a young client with borderline personality disorder asks the nurse why the client has frequent mood changes. Which is the best response by the nurse to explain the neurobiological basis of this behavior?

Brief shifts in mood are caused by an imbalance of nervous system chemicals that help regulate emotions.

28. A 19-year-old female client with a diagnosis of anorexia nervosa wants to help serve dinner trays to other clients on a psychiatric unit. What action should the nurse take? A) Encourage the client's participation in unit activities by asking her to pass trays for the rest of the week. B) Provide an additional challenge by asking the client to also help feed the older clients. C) Suggest another way for this client to participate in unit activities. D) Tell the client that hospital policy does not permit her to pass trays.

Anorexics gain pleasure from providing others with food and watching them eat. Such behaviors reinforce their perception of self-control. These clients should not be allowed to plan or prepare food for unit activities and their desires to do so should be redirected (C). (A and B) are contraindicated for a client with anorexia nervosa. (D) avoids addressing the problem and is manipulative in that the nurse is blaming hospital policy for treatment protocol. Correct Answer(s): C

A client is admitted due to alcohol intoxication and injuries sustained in a fall. The client appears anxious, agitated, and diaphoretic. Vital signs include a pulse of 140 and a blood pressure of 170/98. Delirium is suspected due to the client's claim that bugs are crawling on the bed. Which medication should the nurse expect will be administered to the client?

Chlordiazepoxide (Librium).

A male client with mental illness and substance dependency tells the mental health nurse that he has started using illegal drugs again and wants to seek treatment. Since he has a dual diagnosis, which person is best for the nurse to refer this client to first?

His case manager.

A nurse is caring for a hyperactive, manic client who exhibits flight of ideas and is not eating. What may be the reason why the client is not eating?

Is too busy to take the time to eat

A nurse working on a substance abuse unit knows that opioids most commonly are used because the individual:

Is trying to reduce stress

Survivors of a major earthquake are being interviewed on admission to the hospital. The nurse notes that they exhibit a flattened affect, make minimal eye contact, and speak in a monotone. These behaviors are indicative of the defense mechanism known as:

Isolation

During the admission interview to an inpatient psychiatric unit, the practical nurse (PN) asks a male client who is admitted with depression about recent life events that precipitated his admission. The client remains silent, looks at the floor, and does not answer any of the PN's questions. Which intervention is best for the PN to implement?

D. Record these findings in the medical record under the DSM IV Axis IV.

A client who compulsively performs handwashing rituals throughout the day is being treated for obsessive compulsive disorder (OCD). Which behavior change indicates that treatment has been effective?

Arrives at the dining hall on time for every meal for the last 2 days.

A client who compulsively performs handwashing rituals throughout the day is being treated for obsessive compulsive disorder (OCD). Which behavior change indicates that treatment has been effective? Arrives at the dining hall on time for every meal for the last 2 days. Exhibits good hygiene habits for the last week. Reads educational material regarding his illness. Begins to wash the floors after returning from outside excursions.

Arrives at the dining hall on time for every meal for the last 2 days.

The ANA Guidelines on Psychopharmacology emphasizes that psychiatric-mental health nurses understand how psychotropic drugs affect neurotransmitter systems in the brain. Therefore, the nurse should know that a decrease in γ-aminobutyric acid (GABA), according to the basic neurotransmitter theory, causes:

Anxiety

Benzodiazepines are given for what?

Anxiety Seizures Sedation and alcohol withdrawal

A female client with obsessive-compulsive disorder has become immobilized by her elaborate handwashing and walking rituals. The nurse recalls that the basis of obsessive-compulsive disorder is often feelings of:

Anxiety and guilt

11. At the first meeting of a group of older adults at a daycare center for the elderly, the nurse asks one of the members what kinds of things she would like to do with the group. The older woman shrugs her shoulders and says, "You tell me, you're the leader." What is the best response for the nurse to make? A) Yes, I am the leader today. Would you like to be the leader tomorrow? B) Yes, I will be leading this group. What would you like to accomplish during this time? C) Yes, I have been assigned to be the leader of this group. I will be here for the next six weeks. D) Yes, I am the leader. You seem angry about not being the leader yourself.

Anxiety over participation in a group and testing of the leader characteristically occur in the initial phase of group dynamics. (B) provides information and focuses the group back to defining its function. (A) is manipulative bargaining. Although (C) provides information, it does not focus the group on its purpose or task. (D) is interpreting the client's feelings and is almost challenging. Correct Answer(s): B

11. At the first meeting of a group of older adults at a daycare center for the elderly, the nurse asks one of the members what kinds of things she would like to do with the group. The older woman shrugs her shoulders and says, "You tell me, you're the leader." What is the best response for the nurse to make? A) Yes, I am the leader today. Would you like to be the leader tomorrow? B) Yes, I will be leading this group. What would you like to accomplish during this time? C) Yes, I have been assigned to be the leader of this group. I will be here for the next six weeks. D) Yes, I am the leader. You seem angry about not being the leader yourself.

Anxiety over participation in a group and testing of the leader characteristically occur in the initial phase of group dynamics. (B) provides information and focuses the group back to defining its function. (A) is manipulative bargaining. Although (C) provides information, it does not focus the group on its purpose or task. (D) is interpreting the client's feelings and is almost challenging. Correct Answer(s): B

During the orientation tour for three new staff members, a young, hyperactive manic client greets them by saying, "Welcome to the funny farm. I'm Jo-Jo, the head yo-yo." Which meaning can the nurse assign to the client's statement?

Anxious over the arrival of new staff members

A client is found to have a conversion disorder. What is the typical reaction by the client to the physical symptom?

Apathy

Windows in the recreation room of the adolescent unit have been found broken on numerous occasions. After a group discussion one of the adolescents provides sound evidence that another adolescent has broken them. What nursing action involves an assertive intervention?

Approaching the culprit when alone and, after making eye contact, inquiring about his involvement in these incidents

A nurse is caring for several clients who are going through withdrawal from alcohol. The primary reason for the ingestion of alcohol by clients with a history of alcohol abuse is that they:

Are dependent on it

A nurse recalls that in a conversion disorder, pseudoneurological symptoms such as paralysis or blindness:

Are generally necessary for the client to cope with a stressful situation.

A 65-year-old female client complains to the nurse that recently she has been hearing voices. What question should the nurse ask this client first?

Are you ever alone when you hear the voices

A male client is admitted to the psychiatric unit with a medical diagnosis of paranoid schizophrenia.During the admission procedure, the client looks up and states, "No, it's not MY fault. You can't blame me. I didn't kill him, you did." What action is best for the nurse to take?

Assess the content of the hallucinations by asking the client what he is hearing

Suicide precautions are ordered for a newly admitted client. What is the most therapeutic way to provide these precautions?

Assigning a staff member to be with the client at all times.

A client with a history of a short temper and physically abusive behavior becomes violent and is admitted to the psychiatric service. At the time of admission, the client is extremely anxious. What is the priority nursing action?

Assigning a staff member to supervise the client

When talking with a client in crisis, the crisis intervention nurse should first:

Assist the client in deciding what will be done and how it will be done.

A nurse in a long-term care facility is caring for a bedridden client with multiple chronic illnesses. Although usually continent, the client expresses anger through urinary incontinence. What should the nurse do to best address this situation?

Assist the client in setting realistic short-term goals.

The nurse determines that to help a couple work through their feelings about the husband's terminal illness, it is important to:

Assist the couple to express their feelings about his terminal illness to each other.

A depressed client has feelings of failure and a low self-esteem. In what activity should the client initially be encouraged to become involved?

Assisting a staff member in working on the monthly bulletin board

A female client with a diagnosis of alcohol abuse appears disheveled and disorganized. How can the nurse best gain the client's involvement in personal hygienic care?

Assisting her in bathing and dressing by giving her clear, simple directions

When a nurse is working with a client with psychiatric problems, a primary goal is the establishment of a therapeutic nurse-client relationship. What is the major purpose of this relationship?

Assisting the client in acquiring more effective behavior

What is the most therapeutic nursing intervention to help a late-middle-aged individual cope with the emotional aspects of aging?

Assisting the individual with plans for the future

During a follow-up visit to the mental health clinic, the mother of a 7-year-old client reports that her son has been having side effects from the medication that was ordered to treat ADHD. The mother states that the client has a decreased appetite, complains of abdominal pain, and has been unable to sleep. The health care provider recommends that the medication be changed to a nonstimulant ADHD drug. Which medication should be recommended for this client?

Atomoxetine (Strattera).

During a follow-up visit to the mental health clinic, the mother of a 7-year-old client reports that her son has been having side effects from the medication that was ordered to treat ADHD. The mother states that the client has a decreased appetite, complains of abdominal pain, and has been unable to sleep. The health care provider recommends that the medication be changed to a nonstimulant ADHD drug. Which medication should be recommended for this client? Atomoxetine (Strattera). Methylphenidate (Ritalin). Fluphenazine (Prolixin). Fluoxetine (Prozac).

Atomoxetine (Strattera).

The nurse is teaching a client with agoraphobia the cognitive behavioral technique of reframing. Which statement by the client demonstrates an understanding of how to use the reframing technique?

Attending classes at my local technical school will help me get my dream job

A nurse is interviewing a client newly admitted to an outpatient program after withdrawal from alcohol. What behavior best indicates that the client has accepted that drinking is a problem?

Attends Alcoholics Anonymous meetings daily

As the nurse is discussing psychiatric care with an older adult client, the client says, "When I was growing up I was taught to accept my lot in life and not complain. I'm proud of the fact that despite my issues I can still function independently. I don't want to be just put away." The nurse understands that the factors that influence the client's mental health are examples of:

Attitudes and beliefs

The nurse is planning a community education program about childhood mental health problems that appear to be genetically transmitted. While conducting the program, which problem should the nurse emphasize?

Autistic disorders.

The nurse is providing discharge education for a client prescribed a tricyclic antidepressant. Which statement by the client indicates the need for additional teaching?

It is best for me to take the full dose of the medication when I get up each morning."

The nurse is planning a community education program about childhood mental health problems that appear to be genetically transmitted. While conducting the program, which problem should the nurse emphasize? Autistic disorders. Sleepwalking. Anxiety states. Conduct disorder.

Autistic disorders. Rationale Researchers have discovered several sporadic genetic mutations in children with autism spectrum disorder (ASD). A program that focuses on childhood mental health disorders that have a possible genetic link should include information about ASD.

What developmental task should the nurse consider when caring for toddlers?

Autonomy

A single mother of two children who recently lost her job because her company is downsizing comes to the emergency department. The woman does not know what to do and is in crisis. The most critical factor for the nurse to determine during crisis intervention is the client's:

Available situational supports

MAOI diet

Avoid Tyramine rich foods: -aged cheese -cured meats (pepperoni, salami) -fermented cabbage(sauerkraut) - includes fermented pickles -soy sauce, fish sauce -yeast extract spreads (Marmite) -improperly stored/leftover/ spoiled food -broad bean pods such as fave beans

A nurse is caring for a client with a diagnosis of conversion disorder manifesting as paralysis of the legs. Which is the most therapeutic nursing intervention?

Avoiding focusing on the client's physical symptoms

What approach is best for the practical nurse (PN) to use when establishing a relationship with a severely socially withdrawn male client diagnosed with schizophrenia?

B. Sit with the client in silence several times a day.

A middle-aged adult was discharged from a treatment center 6 weeks ago following treatment for suicide ideation and alcohol abuse. In a follow-up visit to the mental health clinic, the client complains of lethargy, apathy, irritability, and anxiety. Which question is most important for the nurse to ask? A. "Are you taking prescribed antidepressants?" B. "How much alcohol do you consume daily?" C. "What seems to precipitate the anxious feelings?" D. "How many hours do you sleep per day?"

B. "How much alcohol do you consume daily?" Rationale: First, and most importantly, the client's use of alcohol should be determined (B) because further treatment is dependent on the client's sobriety, and asking how much alcohol is being consumed is a better question than asking if the client is drinking, which is a "yes-no" answer that does not promote dialogue. (A, C, and D) provide worthwhile assessment data, but first the nurse should determine if the client is still drinking because all efforts to treat symptoms associated with depression are diminished if the client is still consuming alcohol.

A male client with depression is unresponsive and preoccupied with guilt and hopelessness. Which statement should the practical nurse (PN) use that provides therapeutic feedback to the client?

B. "The group appreciated your comments today."

A 33-year-old client is admitted to a psychiatric facility with a medical diagnosis of major depression. When the nurse is assigning the client to a room, which roommate is best for this client? A. A 35-year-old client who recently attempted suicide B. A manic client who has started lithium carbonate treatment C. A client who is bipolar and is pacing the floor while telling jokes to everyone D. A paranoid client who believes that the staff is trying to poison the food

B. A manic client who has started lithium carbonate treatment Rationale: (B) appears to be the most stable client described since treatment was begun with lithium carbonate (treatment of choice for manic depression). Being around another depressed individual might enhance this client's own depression and possibly support suicidal ideation (A). Clients in the manic stage of bipolar disease (C) enhance the level of anxiety of those around them, which would not be therapeutic for the client at this time. Paranoid ideation (D), which is characterized by suspiciousness, would also increase anxiety in this client.

A male client arrives at the mental health clinic complaining of insomnia, irritability, increased tension, and headaches. He tells the practical nurse that the symptoms began a week ago after he lost his job, and he is concerned that he may have to relocate his family. Which stressor is this client experiencing?

B. A situational crisis.

The practical nurse (PN) is caring for a client with bulimia who continues to deny purging. Which finding should the PN report to the RN?

B. Dental erosion.

A female client tells the practical nurse (PN) that she wants to lead a healthier, more balanced life style. She asks the PN how she should begin the process of self-exploration. Which message should the PN convey?

B. Each adult is responsible for one's own behaviors, including unhealthy behaviors.

A male client is admitted to the hospital with distorted sensory perceptions, disordered thoughts, and an increase in non-goal directed motor activity. The client does not respond to the practical nurse's (PN) calming efforts.

B. Ensure the environment is safe.

The practical nurse (PN) is taking the blood pressure of a middle-aged male who is involved with his children's sports teams as a coach and referee. While establishing a nurse-client relationship, the client tells the PN that he hires and trains teenagers to work part-time in his restaurant. Which psychosocial development stage is the client experiencing?

B. Generativity.

The practical nurse (PN) is assessing a newly admitted client with paranoid schizophrenia who is hypervigilant and who constantly scans the environment. The client tells the PN, "I saw those two doctors in the hall talking about me." What descriptive terminology should the PN document to describe the client's thought process?

B. Ideas of reference

The practical nurse (PN) is inquiring about coping strategies with a male client who is admitted for alcohol abuse. The client tells the PN that his job skills and communication skills are his best assets and support. Which additional information should the PN obtain about maladaptive mechanisms?

B. Self indulgence

The nurse cares for an adolescent with a history of violence who now exhibits signs of sublimation. Which behavior by the adolescent best represents sublimation? A. Recently started wetting the bed. B. Joined a competitive boxing team. C. Kicks the dog after being scolded by his dad. D. Starts a student organization to ban violence.

B: Rationale: Sublimation is a coping mechanism characterized by substituting an unacceptable feeling or action with a more socially acceptable one. Option A is an example of regression, Option C is characteristic of displacement, and Option D is consistent with undoing.

An adult client who lives in a residential facility is mentally retarded and has a history of bipolar disorder. During the past week, the client has refused to wear clothes and frequently exposes his/her body to other residents. Which intervention should the nurse implement? A. Establish a one-to-one relationship to discuss the behavior. B. Redirect the client to physically demanding activities. C. Encourage the client to verbalize thoughts when acting out. D. Restrict social interactions with other residents in the facility.

B: The client is exhibiting manic behavior related to bipolar disorder, and the nurse should redirect the client to activities that are physically demanding so that energy can be expended in a socially acceptable manner. Psychotic clients are not capable of option A. When exhibiting acting-out behavior, the client is distracted and option C is difficult. Option D is likely to increase manic behaviors, such as mood swings and acting-out behaviors.

What characteristic of the environment is most therapeutic for clients with the diagnosis of bulimia nervosa?

Based on realistic limits

60. A 22-year-old male client is admitted to the emergency center following a suicide attempt. His records reveal that this is his third suicide attempt in the past two years. He is conscious, but does not respond to verbal commands for treatment. Which assessment finding should prompt the nurse to prepare the client for gastric lavage? A) He ingested the drug 3 hours prior to admission to the emergency center. B) The family reports that he took an entire bottle of acetaminophen (Tylenol). C) He is unresponsive to instructions and is unable to cooperate with emetic therapy. D) Those with repeated suicide attempts desire punishment to relieve their guilt.

Because the client is unable to follow instructions, emetic therapy would be very difficult to implement and gastric lavage would be necessary (C). (A and B) should be considered in determining the course of treatment, but they are not the basis for determining if gastric lavage will be implemented. Medical treatments should never be used as "punitive" measures (D). Correct Answer(s): C

60. A 22-year-old male client is admitted to the emergency center following a suicide attempt. His records reveal that this is his third suicide attempt in the past two years. He is conscious, but does not respond to verbal commands for treatment. Which assessment finding should prompt the nurse to prepare the client for gastric lavage? A) He ingested the drug 3 hours prior to admission to the emergency center. B) The family reports that he took an entire bottle of acetaminophen (Tylenol). C) He is unresponsive to instructions and is unable to cooperate with emetic therapy. D) Those with repeated suicide attempts desire punishment to relieve their guilt.

Because the client is unable to follow instructions, emetic therapy would be very difficult to implement and gastric lavage would be necessary (C). (A and B) should be considered in determining the course of treatment, but they are not the basis for determining if gastric lavage will be implemented. Medical treatments should never be used as "punitive" measures (D). Correct Answer(s): C

A client was admitted to the inpatient unit 48 hours ago with a diagnosis of a Cluster "C" personality disorder. The client is well rested but refuses to attend meals or group sessions. The client has been started on a SSRI for anxiety but has not demonstrated an effective response. Which other category of anxiolytic medication may be prescribed to produce a more immediate decrease in symptoms

Benzodiazepines.

A client was admitted to the inpatient unit 48 hours ago with a diagnosis of a Cluster "C" personality disorder. The client is well rested, but refuses to attend meals or group sessions. The client has been started on a SSRI for anxiety but has not demonstrated an effective response. Which other category of anxiolytic medication may be prescribed to produce a more immediate decrease in symptoms? MAOIs. Benzodiazepines. Lithium salts. Psychotropic medications.

Benzodiazepines.

A client receiving fluphenazine decanoate (Prolixin Decanoate) develops dystonia early during therapy. What medication does the nurse expect to be prescribed to reverse this side effect?

Benztropine (Cogentin)

Thirty minutes after administering fluphenazine (Prolixin) to a client, the nurse notes that the client's jaw is rigid, the client is drooling, and her speech is slurred. There are a number of as-needed prescriptions in the client's chart. What should the nurse administer?

Benztropine (Cogentin), 2 mg intramuscularly

A client has recently started taking a new neuroleptic drug, and the nurse notes extrapyramidal effects. Which drug does the nurse anticipate will be prescribed to limit these side effects?

Benztropine mesylate (Cogentin)

The nurse is taking a history for a female client who is requesting a routine female exam. Which assessment finding requires follow-up?

Black-out after one drink last night on a date

Donepezil (Aricept) is prescribed for a senior client who has mild dementia of the Alzheimer type. What information does the nurse include when discussing this medication with the client and family?

Blood tests that reflect liver function will be performed routinely.

The sibling of a young client with borderline personality disorder asks the nurse why the client has frequent mood changes. Which is the best response by the nurse to explain the neurobiological basis of this behavior? Brief shifts in mood are caused by an imbalance of nervous system chemicals that help regulate emotions. Shifts in mood are the result of an intolerance to certain chemicals found in food substances. Mood changes are due to the client's emotional immaturity and lack of insight into this behavior. Mood changes are common in clients during this phase of life due to hormonal changes.

Brief shifts in mood are caused by an imbalance of nervous system chemicals that help regulate emotions.

A client describes his delusions in minute detail to the nurse. How should the nurse respond?

By changing the topic to reality-based events

How should a nurse at an assisted living facility encourage a client to effectively complete the tasks of older adulthood?

By fostering a sense of contentment when the client looks back on her achievements

A resident in a nursing home recently immigrated to the United States from Italy. How does the nurse plan to provide emotional support?

By offering choices consistent with the client's heritage

A client newly admitted to the psychiatric unit because of an acute psychotic episode is actively hallucinating. The admitting nurse has documented the content of the auditory hallucinations, which center on the theme of powerlessness. Later the primary nurse approaches the client, who appears to be listening to voices, and comments, "You seem to be listening to something. Tell me what you hear." The primary nurse requests feedback from the psychiatric clinical specialist regarding this nursing intervention. How should the clinical specialist respond?

By reminding the nurse that once the content is known, there is no need to focus on the hallucinations because doing so reinforces them

A client is admitted to a mental health facility because of maladaptive coping behavior. How can the nurse best help the client develop healthier coping mechanisms?

By setting realistic limits on the client's maladaptive behavior

A nurse identifies the establishment of trust as a major nursing goal for a depressed client. How can this goal best be accomplished?

By visiting frequently for short periods with the client each day

A client who was admitted two days earlier to a drug rehabilitation unit tells the nurse, "I'm going to do what you people tell me to do so I can get out of here and get a job." What is the most accurate interpretation of this client's statement? . The treatment program is effective and the client is highly motivated. B. Defense mechanisms are being used to decrease anxiety. C. Manipulation is being used to achieve the client's personal goals. D. The client has insight into his behaviors, so privileges should be given.

C : Drug abusers and patients with anti-social behaviors tend to be manipulative, so option C is the best interpretation of the client's statement at this time in the client's treatment. He has been in treatment only 2 days, which is not enough time to benefit from the program, so options A and D are highly unlikely. Although defense mechanisms are frequently used to decrease anxiety, this statement is more likely because of option C.

The community health nurse is teaching a class of high school students about bullying. One of the students asks about social factors that might contribute to violent behavior. Which response by the nurse is correct? Children consuming too much sugar have a tendency to become violent. Children can learn aggressive behaviors by observing violence in the home. Individuals who are raised in poor families usually become violent. Children of overprotective parents usually develop aggressive behaviors.

Children can learn aggressive behaviors by observing violence in the home.

A client with schizophrenia approaches the practical nurse (PN) and says, "The voices are bothering me. They're yelling and telling me I'm bad. Can't you hear them?" Which response should the PN provide?

C. "I can't hear the voices, but I can see that you're upset."

When the mother of a young child is diagnosed with HIV, she asks the practical nurse (PN), "who will take care of my children if I die soon?" What response is best for the PN to provide?

C. "This is an important consideration, but you may live until they are grown up or even longer."

A male client is admitted with major depression and tells the practical nurse (PN) that he feels like a freak since he is being admitted to a psychiatric unit in the hospital. He feels like he is the only one with this problem. Which information should the PN provide the client?

C. About 50% of the population between the age of 15 and 55 have had a psychiatric disorder.

A male client with dementia who lives in an extended care facility is placed in a wheelchair each day and positioned in the hall where he kicks people who walk past him Which intervention should the practical nurse (PN) implement?

C. Call him by name until he focuses his attention.

The practical nurse (PN) assesses a client with a poor self-concept. This client is most likely to demonstrate which behaviors?

C. Escalation of anxiety.

A client with delusions of persecution has been refusing all the hospital meals for the last 3 days and tells the practical nurse that the food contains poison. What action should the PN implement?

C. Provide foods in the original closed containers.

In which age groups is depression often often underdiagnosed? Children. Adolescents. Young adults. Middle aged adults. Older adults.

Children. Adolescents. Older adults.

An 8-year-old child is seen in the clinic with a green vaginal discharge. Which action is most important for the nurse to implement? A. Assess the child's blood pressure. B. Counsel the child to wear cotton underwear. C. Report as suspected child abuse. D. Determine if the child takes bubble baths.

C. Report as suspected child abuse. Rationale: A green vaginal discharge is indicative of gonorrhea, a sexually transmitted disease. Because the child is 8 years old, the nurse should suspect child abuse and report the incident to the proper authorities (C). (A) is usually not related to infection. (B and D) are helpful in preventing bladder infections, but a green vaginal discharge is not a symptom of a bladder infection.

A practical nurse (PN) is reinforcing the steps for a dressing change for a male client who has a leg ulcer. When the client tries to change the dressing, he says he is inadequate, incompetent, and feels helpless. Which problem should the PN recognize that the client is exhibiting?

C. Self-esteem disturbance.

A client who is admitted for surgery seems to focus only on his immediate concerns and asks the practical nurse (PN) to repeat everything that is said over again. The client seems to follow directions but asks for assistance when filling out admission forms and checklists. He apologizes to the PN often and says he did not hear all of the instructions. This client is experiencing which level of anxiety?

C. Severe

The practical nurse (PN) is answering questions that the mother and her teenage daughter who is admitted with anorexia nervosa are asking about hospitalization. Which statement by the client's mother indicates to the PN that she understands this disease?

C. She sees herself as being very fat even though she is severely underweight.

A woman tells the practical nurse (PN) that for the past 6 months she has been terrified of leaving home. Whenever she thinks about going outdoors her heart pounds, she shakes and cries, and feels dizzy. Based on these findings, which nursing diagnosis should the practical nurse (PN) consider when caring for this client?

C. Social isolation related to avoidance behavior as evidenced by inability to go out of doors.

A nurse in the mental health unit is working with a group of adolescent girls with the diagnosis of anorexia nervosa. The nurse recalls that the major health complication associated with intractable anorexia nervosa is:

Cardiac dysrhythmias resulting in cardiac arrest

At a support meeting of parents of a teenager with polysubstance dependency, a parent states, "Each time my son tries to quit taking drugs, he gets so depressed that I'm afraid he will commit suicide." The nurse's response should be based on which information?

Careful monitoring should be provided during withdrawal from the drugs.

At a support meeting of parents of teenaged addicts, a parent states, "Each time my son tries to quit taking drugs, he gets so depressed that I'm afraid he will commit suicide." The nurse's response should be based on which information

Careful monitoring should be provided during withdrawal from the drugs.

To help establish a therapeutic nurse-client relationship, the mental health nurse uses various communication techniques to convey a willingness to listen and a genuine desire to view the client and his or her needs in a respectful manner. What is the primary underlying principle guiding this process?

Caring is the underlying component of nursing that promotes client care.

A client is admitted to the acute psychiatric unit of the local community hospital. The client is guarded and suspicious. After a thorough evaluation, a diagnosis of schizophrenia, paranoid type, is made. What initial approach should be used by the nurse assigned to establish a therapeutic one-to-one relationship with this client?

Casual and honest

The nurse is assigned to work with a 20-year-old client on an inpatient unit. In assessing the woman, the nurse notes that she is mute, does not show any type of movement, is unresponsive, and appears unaware of her surroundings. What is the best term for the nurse to use to describe these symptoms?

Catatonia

A hospitalized 7-year-old boy wakes up crying because he has wet his bed. It is most appropriate for the nurse to:

Change the child's bed while he changes his pajamas.

What should a nurse recognize that a client who uses the defense mechanism of sublimation is doing?

Channeling unacceptable impulses into socially approved behavior

A married woman is brought to the emergency department of a local hospital. Her eyes are swollen shut, and she has a bruise on her neck. She reports that she is being beaten by her husband. How does the nurse expect the husband to behave when he arrives at the emergency department?

Charming

A nurse understands that after the administration of alprazolam (Xanax) it is important to assess the client for side effects. Initially the nurse should:

Check the blood pressure

The community health nurse is teaching a class of high school students about bullying. One of the students asks about social factors that might contribute to violent behavior. Which response by the nurse is correct?

Children can learn aggressive behaviors by observing violence in the home.

Which paired drugs does the nurse expect the practitioner to prescribe for a client admitted for acute alcohol detoxification?

Chlordiazepoxide (Librium) and thiamine.

A client is admitted due to alcohol intoxication and injuries sustained in a fall. The client appears anxious, agitated, and diaphoretic. Vital signs include a pulse of 140 and a blood pressure of 170/98. Delirium is suspected due to the client's claim that bugs are crawling on the bed. Which medication should the nurse expect will be administered to the client? Chlordiazepoxide (Librium). Disulfiram (Antabuse). Acamprosate calcium (Campral). Mesalamine (Asacol HD).

Chlordiazepoxide (Librium).

A nurse is teaching clients in a medication education group about side effects of medications. Which drug will cause a heightened skin reaction to sunlight?

Chlorpromazine

A nurse is caring for clients who are undergoing therapy for dependence on alcohol. Which member of the health team has the primary responsibility for their rehabilitation?

Client

At a group therapy session, a member, in a teasing manner, makes several negative remarks about the nurse's appearance and behavior. The nurse can best respond by saying to the:

Client, "You seem very interested in my appearance and behavior. What's this all about?"

36. A 27-year-old female client is admitted to the psychiatric hospital with a diagnosis of bipolar disorder, manic phase. She is demanding and active. Which intervention should the nurse include in this client's plan of care? A) Schedule her to attend various group activities. B) Reinforce her ability to make her own decisions. C) Encourage her to identify feelings of anger. D) Provide a structured environment with little stimuli.

Clients in the manic phase of a bipolar disorder require decreased stimuli and a structured environment (D). Plan noncompetitive activities that can be carried out alone. (A) is contraindicated; stimuli should be reduced as much as possible. Impulsive decision-making is characteristic of clients with bipolar disorder. To prevent future complications, the nurse should monitor these clients' decisions and assist them in the decision-making process (B). (C) is more often associated with depression than with bipolar disorder. Correct Answer(s): D

36. A 27-year-old female client is admitted to the psychiatric hospital with a diagnosis of bipolar disorder, manic phase. She is demanding and active. Which intervention should the nurse include in this client's plan of care? A) Schedule her to attend various group activities. B) Reinforce her ability to make her own decisions. C) Encourage her to identify feelings of anger. D) Provide a structured environment with little stimuli.

Clients in the manic phase of a bipolar disorder require decreased stimuli and a structured environment (D). Plan noncompetitive activities that can be carried out alone. (A) is contraindicated; stimuli should be reduced as much as possible. Impulsive decision-making is characteristic of clients with bipolar disorder. To prevent future complications, the nurse should monitor these clients' decisions and assist them in the decision-making process (B). (C) is more often associated with depression than with bipolar disorder. Correct Answer(s): D

Agranulocytosis infection has fever, chills, tachycardia, oral ulcerations and is likely caused by

Clozpine (Clozaril)

A 52-year-old male client in the intensive care unit who has been oriented suddenly becomes disoriented and fearful. Assessment of vital signs and other physical parameters reveal no significant change and the nurse formulates the diagnosis, "Confusion related to ICU psychosis." Which intervention is best to implement?

Cluster care so that brief periods of rest can be scheduled during the day

A pregnant client with a history of delusions, hallucinations, and suspiciousness tells the nurse she is fearful about the upcoming birth and the health of her baby. What is the best initial approach by the nurse?

Commending the client on her ability to express her concerns

During the eighth session of a therapy group, a member who talks frequently is interrupted by one who doesn't. When the interrupting person is finished talking, the one who usually contributes says, "I'm so glad that you feel like talking today." While saying this, the client sits rigidly and looks angry. How should the nurse respond?

Comment on the interrupted client's angry behavior and pleasant words.

A client who has a long history of alcoholism has not worked for the past 10 years. When the nurse asks about daily activities the client responds, "I currently work in the office of a local construction company." Which mental mechanism should the nurse suspect that the client is using?

Confabulation

The nurse is caring for a client who has delusions of infidelity. The client's wife asks if there is any circumstance under which the treatment team is justified in violating the client's right to confidentiality. Which response is correct regarding exceptions to the client's right to confidentiality?

Confidentiality is waived if the client threatens the life of another person.

A male client with paranoid schizophrenia wraps his legs in toilet paper, believing that this will protect him from deadly germs contaminating the floor. What is the best nursing intervention?

Talking with the client about anxiety that focuses on health

The nurse is caring for a client who has delusions of infidelity. The client's wife asks if there is any circumstance under which the treatment team is justified in violating the client's right to confidentiality. Which response is correct regarding exceptions to the client's right to confidentiality? Confidentiality is waived if the client threatens the life of another person. Confidentiality is waived when law enforcement requests information. Confidentiality is waived only at the discretion of the psychiatrist. Confidentiality is to be maintained without exception.

Confidentiality is waived if the client threatens the life of another person. Rationale The duty to warn a person whose life has been threatened by a psychiatric client overrides the client's right to confidentiality. It is the legal responsibility of any health care personnel, the duty to warn when an individual threatens to harm or hurt themselves or another individual.

As a nurse enters a room and approaches a client who has schizophrenia, the client shouts, "Get out of here before I hit you! Go away!" The nurse concludes that this aggressive behavior is probably related to the fact that the client felt:

Confined when the nurse walked into the room

An interdisciplinary treatment team meets to discuss a client diagnosed with paranoid schizophrenia and cannabis abuse who is experiencing increased hallucinations and delusions. How should the team plan an effective treatment?

Consider each diagnosis primary and provide simultaneous treatment.

An interdisciplinary treatment team meets to discuss a client diagnosed with paranoid schizophrenia and cannabis abuse who is experiencing increased hallucinations and delusions. How should the team plan an effective treatment? Consider each diagnosis primary and provide simultaneous treatment. Treat the schizophrenia before establishing goals for substance abuse treatment. Withdraw the client from cannabis before treating the symptoms of schizophrenia. Hospitalize the client for the longest possible stay that insurance will allow.

Consider each diagnosis primary and provide simultaneous treatment. Rationale Clients with dual or co-occurring diagnoses, such as a substance use disorder with a psychiatric disorder, should be treated for both conditions simultaneously. They are both considered a primary diagnosis and need to be addressed for treatment to be effective.

A client on the psychiatric unit who is receiving high-dosage risperidone (Risperdal) is exhibiting tremors of the hands. What should be the nurse's first intervention?

Contacting the health care provider

A client is admitted to the psychiatric unit for severe depression with the potential for suicide. What is the most therapeutic nursing intervention when the client becomes more energized and communicative?

Continuing to assess the client at regular intervals.

Which topics should the nurse include in an education program for clients with schizophrenia and their families? (Select all that apply.) A. Importance of adherence to medication regimen B. Current treatment measures for substance abuse C. Signs and symptoms of an exacerbation D. Prevention of criminal activity E. Behavior modification for aggression F. Chronic grief associated with long-term illness

Correct Answer: A, C, F Rationale: Medication adherence is an important component of successful rehabilitation (A). Clients and their families also need to know the signs and symptoms of an exacerbation or relapse of the disease (C), which is frequently associated with poor medication compliance. Acknowledging the chronic sorrow associated with severe and persistent mental illness (F) helps individuals negotiate the grieving process. (B, D, and E) are not universal problems associated with schizophrenia.

During a home visit, a client with schizophrenia reports hearing voices that tell the client to walk in the middle of the street. The nurse records several statements made by the client. Based on which statement should the nurse determine that the client needs hospitalization? A. "Sometimes I take an extra one of my pills when I hear the voices." B. "The voices are louder when I forget to take my medication." C. "No matter what I do, I cannot make the voices go away." D. "I just try to tell the voices to stop when they bother me."

Correct Answer: C Rationale: Hospitalization is needed if the client continues to hear voices telling the client to do things that can cause self-harm (C). (A or B) do not require hospitalization unless symptoms become severe. The client should continue symptom management strategies (D) to prevent hospitalization.

40. A female client with obsessive-compulsive disorder (OCD) is describing her obsessions and compulsions and asks the nurse why these make her feel safer. What information should the nurse include in this client's teaching plan? (Select all that apply.) A) Compulsions relieve anxiety. B) Anxiety is the key reason for OCD. C) Obsessions cause compulsions. D) Obsessive thoughts are linked to levels of neurochemicals. E) Antidepressant medications increase serotonin levels.

Correct choices are (A, B, D, and E). To promote client understanding and compliance, the teaching plan should include explanations about the origin and treatment options of OCD symptomology. Compulsions are behaviors that help relieve anxiety (A), which is a vague feeling related to unknown fears, that motivate behavior (B) to help the client cope and feel secure. All obsessions (C) do not result in compulsive behavior. OCD is supported by the neurophysiology theory, which attributes a diminished level of neurochemicals (D), particularly serotonin, and responds to selective serotonin reuptake inhibitors (SSRI). Correct Answer(s): A, B, D, E

A client on the psychiatric service is pacing around the unit at a moderate rate and looking to either side of the hall. What is the most appropriate intervention by the nurse?

Talking with the client to assess the meaning of the behavior

40. A female client with obsessive-compulsive disorder (OCD) is describing her obsessions and compulsions and asks the nurse why these make her feel safer. What information should the nurse include in this client's teaching plan? (Select all that apply.) A) Compulsions relieve anxiety. B) Anxiety is the key reason for OCD. C) Obsessions cause compulsions. D) Obsessive thoughts are linked to levels of neurochemicals. E) Antidepressant medications increase serotonin levels.

Correct choices are (A, B, D, and E). To promote client understanding and compliance, the teaching plan should include explanations about the origin and treatment options of OCD symptomology. Compulsions are behaviors that help relieve anxiety (A), which is a vague feeling related to unknown fears, that motivate behavior (B) to help the client cope and feel secure. All obsessions (C) do not result in compulsive behavior. OCD is supported by the neurophysiology theory, which attributes a diminished level of neurochemicals (D), particularly serotonin, and responds to selective serotonin reuptake inhibitors (SSRI). Correct Answer(s): A, B, D, E

A client with stage 3 Alzheimer's disease is admitted to a behavioral health hospital. Due to the progression of the disease, the client is experiencing an increase in amnesia and agnosia and has declined to the point of not recognizing familiar objects and people. Which action should the nurse take to help reduce the client's fear and anxiety and adjust to the new setting?

Cover mirrors and pictures if they are upsetting the client.

A client with stage 3 Alzheimer's disease is admitted to a behavioral health hospital. Due to the progression of the disease, the client is experiencing an increase in amnesia and agnosia, and has declined to the point of not recognizing familiar objects and people. Which action should the nurse take to help reduce the client's fear and anxiety and adjust to the new setting? Cover mirrors and pictures if they are upsetting the client. Keep the TV on in the room throughout the day. Provide the client with a stuffed animal. Serve the client's meals in a private room.

Cover mirrors and pictures if they are upsetting the client.

The nurse is caring for a client with dementia whose expression of emotions is altered. Which behavior is unexpected with this client?

Curiosity

A male client who is hospitalized for depression ruminates over poor financial decisions that he made in the past and calls himself "stupid". Which strategy should the practical nurse (PN) implement to limit the amount of time the client spends on negative self-evaluation?

D. Schedule occupational therapy and unit activities for this client.

Pt teaching for trazodone

Teach pt to take med at night, slow position changes, and long lasting erection is a concern

An older client who is hospitalized with pneumonia becomes disoriented and confused 2 days after admission. Which factor should the practical nurse (PN) identify to differentiate that the client is experiencing delirium, not dementia?

D. Acute onset of symptoms.

A client mumbles out loud whether anyone is talking to her or not, and the client also mumbles in group when others are talking. The nurse determines that the client is experiencing hallucinations. Which intervention should the nurse implement? A. Respond to the client's feelings rather than the illogical thoughts. B. Identify beliefs and thoughts about what the client is experiencing. C. Provide the client with hope that the voices will eventually go away. D. Ask the client how she has previously managed the voices.

D. Ask the client how she has previously managed the voices. Rationale: The nurse should promote symptom management and determine how the client previously managed the voices (D). (A and B) are interventions that are useful with clients who are experiencing delusions. (C) is important, but the most important intervention is to promote symptom management.

Which nursing intervention is best to help a female client with progressive memory deficit?

D. Assist the client to perform simple tasks by giving step-by-step directions.

A practical nurse (PN) is interacting with a female client who is discussing her divorce as a stressor. What areas should be explored with the client to gather the most relevant information?

D. Biopsychosocial responses.

During a prenatal visit, a client who is in the second trimester of pregnancy tells the practical nurse (PN) that she is using cocaine. What information about cocaine is most important for the PN to provide the client?

D. Cocaine can cause miscarriage or premature onset of labor.

The practical nurse (PN) is caring for a female client with chronic psychosis who repeatedly tells the PN that her arm is missing and she cannot participate in the group activities. Which response should the PN offer when providing reality validation to the client?

D. Do you mean, it feels like your arm is missing?

A 20-year-old male client who is admitted to the mental health unit for adjustment disorder is telling the practical nurse (PN) that he wants to find an apartment, but he is afraid he does not make enough money to move out of his parent's home. Using Erikson's theory of psychosocial development, which developmental stage should the PN explore with this client?

D. Sense of freedom in the community.

During a routine prenatal visit, the practical nurse (PN) is assessing a pregnant female client who expresses fears of spousal abuse. Which information should the PN provide to facilitate client disclosure?

D. Share with the client that her situation is not unique and abuse often increases with pregnancy.

A woman who has severe rheumatoid arthritis becomes depressed and is admitted to the psychiatric unit. The nurse begins to work with her in one-on-one sessions to help her cope with her depressive episode. The best long-term goal for this client is that she will:

Decrease negative thinking about herself, others, and life.

Naltrexone (Depade) is used to treat clients with substance abuse problems. In which situation does the nurse anticipate that naltrexone will be administered?

Decrease the recovering alcoholic's desire to drink alcohol.

A homeless person who is in the manic phase of bipolar disorder is admitted to the mental health unit.Which laboratory finding obtained on admission is most important for the nurse to report to the healthcare provider?

Decreased thyroid stimulating hormone level

A student nurse working as an aide in a memory care facility asks the charge nurse if there is a neurobiological basis for the deterioration in cognitive function in Alzheimer's disease. Which explanation by the nurse is correct regarding the etiology of neurocognitive decline?

Decreases in neurotransmitters affect parts of the brain responsible for memory."

A student nurse working as an aide in a memory care facility asks the charge nurse if there is a neurobiological basis for the deterioration in cognitive function in Alzheimer's disease. Which explanation by the nurse is correct regarding the etiology of neurocognitive decline? "Decreases in neurotransmitters affect parts of the brain responsible for memory." "A decrease in body mass causes a decrease in the mass of the brain." "Untreated psychological problems slow down the thinking and reasoning processes." "Nutritional deficiencies cause a decrease in metabolism that inhibits the amount of glucose available to the brain."

Decreases in neurotransmitters affect parts of the brain responsible for memory."

A nurse approaches a depressed client who has just been admitted to the psychiatric unit and says, "Hello! I'm Andrea, your nurse. I'll introduce you and help you settle in with the others here. We'll also talk about anything that concerns you." These statements establish the nurse-client relationship by:

Defining boundaries

A client on the psychiatric unit tells the nurse, "I'm a movie star, and the other clients are my audience." What is an appropriate conclusion for the nurse to document about what the client is experiencing?

Delusion of grandeur

24. On admission, a highly anxious client is described as delusional. The nurse understands that delusions are most likely to occur with which class of disorder? A) Neurotic. B) Personality. C) Anxiety. D) Psychotic.

Delusions are false beliefs associated with psychotic behavior, and psychotic persons are not in touch with reality (D). (A, B, and C) are mental health disorders which are not associated with a break in reality, nor with hallucinations (false sensations such as hearing, or seeing) or delusions (false beliefs). Correct Answer(s): D

24. On admission, a highly anxious client is described as delusional. The nurse understands that delusions are most likely to occur with which class of disorder? A) Neurotic. B) Personality. C) Anxiety. D) Psychotic.

Delusions are false beliefs associated with psychotic behavior, and psychotic persons are not in touch with reality (D). (A, B, and C) are mental health disorders which are not associated with a break in reality, nor with hallucinations (false sensations such as hearing, or seeing) or delusions (false beliefs). Correct Answer(s): D

19. An adult male client who was admitted to the mental health unit yesterday tells the nurse that microchips were planted in his head for military surveillance of his every move. Which response is best for the nurse to provide? A) You are in the hospital, and I am the nurse caring for you. B) It must be difficult for you to control your anxious feelings. C) Go to occupational therapy and start a project. D) You are not in a war area now; this is the United States.

Delusions often generate fear and isolation, so the nurse should help the client participate in activities that avoid focusing on the false belief and encourage interaction with others (C). Delusions are often well-fixed, and though (A) reinforces reality, it is argumentative and dismisses the client's fears. It is often difficult for the client to recognize the relationship between delusions and anxiety (B), and the nurse should reassure the client that he is in a safe place. Dismissing delusional thinking (D) is unrealistic because neurochemical imbalances that cause positive symptoms of schizophrenia require antipsychotic drug therapy. Correct Answer(s): C

19. An adult male client who was admitted to the mental health unit yesterday tells the nurse that microchips were planted in his head for military surveillance of his every move. Which response is best for the nurse to provide? A) You are in the hospital, and I am the nurse caring for you. B) It must be difficult for you to control your anxious feelings. C) Go to occupational therapy and start a project. D) You are not in a war area now; this is the United States.

Delusions often generate fear and isolation, so the nurse should help the client participate in activities that avoid focusing on the false belief and encourage interaction with others (C). Delusions are often well-fixed, and though (A) reinforces reality, it is argumentative and dismisses the client's fears. It is often difficult for the client to recognize the relationship between delusions and anxiety (B), and the nurse should reassure the client that he is in a safe place. Dismissing delusional thinking (D) is unrealistic because neurochemical imbalances that cause positive symptoms of schizophrenia require antipsychotic drug therapy. Correct Answer(s): C

A 70-year-old retired man has difficulty remembering his daily schedule and finding the right words to express himself. He is found to have dementia of the Alzheimer type. The nurse knows that symptoms of this disorder:

Demonstrate a progression of disintegration

An adolescent with a conduct disorder is undergoing behavioral therapy in an attempt to limit behaviors that violate societal norms. A specific outcome criterion unique to adolescents with this problem is:

Demonstration of respect for the rights of others

A mental health nurse is working on a unit where many clients have the diagnosis of alcoholism. The nurse identifies that the defense mechanism most commonly used by clients who are alcoholics is:

Denial.

A major recognizable difference between anorexia nervosa and bulimia nervosa is that clients with anorexia nervosa usually:

Deny the problem, whereas clients with bulimia generally recognize that their eating pattern is abnormal

A female client who is severely incapacitated by obsessive-compulsive behavior has been admitted to the mental health hospital. The client's compulsive ritual involves changing her clothing eight to 12 times a day. She continually asks the nurse for advice regarding her problems but then ignores it. This is an example of the conflict of:

Dependence versus independence

An older client in a long-term care facility is demonstrating excessive sleeping, fatigue, inability to concentrate, weight loss, and slow body movements. The nurse suspects this client is experiencing:

Depression

As a client addicted to cocaine withdraws from the drug, the nurse should expect to observe behavior related to:

Depression

Duloxetine is given for

Depression AND pain (neuropathy and fibromyalgia) -Think of DUL as in DUAL purpose for depression and pain -And think of DUL as it will DULL the pain Test tip - questions may ask why the pt is taking this med if they are NOT depressed.

58. The nurse is planning the care for a 32-year-old male client with acute depression. Which nursing intervention bests helps this client deal with his depression? A) Ensure that the client's day is filled with group activities. B) Assist the client in exploring feelings of shame, anger, and guilt. C) Allow the client to initiate and determine activities of daily living. D) Encourage the client to explore the rationale for his depression.

Depression is associated with feelings of shame, anger, and guilt. Exploring such feelings is an important nursing intervention for the depressed client (B). If the client's day is filled with group activities (A) he might not have the opportunity to explore these feelings. (C) is a good intervention for the chronically depressed client who exhibits vegetative signs of depression. (D) is essentially asking the client "why" he is depressed--avoid "why's" disguised as "rationale." Correct Answer(s): B

A client with a history of violence is becoming increasingly agitated. Which nursing intervention will most likely increase the risk of acting-out behavior?

Teaching relaxation

58. The nurse is planning the care for a 32-year-old male client with acute depression. Which nursing intervention bests helps this client deal with his depression? A) Ensure that the client's day is filled with group activities. B) Assist the client in exploring feelings of shame, anger, and guilt. C) Allow the client to initiate and determine activities of daily living. D) Encourage the client to explore the rationale for his depression.

Depression is associated with feelings of shame, anger, and guilt. Exploring such feelings is an important nursing intervention for the depressed client (B). If the client's day is filled with group activities (A) he might not have the opportunity to explore these feelings. (C) is a good intervention for the chronically depressed client who exhibits vegetative signs of depression. (D) is essentially asking the client "why" he is depressed--avoid "why's" disguised as "rationale." Correct Answer(s): B

The nurse determines that the therapy that has the highest success rate for people with phobias is:

Desensitization involving relaxation techniques

What should a nurse include in the plan of care for a client with vascular dementia?

Details of supportive care interventions

55. A 65-year-old female client complains to the nurse that recently she has been hearing voices. What question should the nurse ask this client first? A) Do you have problems with hallucinations? B) Are you ever alone when you hear the voices? C) Has anyone in your family had hearing problems? D) Do you see things that others cannot see?

Determining if the client is alone when she hears voices (B) will assist in differentiating between hallucinations and hearing loss; this is especially important in the aging population. If the client is experiencing hallucinations, the voices will be real to her, and it is unlikely that (A) would provide accurate information. (C and D) might be good follow-up questions, but would not have the priority of (B). Correct Answer(s): B

55. A 65-year-old female client complains to the nurse that recently she has been hearing voices. What question should the nurse ask this client first? A) Do you have problems with hallucinations? B) Are you ever alone when you hear the voices? C) Has anyone in your family had hearing problems? D) Do you see things that others cannot see?

Determining if the client is alone when she hears voices (B) will assist in differentiating between hallucinations and hearing loss; this is especially important in the aging population. If the client is experiencing hallucinations, the voices will be real to her, and it is unlikely that (A) would provide accurate information. (C and D) might be good follow-up questions, but would not have the priority of (B). Correct Answer(s): B

What is the most difficult initial task in the development of a nurse-client relationship?

Developing an awareness of self and the professional role in the relationship

A nurse is caring for a newly admitted, extremely depressed client. The most appropriate initial goal for the client is:

Developing trust in others

The psychiatrist is concerned that one of the clients receiving haloperidol (Haldol) may be developing neuroleptic malignant syndrome. When assessing the client for this syndrome, for which clinical manifestations should the nurse monitor the client?

Diaphoresis and hyperpyrexia

A client with long-term alcohol addiction is admitted to the emergency department. Which medications should the nurse anticipate the healthcare provider will prescribe for this client? Diazepam. Methadone. Multivitamins. Thiamine (vitamin B1). Monoamine oxidase inhibitors.

Diazepam. Multivitamins. Thiamine (vitamin B1).

When caring for a client with bulimia nervosa, the nurse remembers that bulimia nervosa follows a cyclical pattern. The nurse identifies the first pattern in this cycle as:

Dieting in an attempt to maintain control of one's life

Fourteen months after the traumatic death of a spouse, a client comes to the mental health clinic complaining of continuing depression and states, "I haven't been seeing any of my friends or attending any of the activities I previously enjoyed. My children are married and live in another state, and I almost never see them." What does the nurse determine that the client is experiencing?

Difficulty grieving

A nurse is caring for a client with vascular dementia. What does the nurse expect of this client's mental status?

Difficulty recalling recent events related to cerebral hypoxia

The nurse is reviewing labs for a client who is taking clozapine, 25mg QD for treatment of disorganized schizophrenia. The labs are documented as follows: RBC 4.5 million/mcL, WBC 1000/mcL, TSH 1.2 mc-IU. Based on these results, which order should the nurse anticipate the health care provider to write?

Discontinue clozapine.

A health care provider informs family members that their grandfather has a reversible form of dementia. Which condition is associated with reversible dementia?

Fluid and electrolyte imbalance.

The nurse is reviewing labs for a client who is taking clozapine, 25mg QD for treatment of disorganized schizophrenia. The labs are documented as follows: RBC 4.5 million/mcL, WBC 1000/mcL, TSH 1.2 mc-IU. Based on these results, which order should the nurse anticipate the health care provider to write? Discontinue clozapine. Administer levothyroxine sodium 25 mcg QD. Administer ferrous sulfate 100mg QD. Decrease clozapine to 12.5 mg QD, start levothyroxine sodium 50 mcg QD.

Discontinue clozapine. Rationale Agranulocytosis is an adverse side effect of clozapine that can cause lethal infections. Based on the client's results, clozapine should be discontinued; the thyroid and RBC results are normal and do not need to be treated.

74. A client who is diagnosed with schizophrenia is admitted to the hospital. The nurse assesses the client's mental status. Which assessment finding is most characteristic of a client with schizophrenia? A) Mood swings. B) Extreme sadness. C) Manipulative behavior. D) Flat affect.

Disinterest, and diminished or lack of facial expression is characteristic of schizophrenia and is referred to as a flat affect (D). (A) is associated with bipolar disorder. (B) is associated with depression. (C) is usually associated with personality disorders and is often seen in clients who abuse substances. Correct Answer(s): D

74. A client who is diagnosed with schizophrenia is admitted to the hospital. The nurse assesses the client's mental status. Which assessment finding is most characteristic of a client with schizophrenia? A) Mood swings. B) Extreme sadness. C) Manipulative behavior. D) Flat affect.

Disinterest, and diminished or lack of facial expression is characteristic of schizophrenia and is referred to as a flat affect (D). (A) is associated with bipolar disorder. (B) is associated with depression. (C) is usually associated with personality disorders and is often seen in clients who abuse substances. Correct Answer(s): D

A client who has just experienced her second spontaneous abortion expresses anger toward the practitioner, the hospital, and the "rotten nursing care." When assessing the situation, the nurse concludes that the client may be using the coping mechanism of:

Displacement

A male long-distance jumper improves his distance by 3½ inches (7 cm) and earns the praise of his coach, but on another day, when he does not reach his mark, he forcefully kicks the door of his locker. What defense mechanism does his outburst demonstrate?

Displacement

A nurse is teaching a class about child abuse. What defense mechanism most often used by the physically abusive individual should the nurse include?

Displacement

A nurse greets a client who has been experiencing delusions of persecution and auditory hallucinations by saying, "Good evening. How are you?" The client, who has been referring to himself as "the man," answers, "The man is bad." Of what is this an example?

Dissociation

71. A client with bipolar disorder on the mental health unit becomes loud, and shouts at one of the nurses, "You fat tub of lard! Get something done around here!" What is the best initial action for the nurse to take? A) Have the orderly escort the client to his room. B) Tell the client his healthcare provider will be notified if he continues to be verbally abusive. C) Redirect the client's energy by asking him to tidy the recreation room. D) Call the healthcare provider to obtain a prescription for a sedative.

Distracting the client, or redirecting his energy (C), prevents further escalation of the inappropriate behavior. (A) could result in escalating the abuse and unnecessarily involve another staff member in the abusive situation. (B) is a threat and is using a health team member (healthcare provider) as the threat. (D) may be indicated if the behavior escalates, but, at this time, the best initial action is (C). Correct Answer(s): C

A health care provider informs family members that their grandfather has a reversible form of dementia. Which condition is associated with reversible dementia? HIV infection. Parkinson's disease. Fluid and electrolyte imbalance. Vascular disease.

Fluid and electrolyte imbalance.

What is the antidote for benzodiazepine?

Flumazenil

71. A client with bipolar disorder on the mental health unit becomes loud, and shouts at one of the nurses, "You fat tub of lard! Get something done around here!" What is the best initial action for the nurse to take? A) Have the orderly escort the client to his room. B) Tell the client his healthcare provider will be notified if he continues to be verbally abusive. C) Redirect the client's energy by asking him to tidy the recreation room. D) Call the healthcare provider to obtain a prescription for a sedative.

Distracting the client, or redirecting his energy (C), prevents further escalation of the inappropriate behavior. (A) could result in escalating the abuse and unnecessarily involve another staff member in the abusive situation. (B) is a threat and is using a health team member (healthcare provider) as the threat. (D) may be indicated if the behavior escalates, but, at this time, the best initial action is (C). Correct Answer(s): C

44. A 35-year-old male client who has been hospitalized for two weeks for chronic paranoia continues to state that someone is trying to steal his clothing. Which action should the nurse implement? A) Encourage the client to actively participate in assigned activities on the unit. B) Place a lock on the client's closet. C) Ignore the client's paranoid ideation to extinguish these behaviors. D) Explain to the client that his suspicions are false.

Diverting the client's attention from paranoid ideation and encouraging him to complete assignments can be helpful in assisting him to develop a positive self-image (A). The client's problem is not security, and (B) actually supports his paranoid ideation. (C) is not correct because ignoring the client's symptoms may lower his self-esteem. The nurse should not argue with the client about his delusions (D), and should not try to reason with the client regarding his paranoid ideation. Correct Answer(s): A

44. A 35-year-old male client who has been hospitalized for two weeks for chronic paranoia continues to state that someone is trying to steal his clothing. Which action should the nurse implement? A) Encourage the client to actively participate in assigned activities on the unit. B) Place a lock on the client's closet. C) Ignore the client's paranoid ideation to extinguish these behaviors. D) Explain to the client that his suspicions are false.

Diverting the client's attention from paranoid ideation and encouraging him to complete assignments can be helpful in assisting him to develop a positive self-image (A). The client's problem is not security, and (B) actually supports his paranoid ideation. (C) is not correct because ignoring the client's symptoms may lower his self-esteem. The nurse should not argue with the client about his delusions (D), and should not try to reason with the client regarding his paranoid ideation. Correct Answer(s): A

A nurse who plans to care for a client with an obsessive-compulsive disorder should understand that the client's personality is usually characterized by:

Doubts, fears, and indecisiveness

A nurse is administering hydroxyzine (Vistaril) to a client. For which common side effects of this drug should the nurse monitor the client?

Drowsiness and dry mouth

The nurse reviews the laboratory findings for a client's urine drug screen that is positive for cocaine. Which client behavior should be expected during cocaine withdrawal? A. Psychomotor agitation B. Restlessness and hyperactivity C. Detachment from reality and drowsiness D. Distorted perceptions and hallucinations

During cocaine withdrawal, the nurse should expect option A and a pattern of withdrawal symptoms similar to those of one who uses amphetamines. Options B, C, and D are signs and symptoms of a person who is high on cocaine rather than one who is experiencing withdrawal from cocaine.

A nurse understands that autism is a form of a pervasive developmental disorder (PDD). Which factor unique to autism differentiates it from other forms of PDD?

Early onset, before 36 months of age

5. A client who is being treated with lithium carbonate for bipolar disorder develops diarrhea, vomiting, and drowsiness. What action should the nurse take? A) Notify the healthcare provider immediately and prepare for administration of an antidote. B) Notify the healthcare provider of the symptoms prior to the next administration of the drug. C) Record the symptoms as normal side effects and continue administration of the prescribed dosage. D) Hold the medication and refuse to administer additional amounts of the drug.

Early side effects of lithium carbonate (occurring with serum lithium levels below 2.0 mEq per liter) generally follow a progressive pattern beginning with diarrhea, vomiting, drowsiness, and muscular weakness. At higher levels, ataxia, tinnitus, blurred vision, and large dilute urine output may occur. (B) is the best choice. Although these are expected symptoms, the healthcare provider should be notified prior to the next administration of the drug. (A, C, and D) would not reflect good nursing judgment. Correct Answer(s): B

What medication should the nurse expect to administer to actively reverse the overdose sedative effects of benzodiazepines?

Flumazenil

The school nurse is providing an in-service program to high school girls about date rape drugs. A student expresses interest about particular drugs and asks the nurse what these drugs look like and how they affect the body. Which response by the nurse it correct?

Flunitrazepam (Rohypnol) is a pill that dissolves in liquids and causes muscle relaxation and amnesia.

5. A client who is being treated with lithium carbonate for bipolar disorder develops diarrhea, vomiting, and drowsiness. What action should the nurse take? A) Notify the healthcare provider immediately and prepare for administration of an antidote. B) Notify the healthcare provider of the symptoms prior to the next administration of the drug. C) Record the symptoms as normal side effects and continue administration of the prescribed dosage. D) Hold the medication and refuse to administer additional amounts of the drug.

Early side effects of lithium carbonate (occurring with serum lithium levels below 2.0 mEq per liter) generally follow a progressive pattern beginning with diarrhea, vomiting, drowsiness, and muscular weakness. At higher levels, ataxia, tinnitus, blurred vision, and large dilute urine output may occur. (B) is the best choice. Although these are expected symptoms, the healthcare provider should be notified prior to the next administration of the drug. (A, C, and D) would not reflect good nursing judgment. Correct Answer(s): B

An older adult with a chronic degenerative disease progresses to the stage at which self-care is no longer possible, and admission to a long-term care facility becomes necessary. What is the major developmental conflict for this client, according to Erikson?

Ego integrity versus despair

A client is scheduled for a 6-week electroconvulsive therapy (ECT) treatment program. What intervention is important during the course of treatment?

Elimination of benzodiazepines for nighttime sedation

It is most helpful to the nurse who is attempting to apply the principles of mental health to consider that:

Emotional health is promoted when there is a sense of mastery of self and the environment.

he nurse is interacting with the spouse of a client being treated for substance abuse. During the interview, the spouse makes excuses and takes the blame for the client's actions. Which terms best describe the spouse's behavior? Loyal. Enabling. Tolerance. Antagonistic. Co-dependency.

Enabling. Co-dependency. Rationale Individual's who enable another or create a co-dependent relationship often enjoy the feeling of being needed.

A 35-year-old male client who has been hospitalized for two weeks for chronic paranoia continues tostate that someone is trying to steal his clothing. Which action should the nurse implement?

Encourage the client to actively participate in assigned activities on the unit

A female client has terminal cancer. Her family members are concerned because she appears to be accepting less and less responsibility for her own care. What should the nurse do to help family members plan for the client's care?

Encourage them to accept her regression until she can cope more effectively.

The nurse is planning care for a confused, delusional client. What should be included in the plan to render it as therapeutic as possible?

Encouraging realistic activity based on the client's ability

A client who has been on a psychiatric unit for several weeks continually talks about delusional topics. What response by the nurse is most therapeutic?

Encouraging the client to focus on reality issues

A client becomes angry and threatens another client. What is the nurse's most therapeutic intervention?

Encouraging the client to talk about why he or she became angry and then aggressive

What therapeutic nursing intervention may redirect a hyperactive, manic client?

Encouraging the client to tear pictures out of magazines for a scrapbook

What is the most appropriate nursing intervention when a client is seen openly masturbating in the recreation room?

Escorting the client from the room

The nurse is caring for a client who has attempted suicide. What is the most desirable short-term client outcome during this crisis situation?

Establishing a no-suicide contract

What action should the nurse take when it becomes apparent that communication between the nurse and the client is consistently superficial?

Evaluating how actively the nurse has been listening to the client

A client is receiving substitution therapy during withdrawal from benzodiazepines. Which expected outcome statement has the highest priority when planning nursing care?

Excessive CNS stimulation will be reduced

Which elements have been shown to be beneficial when incorporated into a stress management plan? Retreating from friends. Exercise. Mindfulness practices. Social media. Balanced diet.

Exercise. Mindfulness practices. Balanced diet. Rationale Sleep is essential to engage in activities of daily living. Exercise has been found to decrease stress levels and cortisol levels and increase sense of well-being. Relaxation reduces psychological or physiological distress. Multitasking facilitated by technology has been identified as a significant source of stress. Well-balanced diet can facilitate stress reduction and improve physical well-being.

The nurse is planning therapeutic group sessions for regressed long-term clients. The nurse understands that these clients need to:

Experience a structured setting.

A nurse develops a relationship with a client who has bipolar disorder with episodes of mania. The nurse concludes that their therapeutic interaction has entered the working stage when the client:

Explores the effect of bipolar behavior on the family

A client is admitted with a conversion disorder. What is the primary nursing intervention?

Exploring ways to verbalize feelings

he interdisciplinary team is planning treatment for a client with stage 2 Alzheimer's disease. The team agrees that involving the client in more therapy groups and activities could help improve memory and relieve some depressive symptoms. The team leader recommends reminiscence therapy. Which statement

Facilitates discussions of life's transitions and evokes memories from the client's past.

he interdisciplinary team is planning treatment for a client with stage 2 Alzheimer's disease. The team agrees that involving the client in more therapy groups and activities could help improve memory and relieve some depressive symptoms. The team leader recommends reminiscence therapy. Which statement Facilitates discussions of life's transitions and evokes memories from the client's past. Encourages clients to pick a topic of concern and discuss their fears. Introduces the client to new forms of recreational therapy that may improve memory. Enables clients to develop new skills based on their previous skill sets.

Facilitates discussions of life's transitions and evokes memories from the client's past.

A client arrives at the clinic and tells the nurse about various aches and pains since her spouse's death 3 months ago. The client appears depressed and tense. What is the initial nursing intervention?

Facilitating a discussion of the spouse's death

During a group discussion it is learned that a group member hid suicidal urges and committed suicide several days ago. The nurse leading the group should be prepared to manage the:

Fear by some members that their own suicidal urges may go unnoticed and that they may go unprotected

A client comes to the mental health clinic for treatment of a phobia of large dogs. The nurse should anticipate that this client will demonstrate:

Fear of discussing the phobia

A nurse should base care for grieving clients on the knowledge that the grieving process may last longer for people who have:

Feelings of guilt

The nurse should hold the next scheduled dose of a client's haloperidol (Haldol) based on which assessment finding(s)

Fever of 102° F

Lithium carbonate 600 mg by mouth three times a day is prescribed for a client. The nurse concludes that the teaching about its side effects is understood when the client says that she will call her primary health care provider immediately if she notices any:

Fine hand tremor or slurred speech

The nurse explains to a nursing assistant that behavior usually is viewed and accepted as normal if it:

Fits within standards accepted by one's society

A client who is diagnosed with schizophrenia is admitted to the hospital. The nurse assesses the client's mental status. Which assessment finding is most characteristic of a client with schizophrenia?

Flat affect

The school nurse is providing an in-service program to high school girls about date rape drugs. A student expresses interest about particular drugs and asks the nurse what these drugs look like and how they affect the body. Which response by the nurse it correct? Flunitrazepam (Rohypnol) is a pill that dissolves in liquids and causes muscle relaxation and amnesia. Diphenhydramine (Compose) is a pill or capsule that is used to induce sleep. Acetaminophen plus ephedrine (NyQuil) comes in pill and liquid form and causes drowsiness. Cyclobenzaprine (Flexeril) comes in a pill form and causes drowsiness and relaxation.

Flunitrazepam (Rohypnol) is a pill that dissolves in liquids and causes muscle relaxation and amnesia. Rationale Flunitrazepam (Rohypnol) is a date rape pill that is also called "forget-me-not." It dissolves in water, becomes more potent when combined with alcohol, and causes relaxation and amnesia.

A 22-year-old client is admitted to the psychiatric unit from the medical unit following a suicide attempt with an overdose of diazepam. When developing the nursing care plan for this client, which intervention would be most important for the nurse to include?

Focus on own personal strengths Encouraging the client to focus on his or her strengths helps the client become aware of positive qualities, assists in improving self-image, and aids in coping with past and present situations. Although nursing actions should assist the client in decreasing self-defacing comments and informing the client of daily activities in the milieu, these interventions are not priorities at this time. Option D is not as important as assisting the client to overcome the depression, which resulted in the overdose, and asking "why" is not therapeutic.

A nurse encourages a client to attend Alcoholics Anonymous (AA) meetings after discharge. What do self-help groups such as AA help their members do?

Foster changes in behavior.

35. A male client is admitted to the psychiatric unit with a medical diagnosis of paranoid schizophrenia. During the admission procedure, the client looks up and states, "No, it's not MY fault. You can't blame me. I didn't kill him, you did." What action is best for the nurse to take? A) Reassure the client by telling him that his fear of the admission procedure is to be expected. B) Tell the client that no one is accusing him of murder and remind him that the hospital is a safe place. C) Assess the content of the hallucinations by asking the client what he is hearing. D) Ignore the behavior and make no response at all to his delusional statements.

Further assessment is indicated (C). The nurse should obtain information about what the client believes the voices are telling him--they may be telling him to kill the nurse! (A) is telling the client how he feels (fearful). The nurse should leave communications open and seek more information. (B) is arguing with the client's delusion, and the nurse should never argue with a client's hallucinations or delusions, also (B) is possibly offering false reassurance. (D) is avoiding the situation and the client's needs. Correct Answer(s): C

35. A male client is admitted to the psychiatric unit with a medical diagnosis of paranoid schizophrenia. During the admission procedure, the client looks up and states, "No, it's not MY fault. You can't blame me. I didn't kill him, you did." What action is best for the nurse to take? A) Reassure the client by telling him that his fear of the admission procedure is to be expected. B) Tell the client that no one is accusing him of murder and remind him that the hospital is a safe place. C) Assess the content of the hallucinations by asking the client what he is hearing. D) Ignore the behavior and make no response at all to his delusional statements.

Further assessment is indicated (C). The nurse should obtain information about what the client believes the voices are telling him--they may be telling him to kill the nurse! (A) is telling the client how he feels (fearful). The nurse should leave communications open and seek more information. (B) is arguing with the client's delusion, and the nurse should never argue with a client's hallucinations or delusions, also (B) is possibly offering false reassurance. (D) is avoiding the situation and the client's needs. Correct Answer(s): C

An addiction counselor is teaching a client and family about cross tolerance, cross addiction and alcohol, benzodiazepines, and barbiturates. The counselor is also teaching about the dangers of concurrent use. The nurse is correct to state that alcohol and CNS depressants act on which receptors?

GABA.

An addiction counselor is teaching a client and family about cross tolerance, cross addiction and alcohol, benzodiazepines, and barbiturates. The counselor is also teaching about the dangers of concurrent use. The nurse is correct to state that alcohol and CNS depressants act on which receptors? GABA. Serotonin. Dopamine. Opioid.

GABA.

The health care provider is performing a follow-up exam on a 17-year-old-client who was raped six months ago. The client states that she completed counseling and has been feeling like she is starting to put the experience behind her. Which action by the client is a sign of recovery? Going to her senior prom with her boyfriend. Taking sertraline (Zoloft) for anxiety. Expressing a need for more counseling. Refusing to go out alone in public.

Going to her senior prom with her boyfriend.

The nurse is interviewing a client who has recently been admitted for evaluation of a thought disorder manifested by paranoid behavior. According to the client's mother, the client was previously treated for anxiety, but has become more isolated and withdrawn over the last few weeks. He refuses to leave his room, and he states that he is the "King of Mars" and someone is trying to assassinate him. Which type of delusion should the nurse document?

Grandiose.

The nurse is interviewing a client who has recently been admitted for evaluation of a thought disorder manifested by paranoid behavior. According to the client's mother, the client was previously treated for anxiety, but has become more isolated and withdrawn over the last few weeks. He refuses to leave his room, and he states that he is the "King of Mars" and someone is trying to assassinate him. Which type of delusion should the nurse document? Jealous. Grandiose. Somatic. Erotomanic.

Grandiose.

Which characteristics have been associated with substance abuse? Self reflection. Grandiosity. Manipulation. Low risk taking. Dysfunctional anger.

Grandiosity. Manipulation. Dysfunctional anger.

A nurse begins terminating the consistent one-to-one relationship with a client who is soon to be discharged. How might the nurse expect the client to respond to the termination of their relationship?

Grief

A mother whose daughter has been killed in a school bus accident tells the nurse that her daughter was just getting over the chickenpox and did not want to go to school but she insisted that the girl go. The mother cries bitterly and says that her child's death is her fault. The nurse understands that perceiving a death as preventable most often will influence the grieving process in that it may:

Grow in intensity and duration

Acute dystonia can be induced by

Haldole and presents with painful muscle spasms in the neck, jaw, eyes

A young adult being treated for substance abuse asks the nurse about methadone. The nurse responds that methadone is useful in the treatment of opioid addiction because it:

Has an effect of longer duration

A 65-year-old client is receiving amitriptyline (Elavil). What is the most important recommendation for the nurse to make to this client concerning this medication?

Have an eye examination to check for glaucoma.

An antianxiety medication is prescribed for an extremely anxious client. The client says, "I'm afraid to take these pills because I heard they're addictive." The nurse teaches the client that antianxiety medications:

Have the potential for physiological and psychological dependence

A nurse is caring for a 20-year-old client. According to Erikson's developmental psychosocial theory, what is expected by 20 years of age?

Having a coherent sense of self and plans for self-actualization

Imipramine (Tofranil), 75 mg three times per day, is prescribed for a client. What nursing action is appropriate when this medication is being administered?

Having the client checked for increased intraocular pressure and teaching about symptoms of glaucoma

Which activity is the least therapeutic for a severely depressed client?

Having the client select an activity

Remember when you take a BenzO everything is LOW and SLOW

Heart rate, respiratory and sedation

To begin to establish a therapeutic relationship with a withdrawn, reclusive client, the nurse must:

Help the client keep anxiety to a minimum.

The nurse is caring for a 64 year old with schizophrenia admitted with chills, sore throat and painful ulcers in their mouth. Based on the client's presentation, what intervention is most important for the nurse to implement? HR 110 BP 100/60 RR 20 SpO2 95% Temp 103 Medication Aspirin 81 mg PO daily Clozapine (Clozaril) 300 mg PO BID Levothyroxine 25 mcg PO daily

Hold next dose of clozapine (clozaril)

The nurse is caring for a 72-year-old client who was admitted for treatment of depression. The nurse notices that the health care provider's order for an antidepressant calls for a dose greater than the usual adult dose. Which action is the correct action for the nurse to take?

Hold the medication and consult the health care provider.

The nurse is caring for a 72-year-old client who was admitted for treatment of depression. The nurse notices that the health care provider's order for an antidepressant calls for a dose greater than the usual adult dose. Which action is the correct action for the nurse to take? Hold the medication and consult the health care provider. Check the electronic formulary on the unit's computer. Give the usual adult dose. Implement the order as prescribed.

Hold the medication and consult the health care provider.

The nurse is caring for a 72-year-old client who was admitted for treatment of depression. The nurse notices that the health care provider's order for an antidepressant calls for a dose greater than the usual adult dose. Which action is the correct action for the nurse to take? Hold the medication and consult the health care provider. Check the electronic formulary on the unit's computer. Give the usual adult dose. Implement the order as prescribed.

Hold the medication and consult the health care provider.

A psychiatric nurse is hired to work in the psychiatric emergency department of a large teaching hospital. While reviewing the manuals, the nurse reads, "People with mental health emergencies shall be triaged within 5 minutes of entering the emergency department." What does the nurse consider this statement to represent?

Hospital policy

One morning, during the working phase of a therapeutic relationship after several sessions in which difficult issues were discussed, the client suddenly becomes very hostile. What is the most appropriate interpretation of this behavior by the nurse?

Hostility is being used as a defense because previous self-disclosure has raised anxiety.

A client who has been admitted to the psychiatric unit tells the nurse, "My problems are so bad that noone can help me." Which response is best for the nurse to make?

How can I help?

A client is receiving haloperidol (Haldol) for agitation, and the nurse is monitoring the client for side effects. Which response identified by the nurse is unrelated to an extrapyramidal tract effect?

Hypertensive crisis

27. A homeless person who is in the manic phase of bipolar disorder is admitted to the mental health unit. Which laboratory finding obtained on admission is most important for the nurse to report to the healthcare provider? A) Decreased thyroid stimulating hormone level. B) Elevated liver function profile. C) Increased white blood cell count. D) Decreased hematocrit and hemoglobin levels.

Hyperthyroidism causes an increased level of serum thyroid hormones (T3 and T4), which inhibit the release of TSH (A), so the client's manic behavior may be related to an endocrine disorder. (B, C, and D) are abnormal findings that are commonly found in the homeless population because of poor sanitation, poor nutrition, and the prevalence of substance abuse. Correct Answer(s): A

27. A homeless person who is in the manic phase of bipolar disorder is admitted to the mental health unit. Which laboratory finding obtained on admission is most important for the nurse to report to the healthcare provider? A) Decreased thyroid stimulating hormone level. B) Elevated liver function profile. C) Increased white blood cell count. D) Decreased hematocrit and hemoglobin levels.

Hyperthyroidism causes an increased level of serum thyroid hormones (T3 and T4), which inhibit the release of TSH (A), so the client's manic behavior may be related to an endocrine disorder. (B, C, and D) are abnormal findings that are commonly found in the homeless population because of poor sanitation, poor nutrition, and the prevalence of substance abuse. Correct Answer(s): A

A health care provider prescribes oxazepam (Serax) for a client who is beginning to experience withdrawal symptoms while undergoing detoxification. What are the primary reasons that oxazepam is given during detoxification?

Reduces the anxiety-tremor state and prevents more serious withdrawal symptoms

A health care provider refers a 52-year-old man to the mental health clinic. The history reveals that the man lost his wife to colon cancer 6 months ago and that since that time he has seen his health care provider seven times with the concern that he has colon cancer. All tests have had negative results. Recently the client stopped seeing friends, dropped his hobbies, and stayed home to rest. Which disorder should the nurse identify as consistent with the client's preoccupation with the fear of having a serious disease?

Hypochondriac disorder

The nurse is leading a "current events group" with chronic psychiatric clients. One group member states, "Saddam Hussein was my nurse during my last hospitalization. He was a very mean nurse and wasn't nice to me." Which response is best for the nurse to make?

I agree that Saddam Hussein is not a very nice man

A client who had knee surgery several months ago continues to request pain medication and has stopped going to work because of the pain. Assessment reveals that the operative site has healed without complications. How should the nurse respond?

I am concerned about your use of pain medication

A 72-year-old female client is admitted to the psychiatric unit with a medical diagnosis of major depression. Which statement by the client should concern the nurse and require further assessment?

I think my cat is going to die

A 38-year-old female client is admitted with a diagnosis of paranoid schizophrenia. When her tray is brought to her, she refuses to eat and tells the nurse, "I know you are trying to poison me with that food." Which response is most appropriate for the nurse to make?

I'll leave your tray here. I am available if you need anything else.

A client with schizophrenia sees a group of visitors sitting together talking. The client tells the nurse, "I know they're talking about me." Which altered thought process should the nurse identify?

Ideas of reference

70. A client on the psychiatric unit appears to imitate a certain nurse on the unit. The client seeks out this particular nurse and imitates her mannerisms. The nurse knows that the client is using which defense mechanism? A) Sublimation. B) Identification. C) Introjection. D) Repression.

Identification (B) is an attempt to be like someone or emulate the personality traits of another. (A) is substituting an unacceptable feeling for one that is more socially acceptable. (C) is incorporating the values or qualities of an admired person or group into one's own ego structure. (D) is the involuntary exclusion of painful thoughts or memories from one's awareness. Correct Answer(s): B

70. A client on the psychiatric unit appears to imitate a certain nurse on the unit. The client seeks out this particular nurse and imitates her mannerisms. The nurse knows that the client is using which defense mechanism? A) Sublimation. B) Identification. C) Introjection. D) Repression.

Identification (B) is an attempt to be like someone or emulate the personality traits of another. (A) is substituting an unacceptable feeling for one that is more socially acceptable. (C) is incorporating the values or qualities of an admired person or group into one's own ego structure. (D) is the involuntary exclusion of painful thoughts or memories from one's awareness. Correct Answer(s): B

A client recently admitted to the psychiatric unit is found to be experiencing command auditory hallucinations. The nurse conducts an initial one-on-one session centered on the development of trust. What is the next important nursing intervention?

Identifying the content of the messages in the auditory hallucinations

A college student visits the health center and describes anxiety about having to declare an academic major. What developmental conflict, according to Erikson, is this client still attempting to resolve?

Identity versus role confusion

The parents of a 14-year-old boy bring their son to the hospital. He is lethargic, but responsive. The mother states, "I think he took some of my pain pills." During initial assessment of the teenager, what information is most important for the nurse to obtain from the parents?

If he might have taken any other drugs

A 46-year-old female client has been on antipsychotic neuroleptics for the past three days. She has had a decrease in psychotic behavior and appears to be responding well to the medication. On the fourth day, the client's blood pressure increases, she becomes pale and febrile, and demonstrates muscular rigidity. Which action should the nurse initiate?

Immediately transfer the client to ICU

A client is admitted to the psychiatric unit wearing evening clothes. During the first 24 hours the client paces continually and laughs loudly. When approached by the nurse, the client refuses to cooperate with any requests, shouting, "I'm in charge. I give the orders!" How does the nurse interpret this behavior?

It attempts to ward off depression

The mother of an 18-year-old man comes to the local mental health center. She is extremely upset because of her son's behavior since the young man returned from his freshman year at college. He takes his brother's clothing, comes in at all hours, and refuses to get a job. Sometimes he is happy and outgoing, but at other times he is withdrawn. The mother asks why her son is like this. While contemplating this situation, the nurse considers that adolescents are usually:

Impulsive and self-centered

Failure to keep medical appointments, arriving late, and being non-compliant with prescribed medications are common behaviors exhibited by clients with personality disorders. Which intervention should be incorporated into the plan of care when a client demonstrates any of these behaviors?

Include the client in the decision-making process regarding treatments and appointments.

Failure to keep medical appointments, arriving late, and being non-compliant with prescribed medications are common behaviors exhibited by clients with personality disorders. Which intervention should be incorporated into the plan of care when a client demonstrates any of these behaviors? Designate a staff member to be responsible for imposing compliance with treatments and appointments. Include the client in the decision-making process regarding treatments and appointments. Designate a family member to help the client remain compliant with treatments and appointments. Restrict privileges until the client demonstrates compliance with treatments and appointments.

Include the client in the decision-making process regarding treatments and appointments.

The treatment team is reviewing the plan of care for a client diagnosed with borderline personality disorder. The client has been acting out during group meetings and creating conflict between staff members. Which interventions by the staff will be most effective to decrease the behavior of "splitting" the staff?

Increase frequency of staff meetings to discuss concerns and plan strategies.

The treatment team is reviewing the plan of care for a client diagnosed with borderline personality disorder. The client has been acting out during group meetings and creating conflict between staff members. Which interventions by the staff will be most effective to decrease the behavior of "splitting" the staff? Increase frequency of staff meetings to discuss concerns and plan strategies. Designate one staff member to work with the client. Plan a meeting with the client's family to discuss the client's disruptive behavior. Exclude the client from attending groups until the behavior improves.

Increase frequency of staff meetings to discuss concerns and plan strategies.

A nurse is educating a client who is taking clozapine (Clozaril) for paranoid schizophrenia. What should the nurse emphasize about the side effects of clozapine?

Increase in temperature

The nurse is assessing the parents of a nuclear family who are attending a support group for parents of adolescents. According to Erikson, these parents who are adapting to middle adulthood should exhibit which characteristic?

Increased self understanding

A female client in the mental health clinic has pressured speech and mumbles incoherently. What is the most appropriate nursing intervention?

Indicating to the client that she needs to slow down because what she says is important and cannot be understood

A nurse considers the cultural factors that may influence the development of eating disorders. The nurse recalls that eating disorders exist more frequently in:

Industrialized societies

The nurse is planning discharge for a male client with schizophrenia. The client insists that he is returning to his apartment, although the healthcare provider informed him that he will be moving to a boarding home. What is the most important nursing diagnosis for discharge planning?

Ineffective denial related to situational anxiety

64. Within several days of hospitalization, a client is repeatedly washing the top of the same table. Which initial intervention is best for the nurse to implement to help the client cope with anxiety related to this behavior? A) Administer a prescribed PRN antianxiety medication. B) Assist the client to identify stimuli that precipitates the ritualistic activity. C) Allow time for the ritualistic behavior, then redirect the client to other activities. D) Teach the client relaxation and thought stopping techniques.

Initially, the nurse should allow time for the ritual (C) to prevent anxiety. (A) may help reduce the client's anxiety, but will not prevent ritualistic behavior resulting from the client's ineffective coping ability. (B) is a long-term goal of individual therapy, but is not directly related to controlling the behavior at this time. (D) lists techniques that can be used to assist the client in learning new ways of interrupting obsessive thoughts and resulting ritualistic behavior as treatment progresses. Correct Answer(s): C

64. Within several days of hospitalization, a client is repeatedly washing the top of the same table. Which initial intervention is best for the nurse to implement to help the client cope with anxiety related to this behavior? A) Administer a prescribed PRN antianxiety medication. B) Assist the client to identify stimuli that precipitates the ritualistic activity. C) Allow time for the ritualistic behavior, then redirect the client to other activities. D) Teach the client relaxation and thought stopping techniques.

Initially, the nurse should allow time for the ritual (C) to prevent anxiety. (A) may help reduce the client's anxiety, but will not prevent ritualistic behavior resulting from the client's ineffective coping ability. (B) is a long-term goal of individual therapy, but is not directly related to controlling the behavior at this time. (D) lists techniques that can be used to assist the client in learning new ways of interrupting obsessive thoughts and resulting ritualistic behavior as treatment progresses. Correct Answer(s): C

When having a conversation with a nurse, an older client states, "I've lived a good life. I don't want to die, but I accept it as a part of life." What developmental stage, according to Erikson, has the client completed?

Integrity

A client with diabetes mellitus is able to discuss in detail the diabetic metabolic process while eating a piece of chocolate cake. What defense mechanism does the nurse identify when evaluating this behavior?

Intellectualization

The nurse is assessing a client's intelligence. Which factor should the nurse remember during this part of the mental status exam?

Intelligence is influenced by social and cultural beliefs.

To provide appropriate psychosocial support to clients, a nurse must understand development across the life span. What theory is the nurse using in considering relationships and resulting behaviors the central factors that influence development?

Interpersonal theory

An 85-year-old client is bought to the emergency department after a fall at home. The client appears confused, malnourished, and is severely dehydrated. The client appears reluctant to explain how the fall happened. The client's daughter speaks for the client and does not allow the client to answer questions. Based on this information, which nursing intervention is a priority?

Interview the client alone and assess for abuse.

An 85-year-old client is bought to the emergency department after a fall at home. The client appears confused, malnourished, and is severely dehydrated. The client appears reluctant to explain how the fall happened. The client's daughter speaks for the client and does not allow the client to answer questions. Based on this information, which nursing intervention is a priority? Interview the client alone and assess for abuse. Take the history from the daughter because the client is confused. Provide the daughter with nutritional counseling. Request a psychiatric evaluation for the client.

Interview the client alone and assess for abuse.

A nurse is assessing a young adult for evidence of achievement of the age-related developmental stage set forth in Erikson's developmental theory. What developmental crisis is associated with this age group?

Intimacy versus isolation

A client with a history of alcoholism is found to have Wernicke encephalopathy associated with Korsakoff syndrome. What does the nurse anticipate will be prescribed?

Intramuscular injections of thiamin

A client is admitted to the hospital with a history of increasingly bizarre behavior. The client says, "I'm wired to the TV, and it told me that my family is out to kill me." What is the best initial action by the admitting nurse?

Introducing the client to the primary nurse who will be assigned to work on a one-to-one basis with the client

A nurse notes that a male client sitting alone in the corner is smiling and talking to himself. Concluding that the client is hallucinating, the nurse should:

Invite the client to help decorate the dayroom.

After counseling an older widowed client, a nurse concludes that the grieving process has been successfully completed when the client:

Is able to plan to start new relationships

When planning interventions to help a client with bipolar I disorder, manic episode, meet rest and sleep needs, the nurse must remember that the manic client:

Is easily stimulated, and this interferes with sleep

The nurse is providing discharge education for a client prescribed a tricyclic antidepressant. Which statement by the client indicates the need for additional teaching? "It is best for me to take the full dose of the medication when I get up each morning." "My mood should improve within 7-28 days after starting my medicine." "It may take up to 6-8 weeks for me to experience the full effect of the medication." "The side effects of drowsiness and dizziness usually go away after the first few weeks."

It is best for me to take the full dose of the medication when I get up each morning." Rationale If possible, the client should take the full dose of tricyclic antidepressants at bedtime to reduce the experience of side effects during the day.

29. A male client is admitted to a mental health unit on Friday afternoon and is very upset on Sunday because he has not had the opportunity to talk with the healthcare provider. Which response is best for the nurse to provide this client? A) Let me call and leave a message for your healthcare provider. B) The healthcare provider should be here on Monday morning. C) How can I help answer your questions? D) What concerns do you have at this time?

It is best for the nurse to call the healthcare provider (A) because clients have the right to information about their treatment. Suggesting that the healthcare provider will be available the following day (B) does not provide immediate reassurance to the client. The nurse can also implement offer to assist the client (C and D), but the highest priority intervention is contacting the healthcare provider. Correct Answer(s): A

29. A male client is admitted to a mental health unit on Friday afternoon and is very upset on Sunday because he has not had the opportunity to talk with the healthcare provider. Which response is best for the nurse to provide this client? A) Let me call and leave a message for your healthcare provider. B) The healthcare provider should be here on Monday morning. C) How can I help answer your questions? D) What concerns do you have at this time?

It is best for the nurse to call the healthcare provider (A) because clients have the right to information about their treatment. Suggesting that the healthcare provider will be available the following day (B) does not provide immediate reassurance to the client. The nurse can also implement offer to assist the client (C and D), but the highest priority intervention is contacting the healthcare provider. Correct Answer(s): A

17. An 86-year-old female client with Alzheimer's disease is wandering the busy halls of the extended care facility and asks the nurse, "Where should I stand for the parade?" Which response is best for the nurse to provide? A) Anywhere you want to stand as long as you do not get hurt by those in the parade. B) You are confused because of all the activity in the hall. There is no parade. C) Let us go back to the activity room and see what is going on in there. D) Remember I told you that this is a nursing home and I am your nurse.

It is common for those with Alzheimer's disease to use the wrong words. Redirecting the client (using an accepting non-judgmental dialogue) to a safer place and familiar activities (C) is most helpful because clients experience short-term memory loss. (A) dismisses the client's attempt to find order and does not help her relate to her surroundings. (B) dismisses the client and may increase her anxiety level because it merely labels the client's behavior and offers no solution. It is very frustrating for those with Alzheimer's disease to "remember," and scolding them (D) may hurt their feelings. Correct Answer(s): C

17. An 86-year-old female client with Alzheimer's disease is wandering the busy halls of the extended care facility and asks the nurse, "Where should I stand for the parade?" Which response is best for the nurse to provide? A) Anywhere you want to stand as long as you do not get hurt by those in the parade. B) You are confused because of all the activity in the hall. There is no parade. C) Let us go back to the activity room and see what is going on in there. D) Remember I told you that this is a nursing home and I am your nurse.

It is common for those with Alzheimer's disease to use the wrong words. Redirecting the client (using an accepting non-judgmental dialogue) to a safer place and familiar activities (C) is most helpful because clients experience short-term memory loss. (A) dismisses the client's attempt to find order and does not help her relate to her surroundings. (B) dismisses the client and may increase her anxiety level because it merely labels the client's behavior and offers no solution. It is very frustrating for those with Alzheimer's disease to "remember," and scolding them (D) may hurt their feelings. Correct Answer(s): C

The mother of an adolescent client, admitted for substance abuse, tells the nurse that she feels like "giving up" on her child. How should the nurse respond?

It is hard to imagine what you are going through right now; it is good that you are able to express these feelings so we can work on them

10. The nurse observes a female client with schizophrenia watching the news on TV. She begins to laugh softly and says, "Yes, my love, I'll do it." When the nurse questions the client about her comment she states, "The news commentator is my lover and he speaks to me each evening. Only I can understand what he says." What is the best response for the nurse to make? A) What do you believe the news commentator said to you? B) Let's watch news on a different television channel. C) Does the news commentator have plans to harm you or others? D) The news commentator is not talking to you.

It is imperative that the nurse determine what the client believes she heard (A). The idea of reference may be to hurt herself or someone else, and the main function of a psychiatric nurse is to maintain safety. (B) is acceptable, but it is best to determine the client's beliefs. (C) is validating the idea of reference, while (D) is challenging the client. Correct Answer(s): A

10. The nurse observes a female client with schizophrenia watching the news on TV. She begins to laugh softly and says, "Yes, my love, I'll do it." When the nurse questions the client about her comment she states, "The news commentator is my lover and he speaks to me each evening. Only I can understand what he says." What is the best response for the nurse to make? A) What do you believe the news commentator said to you? B) Let's watch news on a different television channel. C) Does the news commentator have plans to harm you or others? D) The news commentator is not talking to you.

It is imperative that the nurse determine what the client believes she heard (A). The idea of reference may be to hurt herself or someone else, and the main function of a psychiatric nurse is to maintain safety. (B) is acceptable, but it is best to determine the client's beliefs. (C) is validating the idea of reference, while (D) is challenging the client. Correct Answer(s): A

67. A nurse working in the emergency room of a children's hospital admits a child whose injuries could have resulted from abuse. Which statement most accurately describes the nurse's responsibility in cases of suspected child abuse? A) The nurse should obtain objective data such as x-rays before reporting suspicions to the authorities. B) The nurse should confirm any suspicions of child abuse with the healthcare provider before reporting to the authorities. C) The nurse should report any case of suspected child abuse to the nurse in charge. D) The nurse should note in the client's record any suspicions of child abuse so that a history of such suspicions can be tracked.

It is the nurse's legal responsibility to report all suspected cases of child abuse. Notifying the charge nurse starts the legal reporting process (C). Correct Answer(s): C

67. A nurse working in the emergency room of a children's hospital admits a child whose injuries could have resulted from abuse. Which statement most accurately describes the nurse's responsibility in cases of suspected child abuse? A) The nurse should obtain objective data such as x-rays before reporting suspicions to the authorities. B) The nurse should confirm any suspicions of child abuse with the healthcare provider before reporting to the authorities. C) The nurse should report any case of suspected child abuse to the nurse in charge. D) The nurse should note in the client's record any suspicions of child abuse so that a history of such suspicions can be tracked.

It is the nurse's legal responsibility to report all suspected cases of child abuse. Notifying the charge nurse starts the legal reporting process (C). Correct Answer(s): C

A health care provider prescribed donepezil (Aricept), for a client newly diagnosed with early-stage dementia. The client asks the nurse, how the medication is going to slow down the process of memory impairment. Which explanation by the nurse is accurate regarding the purpose of this medication?

It prevents the breakdown of acetylcholine, which assists in transmission of nerve impulses.

A health care provider prescribed donepezil (Aricept), for a client newly diagnosed with early-stage dementia. The client asks the nurse, how the medication is going to slow down the process of memory impairment. Which explanation by the nurse is accurate regarding the purpose of this medication? It prevents the breakdown of acetylcholine, which assists in transmission of nerve impulses. It acts as a sleep aid to address sleep difficulties that may accelerate dementia. It decreases the circulation of calcium in the cardiovascular system, which is responsible for memory loss. It increases the amount of glucose available to the brain to maintain current brain function.

It prevents the breakdown of acetylcholine, which assists in transmission of nerve impulses.

A health care provider prescribed donepezil (Aricept), for a client newly diagnosed with early-stage dementia. The client asks the nurse, how the medication is going to slow down the process of memory impairment. Which explanation by the nurse is accurate regarding the purpose of this medication? It prevents the breakdown of acetylcholine, which assists in transmission of nerve impulses. It acts as a sleep aid to address sleep difficulties that may accelerate dementia. It decreases the circulation of calcium in the cardiovascular system, which is responsible for memory loss. It increases the amount of glucose available to the brain to maintain current brain function.

It prevents the breakdown of acetylcholine, which assists in transmission of nerve impulses.

A couple arrives at the mental health clinic for counseling because the husband consistently believes that his wife is having multiple affairs. After several sessions a delusional disorder is diagnosed. What specific subtype of the delusion does the nurse identify?

Jealousy

The treatment team discusses plans regarding two clients who both exhibit self-mutilating behavior. The nurse expresses concern about the decision to use restraints on one client, while assigning one-to-one supervision for the other. Which ethical principal is the nurse concerned about violating?

Justice.

The treatment team discusses plans regarding two clients who both exhibit self-mutilating behavior. The nurse expresses concerns about the decision to use restraints on one client, while assigning one-to-one supervision for the other. Which ethical principal is the nurse concerned about violating? Justice. Autonomy. Beneficence. Fidelity.

Justice.

When preparing a teaching plan for a client who is to be discharged with a prescription for lithium carbonate (Lithonate), it is most important for the nurse to include which instruction?

Keep your dietary salt intake consistent

A client has been in the alcohol detoxification unit for 5 days. In the evening the client complains of numbness and tingling in the feet and legs. What is the most appropriate nursing intervention?

Keeping the bed linens off the client's legs with a mechanical aid.

6. The parents of a 14-year-old boy bring their son to the hospital. He is lethargic, but responsive. The mother states, "I think he took some of my pain pills." During initial assessment of the teenager, what information is most important for the nurse to obtain from the parents? A) If he has seemed depressed recently. B) If a drug overdose has ever occurred before. C) If he might have taken any other drugs. D) If he has a desire to quit taking drugs.

Knowledge of all substances taken (C) will guide further treatment, such as administration of antagonists, so obtaining this information has the highest priority. (A and B) are also valuable in planning treatment. (D) is not appropriate during the acute management of a drug overdose. Correct Answer(s): C

6. The parents of a 14-year-old boy bring their son to the hospital. He is lethargic, but responsive. The mother states, "I think he took some of my pain pills." During initial assessment of the teenager, what information is most important for the nurse to obtain from the parents? A) If he has seemed depressed recently. B) If a drug overdose has ever occurred before. C) If he might have taken any other drugs. D) If he has a desire to quit taking drugs.

Knowledge of all substances taken (C) will guide further treatment, such as administration of antagonists, so obtaining this information has the highest priority. (A and B) are also valuable in planning treatment. (D) is not appropriate during the acute management of a drug overdose. Correct Answer(s): C

When thinking of Lithium remember LITH:

L - Levels over 1.5 MEQ/L = TOXIC!! I - Increase fluid and sodium (NA+) - Lithium lets go of fluid so do not limit sodium or water intake. T - toxic signs H - HOLD NSAIDS

A 16-year-old high school student who has anorexia nervosa tells the school nurse that she thinks she that is pregnant even though she has had intercourse only once, more than a year ago. What is the most appropriate inference for the nurse to make about the student?

Lacking knowledge that anorexia can cause amenorrhea

How can the nurse best minimize psychological stress in an anxious client who has been admitted to the psychiatric unit?

Learn what is of particular importance to the client.

A male client is admitted to a mental health unit on Friday afternoon and is very upset on Sunday because he has not had the opportunity to talk with the healthcare provider. Which response is best for the nurse to provide this client?

Let me call and leave a message for your healthcare provider

An 86-year-old female client with Alzheimer's disease is wandering the busy halls of the extended care facility and asks the nurse, "Where should I stand for the parade?" Which response is best for the nurse to provide?

Let us go back to the activity room and see what is going on in there

50. A client who is known to abuse drugs is admitted to the psychiatric unit. Which medication should the nurse anticipate administering to a client who is exhibiting benzodiazepine withdrawal symptoms? A) Perphenazine (Trilafon). B) Diphenhydramine (Benadryl). C) Chlordiazepoxide (Librium). D) Isocarboxazid (Marplan).

Librium (C), an antianxiety drug, as well as other benzodiazepines, are used in titrated doses to reduce the severity of abrupt benzodiazepine withdrawal. (A) is an antipsychotic agent. (B) is an antihistamine and antianxiety drug. (D) is an MAO inhibitor. Correct Answer(s): C

50. A client who is known to abuse drugs is admitted to the psychiatric unit. Which medication should the nurse anticipate administering to a client who is exhibiting benzodiazepine withdrawal symptoms? A) Perphenazine (Trilafon). B) Diphenhydramine (Benadryl). C) Chlordiazepoxide (Librium). D) Isocarboxazid (Marplan).

Librium (C), an antianxiety drug, as well as other benzodiazepines, are used in titrated doses to reduce the severity of abrupt benzodiazepine withdrawal. (A) is an antipsychotic agent. (B) is an antihistamine and antianxiety drug. (D) is an MAO inhibitor. Correct Answer(s): C

An effective mood-stabilizing drug used in clients with bipolar disorder in the acute treatment of mania and prevention of recurrent mania and depressive episodes is:

Lithium carbonate (Lithium)

56. When preparing a teaching plan for a client who is to be discharged with a prescription for lithium carbonate (Lithonate), it is most important for the nurse to include which instruction? A) It may take 3 to 4 weeks to achieve therapeutic effects. B) Keep your dietary salt intake consistent. C) Avoid eating aged cheese and chicken liver. D) Eat foods high in fiber such as whole grain breads.

Lithium's effectiveness is influenced by salt intake (B). Too much salt causes more lithium to be excreted, thereby decreasing the effectiveness of the drug. Too little salt causes less lithium to be excreted, potentially resulting in toxicity. (A, C, and D) are not specific instructions pertinent to teaching about lithium carbonate (Lithonate). Correct Answer(s): B

A nurse becomes aware of an older client's feeling of loneliness when the client states, "I only have a few friends. My daughter lives in another state and couldn't care less whether I live or die. She doesn't even know I'm in the hospital." The nurse identifies the client's communication as a:

Reflection of depression that is causing feelings of hopelessness

What assessment of a group member does the nurse use to identify the emotional-informal leader of the group?

Reflects the feeling tone of the group

56. When preparing a teaching plan for a client who is to be discharged with a prescription for lithium carbonate (Lithonate), it is most important for the nurse to include which instruction? A) It may take 3 to 4 weeks to achieve therapeutic effects. B) Keep your dietary salt intake consistent. C) Avoid eating aged cheese and chicken liver. D) Eat foods high in fiber such as whole grain breads.

Lithium's effectiveness is influenced by salt intake (B). Too much salt causes more lithium to be excreted, thereby decreasing the effectiveness of the drug. Too little salt causes less lithium to be excreted, potentially resulting in toxicity. (A, C, and D) are not specific instructions pertinent to teaching about lithium carbonate (Lithonate). Correct Answer(s): B

A nurse is assessing an adolescent client with the diagnosis of schizophrenia, undifferentiated type. Which signs and symptoms should the nurse expect the client to experience?

Loosened associations and hallucinations

Things to remember about MAOIs

M - massive hypertension crisis risk. (Massive headache) A - avoid tyramine (wine and cheese, beer and sausage, salami, chocolate. REMEMBER - tyramINE - phenelzINE, selegilINE O - over the counter drugs = hypertension crisis risk. (Calcium, antacids, acetaminophen, NSAIDS - Naproxen and ibuprofen) O - other antidepressant - SSRI, SNRI, TCA. Can cause serotonin syndrome. NO escitalopram (SSRI) and NO imipramine (TCA) I - increased suicide risk - usually in first few weeks of treatments or when it is increased.

What class of meds puts a patient at risk for massive hypertension crisis?

MAOI's - biggest sign and symptom is massive headache

A client begins taking an atypical antipsychotic medication. The nurse must provide informed consent and education about common medication side effects. Which client education will be most important? A. Maintain a balanced diet and adequate exercise. B. Be sure that the diet is adequate in salt intake. C. Monitor for any changes in sleep pattern. D. Report any unusual facial movements.

Maintain a balanced diet and adequate exercise. Several atypical antipsychotic medications can cause significant weight gain, so the client should be advised to maintain a balanced diet and adequate exercise. Option B is important with lithium, a mood stabilizer. Options C and D are less common than weight gain.

The charge nurse is collaborating with the nursing staff about the plan of care for a client who is very depressed. What is the most important intervention to implement during the first 48 hours after the client's admission to the unit?

Maintain safety in the client's milieu.

A nurse is admitting a client to the unit. What interaction demonstrates effective therapeutic communication principles?

Maintaining a distance of at least 3 feet from the client

The family and friends of a client with a heroin addiction are planning an intervention meeting to convince the client to seek help. Which strategy should the group employ to help ensure a successful intervention?

Make notes on what to say to the client and rehearse before the meeting.

The family and friends of a client with a heroin addiction are planning an intervention meeting to convince the client to seek help. Which strategy should the group employ to help ensure a successful intervention? Make notes on what to say to the client and rehearse before the meeting. Attempt the intervention at a time when the client is under the influence. Stage the intervention in a public place that is familiar to the client. Set boundaries and be prepared to act in case the client behaves defensively.

Make notes on what to say to the client and rehearse before the meeting.

A nurse administers prescribed anxiolytics to clients with severe emotional disorders. What is the goal of this treatment?

Makes the client more amenable to psychotherapy

A client has been in an acute care psychiatric unit for 3 days and is receiving haloperidol (Haldol) tablets orally to reduce agitation and preoccupation with auditory hallucinations. There has been no decrease in the client's agitation or preoccupation with auditory hallucinations since the medication was started. What should the nurse's priority intervention be?

Making certain that the client is swallowing the medication

A nurse determines that a client is pretending to be ill. What does this behavior usually indicate?

Malingering

A health care provider prescribes divalproex (Depakote). What does the nurse consider an appropriate indication for the use of this drug?

Management of manic episodes of bipolar disorder

A client with dementia has been cared for by the spouse for 5 years. During the last month the client has become agitated and aggressive and is incontinent of urine and feces. What is the priority nursing care while this client is in an inpatient mental health facility?

Managing the behavior

A young mother of three children, all born 1 year apart, has been hospitalized after trying to hang herself. The client is being treated with milieu therapy. The nurse understands that this therapeutic modality consists of:

Manipulating the environment to benefit the client

A client with a history of chronic alcoholism is admitted to the mental health unit. What does the nurse identify as the cause of a client's use of confabulation?

Marked loss of memory

When caring for clients who are demonstrating manic behavior, the nurse must constantly reassess these clients' physical needs. What characteristic about these clients makes this particularly important?

May become exhausted from excessive activity

The community health nurse talks to a male client who has bipolar disorder. The client explains that he sleeps 4 to 5 hours a night and is working with his partner to start two new businesses and build an empire. The client stopped taking his medications several days ago. What nursing problem has the highest priority?

Medication management

A 55-year-old man who has a long history of drug and alcohol abuse tells the nurse during an interview that he is taking ginkgo biloba. The nurse knows that ginkgo biloba is taken to treat:

Memory impairment

Which assessment data supports a diagnosis of acrophobia?

Refuses to drive or walk across any type of bridge.

33. Which statement about contemporary mental health nursing practice is accurate? A) There is one approved theoretical framework for psychiatric nursing practice. B) Psychiatric nursing has yet to be recognized as a core mental health discipline. C) Contemporary practice of psychiatric nursing is primarily focused on inpatient care. D) The psychiatric nursing client may be an individual, family, group, organization, or community.

Mental health nursing is not only concerned with one-on-one interactions. Psychiatric stressors can impact and be reflected in the overall direction, activities, and responses involving families, groups, and entire communities (D). (A, B, and C) are incorrect statements about the status of mental health nursing. Correct Answer(s): D

33. Which statement about contemporary mental health nursing practice is accurate? A) There is one approved theoretical framework for psychiatric nursing practice. B) Psychiatric nursing has yet to be recognized as a core mental health discipline. C) Contemporary practice of psychiatric nursing is primarily focused on inpatient care. D) The psychiatric nursing client may be an individual, family, group, organization, or community.

Mental health nursing is not only concerned with one-on-one interactions. Psychiatric stressors can impact and be reflected in the overall direction, activities, and responses involving families, groups, and entire communities (D). (A, B, and C) are incorrect statements about the status of mental health nursing. Correct Answer(s): D

What are some of the family risk factors the nurse should look for when interviewing a client who is suspected of being in an abusive relationship? Mental health problems in the nuclear family.. Substance abuse by household members. Family relationships that appear dysfunctional. The ethnic and cultural background of the family. Educational background of client and family members.

Mental health problems in the nuclear family.. Substance abuse by household members. Family relationships that appear dysfunctional.

The spouse of a heroin addict asks the nurse, "What is methadone maintenance all about?" Which response by the nurse is correct?

Methadone reduces cravings and blocks the action of opiates.

The spouse of a heroin addict asks the nurse, "What is methadone maintenance all about?" Which response by the nurse is correct? Methadone reduces cravings and blocks the action of opiates. Methadone reduces craving without any sedative effects. Methadone is less potent than other replacement drugs. Methadone is a deterrent to the use of both "hard" and "soft" drugs.

Methadone reduces cravings and blocks the action of opiates.

38. The nurse is assessing the parents of a nuclear family who are attending a support group for parents of adolescents. According to Erikson, these parents who are adapting to middle adulthood should exhibit which characteristic? A) Loss of independence. B) Increased self-understanding. C) Isolation from society. D) Development of intimate relationships.

Middle adulthood is characterized by self-reflection, understanding, and acceptance (B), and generativity or guidance of children. (A and C) are maladaptive behaviors in middle adulthood. Although middle-aged adults may delay or re-establish intimate relationships, (D) is initially developed during young adulthood. Correct Answer(s): B

38. The nurse is assessing the parents of a nuclear family who are attending a support group for parents of adolescents. According to Erikson, these parents who are adapting to middle adulthood should exhibit which characteristic? A) Loss of independence. B) Increased self-understanding. C) Isolation from society. D) Development of intimate relationships.

Middle adulthood is characterized by self-reflection, understanding, and acceptance (B), and generativity or guidance of children. (A and C) are maladaptive behaviors in middle adulthood. Although middle-aged adults may delay or re-establish intimate relationships, (D) is initially developed during young adulthood. Correct Answer(s): B

An obviously upset client comes to the mental health clinic and, after pushing ahead of the other clients, states, "I had an argument with my daughter, and now I'm tense, and worried, and angry." What level of anxiety does the nurse determine that the client is experiencing?

Moderate

A nurse working on a mental health unit receives a community call from a person who is tearful and states, "I just feel so nervous all of the time. I don't know what to do about my problems. I haven't been able to sleep at night and have hardly eaten for the past 3 or 4 days." The nurse should initiate a referral based on which assessment?

Moderate levels of anxiety

A 42-year-old adult with a long history of alcohol abuse seeks help in one of the local hospitals. The nurse considers that the major underlying factor for success in an alcohol treatment program is the client's:

Motivation

A nurse is caring for a client who is delusional and talking about people who are plotting to do harm. The staff members notice that the client is pacing more than usual, and the primary nurse concludes that the client is beginning to lose control. What is the most therapeutic nursing intervention?

Moving the client to a quiet place

The nurse is interviewing a newly admitted client with major depression. Which statement by the client is evidence of a genetic etiology of this mood disorder?

My mother was diagnosed with bipolar affective disorder.

The nurse is interviewing a newly admitted client with major depression. Which statement by the client is evidence of a genetic etiology of this mood disorder? My mother was diagnosed with bipolar affective disorder. My uncle was diagnosed with schizophrenia. My cousin was hospitalized with suicidal ideation last year. My niece is being evaluated for borderline personality disorder.

My mother was diagnosed with bipolar affective disorder.

The nurse is preparing to administer phenelzine sulfate (Nardil) to a client on the psychiatric unit.Which complaint related to administration of this drug should the nurse expect this client to make?

My mouth feels like cotton

Can a patient take Selegiline (MAOI) with Sertraline (SSRI)?

NO - never mix SSRI and MAOI

A patient has diarrhea and is vomiting should you give lithium?

NO - pt is probably already very dehydrated and lithium can cause dehydration

A client is brought by ambulance to the emergency department. The client's signs and symptoms are indicative of opioid overdose. What should the nurse expect the practitioner to prescribe?

Naloxone

What is the antidote for opiods?

Naloxone (Narcan)

The nurse is caring for a client diagnosed with a personality disorder. The client states that he must constantly fend off females who seek his attention. He boasts that he is a former high school baseball player who was once voted most valuable player, and he claims that no other baseball player will ever be as good as he was in baseball and he should play professionally, but his salary would be too high for the professioal leagues to pay him. These statements reflect which personality disorder? Narcissistic personality disorder. Obsessive-compulsive disorder. Histrionic personality disorder. Borderline personality disorder.

Narcissistic personality disorder.

The nurse is caring for a client diagnosed with a personality disorder. The client states that he must constantly fend off females who seek his attention. He boasts that he is a former high school baseball player who was once voted most valuable player, and he claims that no other baseball player will ever be as good as he was in baseball and he should play professionally, but his salary would be too high for the professional leagues to pay him. These statements reflect which personality disorder?

Narcissistic personality disorder.

A nurse assesses a client recently admitted to an alcohol detoxification unit. What common clinical manifestation should the nurse expect during the initial stage of alcohol detoxification?

Nausea

The nurse is scheduled to be the co-leader of a therapy group being formed in the mental health clinic. When planning for the first meeting, it is of primary importance that the nurse considers the:

Needs of the clients being included

53. A client is admitted with a diagnosis of depression. The nurse knows that which characteristic is most indicative of depression? A) Grandiose ideation. B) Self-destructive thoughts. C) Suspiciousness of others. D) A negative view of self and the future.

Negative self-image and feelings of hopelessness about the future (D) are specific indicators for depression. (A and/or C) occurs with paranoia or paranoid ideation. (B) may be seen in depressed clients, but are not always present, so (D) is a better answer than (B). Correct Answer(s): D

53. A client is admitted with a diagnosis of depression. The nurse knows that which characteristic is most indicative of depression? A) Grandiose ideation. B) Self-destructive thoughts. C) Suspiciousness of others. D) A negative view of self and the future.

Negative self-image and feelings of hopelessness about the future (D) are specific indicators for depression. (A and/or C) occurs with paranoia or paranoid ideation. (B) may be seen in depressed clients, but are not always present, so (D) is a better answer than (B). Correct Answer(s): D

An adolescent client was recently admitted to the eating disorder unit and is complaining of severe muscle aches, fatigue, and weakness. The nurse completes a physical assessment and reviews the client's most recent lab results. All lab values are within normal limits except the potassium level, which is 2.7 meq/L. Based on this assessment, which nursing intervention is the priority?

Notify the health care provider that the client is hypokalemic and complaining of muscle aches.

Which assessment data supports a diagnosis of acrophobia? Refuses to drive or walk across any type of bridge. Avoids leaving the house for more than an hour. Refuses to go outside to watch baseball even though he loves the sport. Avoids basements due to a fear of encountering spiders.

Refuses to drive or walk across any type of bridge.

An adolescent client was recently admitted to the eating disorder unit and is complaining of severe muscle aches, fatigue, and weakness. The nurse completes a physical assessment and reviews the client's most recent lab results. All lab values are within normal limits except the potassium level, which is 2.7 meq/L. Based on this assessment, which nursing intervention is the priority? Notify the health care provider that the client is hypokalemic and complaining of muscle aches. Medicate the client with cyclobenzaprine 5mg every 8 hours PRN for muscle aches. Encourage the client to take a hot shower to relieve aches and pains. Assess the client's pain and administer pain medication as ordered.

Notify the health care provider that the client is hypokalemic and complaining of muscle aches.

A client who is being treated with lithium carbonate for bipolar disorder develops diarrhea, vomiting, and drowsiness. What action should the nurse take?

Notify the healthcare provider of the symptoms prior to the next administration of the drug.

A woman who was sexually assaulted by a stranger in the elevator of her apartment building is brought by her husband to the emergency department. What is the priority nursing intervention?

Obtaining information about her perception of the incident

A nurse is teaching clients about dietary restrictions during monoamine oxidase inhibitor (MAOI) therapy. What response does the nurse tell them to anticipate if they do not follow these restrictions?

Occipital headaches

72. A client who has been admitted to the psychiatric unit tells the nurse, "My problems are so bad that no one can help me." Which response is best for the nurse to make? A) How can I help? B) Things probably aren't as bad as they seem right now. C) Let's talk about what is right with your life. D) I hear how miserable you are, but things will get better soon.

Offering self shows empathy and caring (A), and is the best of the choices provided. Combining the first part of (D) with (A) would be the best response, but this is not a fill-in-the-blank or an essay test! Choose the best of those choices provided and move on. (B) dismisses the client, things are bad as far as this client is concerned. (C) avoids the client's problems and promotes denial. "I hear how miserable you are" is an example of reflective dialogue and would be the best choice if it were not for the rest of the sentence--"but things will get better" which is offering false reassurance. Correct Answer(s): A

72. A client who has been admitted to the psychiatric unit tells the nurse, "My problems are so bad that no one can help me." Which response is best for the nurse to make? A) How can I help? B) Things probably aren't as bad as they seem right now. C) Let's talk about what is right with your life. D) I hear how miserable you are, but things will get better soon.

Offering self shows empathy and caring (A), and is the best of the choices provided. Combining the first part of (D) with (A) would be the best response, but this is not a fill-in-the-blank or an essay test! Choose the best of those choices provided and move on. (B) dismisses the client, things are bad as far as this client is concerned. (C) avoids the client's problems and promotes denial. "I hear how miserable you are" is an example of reflective dialogue and would be the best choice if it were not for the rest of the sentence--"but things will get better" which is offering false reassurance. Correct Answer(s): A

The health care provider is evaluating a client with bulimia who is being treated with fluoxetine (Prozac), a selective serotonin reuptake inhibitor. It is determined that medication has been ineffective because the client's mood and obsessive compulsive behaviors have not improved. Which unconventional antipsychotic agent should the health care provider recommend for the client?

Olanzapine (Zyprexa).

A nurse is assessing a client with the diagnosis of schizophrenia, undifferentiated type. What defense mechanisms should the nurse anticipate that this client might use?

Regression

The health care provider is evaluating a client with bulimia who is being treated with fluoxetine (Prozac), a selective serotonin reuptake inhibitor. It is determined that medication has been ineffective because the client's mood and obsessive compulsive behaviors have not improved. Which unconventional antipsychotic agent should the health care provider recommend for the client? Olanzapine (Zyprexa). Sertraline (Zoloft). Lorazepam (Ativan). Zolpidem (Ambien).

Olanzapine (Zyprexa). Rationale Olanzapine (Zyprexa) is an unconventional antipsychotic agent used to treat some mental disorders. It has proven successful in some clients by decreasing compulsions and improving mood.

A nurse working on a mental health unit is caring for several clients who are at risk for suicide. Which client is at the greatest risk for successful suicide?

Older single man just found to have pancreatic cancer

13. Which diet selection by a client who is depressed and taking the MAO inhibitor tranylcypromine sulfate (Parnate) indicates to the nurse that the client understands the dietary restrictions imposed by this medication regimen? A) Hamburger, French fries, and chocolate milkshake. B) Liver and onions, broccoli, and decaffeinated coffee. C) Pepperoni and cheese pizza, tossed salad, and a soft drink. D) Roast beef, baked potato with butter, and iced tea.

Only (D) contains no tyramine. Tyramine in foods interacts with MAOI in the body causing a hypertensive crisis which is life-threatening, and Parnate is classified as an MAOI antidepressant. Some items in (A, B, and C) contain tyramine and would not be permitted for a client taking Parnate. Correct Answer(s): D

13. Which diet selection by a client who is depressed and taking the MAO inhibitor tranylcypromine sulfate (Parnate) indicates to the nurse that the client understands the dietary restrictions imposed by this medication regimen? A) Hamburger, French fries, and chocolate milkshake. B) Liver and onions, broccoli, and decaffeinated coffee. C) Pepperoni and cheese pizza, tossed salad, and a soft drink. D) Roast beef, baked potato with butter, and iced tea.

Only (D) contains no tyramine. Tyramine in foods interacts with MAOI in the body causing a hypertensive crisis which is life-threatening, and Parnate is classified as an MAOI antidepressant. Some items in (A, B, and C) contain tyramine and would not be permitted for a client taking Parnate. Correct Answer(s): D

The nurse is conducting discharge teaching for a client with schizophrenia who plans to live in a grouphome. Which statement is most indicative of the need for careful follow-up after discharge?

Only my belief in God can help me.

The nurse should first discuss terminating the nurse-client relationship with a client during the:

Orientation phase, when a contract is established

A 45-year-old male client tells the nurse that he used to believe that he was Jesus Christ, but now heknows he is not. Which response is best for the nurse to make?

Others have had similar thoughts when under stress

A client with recurrent episodes of depression comes to the mental health clinic for a routine follow-up visit. The nurse suspects that the client is at increased risk for suicide. What is a contributing factor to the client's risk for suicide?

Overwhelming feelings of guilt

A client with an organic mental disorder becomes increasingly agitated and abusive. The practitioner prescribes haloperidol (Haldol). For what untoward effects should the nurse assess the client?

Parkinsonism and agranulocytosis

A client is undergoing treatment for schizophrenia. Which outcome provides evidence that the client's negative symptoms are improving?

Participates in music therapy and states that he enjoys playing the drums..

A client is undergoing treatment for schizophrenia. Which outcome provides evidence that the client's negative symptoms are improving? Refrains from yelling and trying to touch the health care provider for the last 48 hours. Participates in music therapy and states that he enjoys playing the drums. Eats meals in his room instead of causing disturbances during mealtime. Reports having no hallucinations for the last week.

Participates in music therapy and states that he enjoys playing the drums..

The nurse is caring for a victim of severe emotional violence inflicted by her husband. The client states that the abuse occurs most often when her husband is intoxicated, and that he is always remorseful afterwards. She also tells the nurse that her husband's father was an alcoholic who beat him and his mother. What evidence exists that the husband is at risk of becoming a perpetrator of physical abuse?

Past childhood abuse.

The nurse is caring for a victim of severe emotional violence inflicted by her husband. The client states that the abuse occurs most often when her husband is intoxicated, and that he is always remorseful afterwards. She also tells the nurse that her husband's father was an alcoholic who beat him and his mother. What evidence exists that the husband is at risk of becoming a perpetrator of physical abuse? Past childhood abuse. Feelings of remorse. Temporary behavioral changes. Excessive alcohol consumption.

Past childhood abuse.

The nurse is providing care to a client suspected of Munchausen. The nurse should understand that which example demonstrates a breach of a client's constitutional right to privacy?

Releasing information to the client's employer without consent.

42. On admission to a residential care facility, an elderly female client tells the nurse that she enjoys cooking, quilting, and watching television. Twenty-fours after admission, the nurse notes that the client is withdrawn and isolated. It is best for the nurse to encourage this client to become involved in which activity? A) Clean the unit kitchen cabinets. B) Participate in a group quilting project. C) Watch television in the activity room. D) Bake a cake for a resident's birthday.

Peer interaction in a group activity (B) will help to prevent social isolation and withdrawal. (A, C, and D) are activities that can be accomplished alone, without peer interaction. Correct Answer(s): B

42. On admission to a residential care facility, an elderly female client tells the nurse that she enjoys cooking, quilting, and watching television. Twenty-fours after admission, the nurse notes that the client is withdrawn and isolated. It is best for the nurse to encourage this client to become involved in which activity? A) Clean the unit kitchen cabinets. B) Participate in a group quilting project. C) Watch television in the activity room. D) Bake a cake for a resident's birthday.

Peer interaction in a group activity (B) will help to prevent social isolation and withdrawal. (A, C, and D) are activities that can be accomplished alone, without peer interaction. Correct Answer(s): B

During the intake conversation of a client with schizophrenia the nurse observes that the client constantly looks at a spot on the wall. What is the best statement for the nurse to make at this time?

People with schizophrenia sometimes see things that are not real. Is that what you are experiencing right now?

The nurse identifies that a client is expressing feelings of self-effacement. Which client assessment supports this conclusion?

Perception that no one is listening

As the nurse considers a client's placement on the continuum of anxiety, a key in determining the degree of anxiety being experienced is the client's:

Perceptual field

A client with stage 2 Alzheimer's disease is being cared for at home by the spouse. The client's spouse tells the nurse about the emotional difficulties involved in providing fulltime care at home. Which self-care activity is most important for the nurse to recommend to the spouse?

Periodic times of respite from caregiving.

A client with stage 2 Alzheimer's disease is being cared for at home by the spouse. The client's spouse tells the nurse about the emotional difficulties involved in providing fulltime care at home. Which self-care activity is most important for the nurse to recommend to the spouse? Periodic times of respite from caregiving. Regular attendance at church services. Participation in reminiscence therapy. Establishment of a predictable daily schedule.

Periodic times of respite from caregiving. Rationale Caregiver role strain may be attributed to many different factors. The nurse must become familiar with this diagnosis in order to accurately assess the caregiver and offer effective interventions. One important recommendation is to have the caregiver incorporate periodic breaks as part of the daily routine to relieve stress. The nurse should contact the client's manager and provide the client's caregiver a list of agencies offering "Respite Care".

The nurse should include which interventions in the plan of care for a severely depressed client with neurovegetative symptoms?

Permit rest periods as needed Speaking slowly and simply Allow the client extra time to complete tasks Observe and encourage food and fluid intake Encourage mild exercise and shorts walks on the unit

46. A child is brought to the emergency room with a broken arm. Because of other injuries, the nurse suspects the child may be a victim of abuse. When the nurse tries to give the child an injection, the child's mother becomes very loud and shouts, "I won't leave my son! Don't you touch him! You'll hurt my child!" What is the best interpretation of the mother's statements? The mother is A) regressing to an earlier behavior pattern. B) sublimating her anger. C) projecting her feelings onto the nurse. D) suppressing her fear.

Projection is attributing one's own thoughts, impulses, or behaviors onto another--it is the mother who is probably harming the child and she is attributing her actions to the nurse (C). The mother may be immature, but (A) is not the best description of her behavior. (B) is substituting a socially acceptable feeling for an unacceptable one. These are not socially acceptable feelings. The mother may be suppressing her fear (D) by displaying anger, but such an interpretation cannot be concluded from the data presented. Correct Answer(s): C

1. A male client with schizophrenia who is taking fluphenazine decanoate (Prolixin decanoate) is being discharged in the morning. A repeat dose of medication is scheduled for 20 days after discharge. The client tells the nurse that he is going on vacation in the Bahamas and will return in 18 days. Which statement by the client indicates a need for health teaching? A) When I return from my tropical island vacation, I will go to the clinic to get my Prolixin injection. B) While I am on vacation and when I return, I will not eat or drink anything that contains alcohol. C) I will notify the healthcare provider if I have a sore throat or flu-like symptoms. D) I will continue to take my benztropine mesylate (Cogentin) every day.

Photosensitivity is a side effect of Prolixin and a vacation in the Bahamas (with its tropical island climate) increases the client's chance of experiencing this side effect. He should be instructed to avoid direct sun (A) and wear sunscreen. (B, C, and D) indicate accurate knowledge. Alcohol acts synergistically with Prolixin (B). (C) lists signs of agranulocytosis, which is also a side effect of Prolixin. In order to avoid extrapyramidal symptoms (EPS), anticholinergic drugs, such as Cogentin, are often prescribed prophylactically with Prolixin. Correct Answer(s): A

1. A male client with schizophrenia who is taking fluphenazine decanoate (Prolixin decanoate) is being discharged in the morning. A repeat dose of medication is scheduled for 20 days after discharge. The client tells the nurse that he is going on vacation in the Bahamas and will return in 18 days. Which statement by the client indicates a need for health teaching? A) When I return from my tropical island vacation, I will go to the clinic to get my Prolixin injection. B) While I am on vacation and when I return, I will not eat or drink anything that contains alcohol. C) I will notify the healthcare provider if I have a sore throat or flu-like symptoms. D) I will continue to take my benztropine mesylate (Cogentin) every day.

Photosensitivity is a side effect of Prolixin and a vacation in the Bahamas (with its tropical island climate) increases the client's chance of experiencing this side effect. He should be instructed to avoid direct sun (A) and wear sunscreen. (B, C, and D) indicate accurate knowledge. Alcohol acts synergistically with Prolixin (B). (C) lists signs of agranulocytosis, which is also a side effect of Prolixin. In order to avoid extrapyramidal symptoms (EPS), anticholinergic drugs, such as Cogentin, are often prescribed prophylactically with Prolixin. Correct Answer(s): A

A practitioner prescribes haloperidol (Haldol) 10 mg by mouth twice a day for a client who is also receiving phenytoin (Dilantin) for control of epilepsy. When planning the client's care, the nurse considers that anticonvulsants may interact with haloperidol to:

Potentiate its central nervous system depressant effect.

What does a public health nurse expect to encounter when working with families raised in a culture of poverty?

Powerlessness relative to changing their situation

A young client who has become a mother for the first time is showing signs of being anxious about her new parenting role. With the nurse's encouragement, she joins the new mothers' support group at the local YMCA. What kind of prevention does this activity reflect?

Primary prevention

A client who is taking lithium arrives at the mental health center for a routine visit. The client has slurred speech, has an ataxic gait, and complains of nausea. The nurse knows that these signs and symptoms are:

Probably associated with a toxic level of lithium

A child is brought to the emergency room with a broken arm. Because of other injuries, the nurse suspects the child may be a victim of abuse. When the nurse tries to give the child an injection, the child's mother becomes very loud and shouts, "I won't leave my son! Don't you touch him! You'll hurt my child!" What is the best interpretation of the mother's statements? The mother is

Projecting her feelings onto the nurse

A female adolescent in group therapy tells the other group members that while out on a pass she used marijuana because her boyfriend made her smoke it. What defense mechanism is the client using?

Projection

The nurse understands that paranoid delusions may be related to the defense mechanism of:

Projection

A nurse anticipates that most clients with phobias will use the defense mechanisms of:

Projection and displacement

A client is admitted to the hospital with a diagnosis of depression. What clinical manifestations of depression does the nurse expect when assessing this client?

Psychomotor retardation

On admission, a highly anxious client is described as delusional. The nurse understands that delusions are most likely to occur with which class of disorder?

Psychotic

The nurse is providing information about codependent behaviors to the wife of a client with alcoholism. The nurse is correct to identify and point out to the wife which behavior as an example of codependency?

She calls in sick for her husband when he is too hung over to work.

46. A child is brought to the emergency room with a broken arm. Because of other injuries, the nurse suspects the child may be a victim of abuse. When the nurse tries to give the child an injection, the child's mother becomes very loud and shouts, "I won't leave my son! Don't you touch him! You'll hurt my child!" What is the best interpretation of the mother's statements? The mother is A) regressing to an earlier behavior pattern. B) sublimating her anger. C) projecting her feelings onto the nurse. D) suppressing her fear.

Projection is attributing one's own thoughts, impulses, or behaviors onto another--it is the mother who is probably harming the child and she is attributing her actions to the nurse (C). The mother may be immature, but (A) is not the best description of her behavior. (B) is substituting a socially acceptable feeling for an unacceptable one. These are not socially acceptable feelings. The mother may be suppressing her fear (D) by displaying anger, but such an interpretation cannot be concluded from the data presented. Correct Answer(s): C

A client who was sexually assaulted 3 hours ago comes to the emergency department of the hospital. The priority is for the staff to help the client feel:

Protected

A 27-year-old female client is admitted to the psychiatric hospital with a diagnosis of bipolar disorder,manic phase. She is demanding and active. Which intervention should the nurse include in this client's plan of care?

Provide a structured environment with little stimuli

The nurse notes that a client is experiencing panic-level anxiety during a group meeting in the community room. Which intervention should be implemented immediately?

Provide calm, brief, and directive communication.

The nurse notes that a client is experiencing panic-level anxiety during a group meeting in the community room. Which intervention should be implemented immediately? Provide calm, brief, and directive communication. Administer anxiolytic medication, as ordered. Teach the client relaxation techniques. Prepare staff members to restrain the client.

Provide calm, brief, and directive communication. Rationale Clients experiencing severe- to panic-level anxiety often feel out of control and need to know they are safe from their own impulses. This can be achieved through firm, short, and simple statements. Reinforcing commonalities in the environment and pointing out reality when there are distortions can also be useful when caring for a severely anxious client.

A registered nurse (RN) in charge of a mental health unit has two additional staff members: a licensed practical nurse (LPN) and a nursing assistant (NA). The unit has 20 clients, with one client on constant observation for acute suicidality. What should the nurse in charge do when making the daily assignments?

Provide client care and administrative duties and assign the LPN to administer medications and to maintain constant observation of the suicidal client

Hospitalization or day-treatment centers are often indicated for the treatment of a client with obsessive-compulsive disorder because these settings:

Provide the neutral environment the client needs to work through conflicts.

Two weeks after a client has been admitted to the mental health hospital, the client's depression begins to lift. The nurse encourages involvement with unit activities, primarily because this type of activity:

Provides for group interaction

A nurse on the psychiatric unit is conducting group therapy with clients who have diagnoses of polydrug abuse. This is a closed group. Four sessions have been held, and the group is now in the working phase. Which strategy is most beneficial for the nurse facilitator to use during the next session?

Providing a balance between support and skillful therapeutic confrontation

During a group discussion of the local news, a client asks how the nurse feels about the events in question. What is the best response by the nurse?

Providing a brief answer and redirecting the focus of the conversation

What should be a priority of nursing care for a client with a dementia resulting from AIDS?

Providing basic intellectual stimulation

The emergency department nurse is providing care for a rape victim. Which action represents an essential element of care for this client?

Providing nonjudgmental care.

The emergency department nurse is providing care for a rape victim. Which action represents an essential element of care for this client? Providing nonjudgmental care. Conveying outrage that this occurred. Sympathizing with the client's sense of shame. Encouraging the client to divulge all the details

Providing nonjudgmental care.

The emergency department nurse is providing care for a rape victim. Which action represents an essential element of care for this client? Providing nonjudgmental care. Conveying outrage that this occurred. Sympathizing with the client's sense of shame. Encouraging the client to divulge all the details

Providing nonjudgmental care.

Which psychotherapeutic theory uses hypnosis, dream interpretation, and free association as methods to release repressed feelings?

Psychoanalytic mode

n a mental health day treatment program, a psychiatric nurse is assessing a client's activity level. The client starts to walk swiftly around the room while rubbing the hands together. What should the nurse conclude that the client is exhibiting?

Psychomotor agitation

51. A 35-year-old male client on the psychiatric ward of a general hospital believes that someone is trying to poison him. The nurse understands that a client's delusions are most likely related to his A) early childhood experiences involving authority issues. B) anger about being hospitalized. C) low self-esteem. D) phobic fear of food.

Psychotic clients have difficulty with trust and have low self-esteem (C). Nursing care should be directed at building trust and promoting positive self-esteem. Activities with limited concentration and no competition should be encouraged in order to build self-esteem. (A, B, and D) are not specifically related to the development of delusions. Correct Answer(s): C

51. A 35-year-old male client on the psychiatric ward of a general hospital believes that someone is trying to poison him. The nurse understands that a client's delusions are most likely related to his A) early childhood experiences involving authority issues. B) anger about being hospitalized. C) low self-esteem. D) phobic fear of food.

Psychotic clients have difficulty with trust and have low self-esteem (C). Nursing care should be directed at building trust and promoting positive self-esteem. Activities with limited concentration and no competition should be encouraged in order to build self-esteem. (A, B, and D) are not specifically related to the development of delusions. Correct Answer(s): C

The staff of a mental health unit is conducting an orientation meeting for newly admitted clients. What should the nurse plan to address first at the meeting?

Purpose of the group meeting

Two depressed clients are sharing a room. The health team has established a goal of increased socialization for each client. Which action will be most effective in facilitating interaction between these two clients?

Putting a puzzle together with them in their room

What should the nurse do when determining whether a client is experiencing adverse effects of risperidone (Risperdal)?

Question if dizziness is experienced.

A client has been taking amoxapine (Asendin) for the past 3 months with no improvement. The practitioner prescribes phenelzine (Nardil) to be given as well. The nurse should:

Question the prescription and withhold the medication..

A client begins fighting and biting other clients. The practitioner prescribes a stat injection of haloperidol (Haldol). How should the nurse implement this prescription?

Quickly, with an attitude of concern

The wife of a client who is dying tells the nurse that although she wants to visit her husband daily, she can visit only twice a week because she works and has to take care of the house and their cat and dog. What defense mechanism does the nurse conclude that the client's wife is using?

Rationalization

A 30-year-old female client asks the nurse to change her room, stating that she hates her roommate and can't stand to be in the same room with her. Just as she finishes speaking, her roommate enters, and the client tells her she missed her and has been all over the unit looking for her. The nurse recognizes that the client is using:

Reaction formation

A client is found to have generalized anxiety disorder. For what behavior should the nurse assess the client to determine the effectiveness of therapy?

Recognizing when anxiety is developing

During a home care visit for a client diagnosed with schizophrenia, the spouse complains to the nurse that the client is refusing to eat or drink anything that is prepared for them. Which intervention should the nurse recommend?

Recommend that unopened, prepackaged food to be given to the client.

During a home care visit for a client diagnosed with schizophrenia, the spouse complains to the nurse that the client is refusing to eat or drink anything that is prepared for them. Which intervention should the nurse recommend? Instruct the caregiver to place the prepared food next to the client and leave the room. Teach the caregiver how to gentle force feed the client with soft mechanical diet. Recommend that unopened, prepackaged food to be given to the client. Suggest that a tube feeding nutrition therapy be initiated.

Recommend that unopened, prepackaged food to be given to the client. Rationale Clients diagnosed with schizophrenia can experience periods of paranoia and this client may believe their food has been tampered and posioned. Offering the client prepackaged food that has not been open, may be the only alternative to get the client to eat and drink.

A client with a conversion disorder is experiencing paralysis of a leg. The nurse can expect this client to:

Recover use of the affected leg but, under stress, to again experience these symptoms.

A manic depressive client on the psychiatric ward becomes loud, and shouts at one of the nurses, "You fat tub of lard! Get something done around here!" What is the best initial action for the nurse to take?

Redirect the client's energy by asking him to tidy the recreation room

The nurse is providing care to a client suspected of Munchausen. The nurse should understand that which example demonstrates a breach of a client's constitutional right to privacy? Releasing information to the client's employer without consent. Discussing the client's history with other staff during care planning. Documenting the client's daily behaviors during hospitalization. Asking the family to share information about the client's behavior before hospitalization.

Releasing information to the client's employer without consent.

A client's history demonstrates a pervasive pattern of unstable and intense relationships, impulsiveness, inappropriate anger, manipulation, offensive behavior, and hostility. The admitting diagnosis is borderline personality disorder. What does the nurse anticipate that this client may attempt to do?

Remain removed from others to avoid interacting with them.

A nurse expects that when an individual successfully completes the grieving process after the death of a significant other, the individual will be able to:

Remember the significant other realistically.

A client is to begin lithium carbonate therapy. The nurse should ensure that before the drug's administration the client has baseline:

Renal studies

Three days after a stressful incident a client can no longer remember why it was stressful. The nurse, in relating to this client, can be most therapeutic by identifying that the inability to recall the situation is an example of the defense mechanism of:

Repression

A female client's stream of consciousness is occupied exclusively with thoughts of her mother's death. The nurse plans to help the client through this stage of grieving, which is known as:

Resolving the loss

What is the priority goal in the planning of care for a client in crisis?

Restoring the client's psychological equilibrium

The multidisciplinary team decides to use a behavior modification approach for a young woman with anorexia nervosa. Which planned nursing intervention is an appropriate approach to use with this client?

Restricting the client to her room until she has gained 2 lb

A client who has been sexually assaulted and is aware of the possible legal implications decides to seek prosecution of the rapist. The nurse carefully listens and documents all assessments. This is done because with a charge of rape the burden of proof:

Rests with the criminal justice system in collaboration with the victim

A client who has been experiencing excessive stress is hospitalized because of an inability to walk. After a physiological basis for the problem is ruled out, a diagnosis of somatoform disorder, conversion type, is made. The nurse concludes that the client's paralysis is a:

Result of intrapsychic conflict

Which nursing diagnosis is the priority for a client who is being treated with lithium?

Risk for fluid imbalance.

Which nursing diagnosis is the priority for a client who is being treated with lithium? Risk for Infection. Self-care deficit. Risk for fluid imbalance. Nutritional deficit.

Risk for fluid imbalance.

2A male client is admitted to the mental health unit because he was feeling depressed about the loss of his wife and job. The client has a history of alcohol dependency and admits that he was drinking alcohol 12 hours ago. Vital signs are: temperature, 100° F, pulse 100, and BP 142/100. The nurse plans to give the client lorazepam (Ativan) based on which priority nursing diagnosis?

Risk for injury related to alcohol detoxification.

During a homecare visit, the nurse is assessing an elderly client diagnosed with bipolar depression who lives alone. The client appears disheveled and is wearing slippers on the wrong feet. The nurse observes various throw rugs, old food containers, and other trash scattered throughout the house. No smoke detectors are visible. The client is unable to recall the date or time of year and not able to have a coherent conversation. Which nursing diagnosis is the priority concern?

Risk for injury.

During a homecare visit, the nurse is assessing an elderly client diagnosed with bipolar depression who lives alone. The client appears disheveled and is wearing slippers on the wrong feet. The nurse observes various throw rugs, old food containers, and other trash scattered throughout the house. No smoke detectors are visible. The client is unable to recall the date or time of year and not able to have a coherent conversation. Which nursing diagnosis is the priority concern? Risk for injury. Self-care deficit. Nutritional deficit. Risk for depression.

Risk for injury. Rationale Safety is always the primary concern when assessing a client. Other nursing diagnoses may need to be addressed, but only after safety needs are met. Information provided in this example includes several factors that contribute to the client's risk for injury (e.g., cognitive decline, trip hazards, etc.).

A client with a family history of diabetes is concerned about the effects of psychiatric medication on the endocrine system. Which psychotropic medication is most likely to cause metabolic syndrome?

Risperidone (Risperdal)

Which diet selection by a depressed client taking the MAO inhibitor tranylcypromine sulfate (Parnate)indicates to the nurse that the client understands the dietary restrictions imposed by this medication regimen?

Roast beef, baked potato with butter, and iced tea

Which diet selection by a client who is depressed and taking the MAO inhibitor tranylcypromine sulfate (Parnate) indicates to the nurse that the client understands the dietary restrictions imposed by this medication regimen?

Roast beef, baked potato with butter, and iced tea.

A constructive but lengthy method of confronting the stress of adolescence and preventing a negative and unhealthy developmental outcome is:

Role experimentation

The nurse is providing information about codependent behaviors to the wife of a client with alcoholism. The nurse is correct to identify and point out to the wife which behavior as an example of codependency? She calls in sick for her husband when he is too hung over to work. She prepares dinner for her husband every night. She shops for all her husband's clothes. She frequently drives her husband to work.

She calls in sick for her husband when he is too hung over to work.

A client is admitted to the medical unit with needle tracks on both arms. A friend has stated that the client uses heroin. The nurse should recognize which group of signs and symptoms in clients suffering from opioid withdrawal?

Runny nose, yawning, insomnia, and chills.

A client is admitted to the medical unit with needle tracks on both arms. A friend has stated that the client uses heroin. The nurse should recognize which group of signs and symptoms in clients suffering from opioid withdrawal? Runny nose, yawning, insomnia, and chills. Slurred speech, excessive drowsiness, and bradycardia. Anxiety, agitation, and aggression. Paranoid delusions, tactile hallucinations, and panic attacks.

Runny nose, yawning, insomnia, and chills.

Antidepressants

SSRI, SNRI, TCA, MAOI

An adolescent on a mental health unit becomes hyperactive. In which activity should the nurse encourage the client to become involved?

Sanding and assembling wooden bookends

An older client whose family has been visiting him in the psychiatric unit is visibly angry and says to the nurse, "My daughter-in-law says they can't take me home until the doctor lets me go. She doesn't understand how important this is to me; she's not from our culture." What should the nurse do?

Say, "You feel she doesn't want you at home."

A client's severe anxiety and panic are often considered "contagious." What action should be taken when a nurse's personal feelings of anxiety are increasing?

Saying, "Another staff member is coming in. I'll leave and come back later."

The nurse observes a client who is admitted to the mental health unit and identifies that the client is talking continuously, using words that rhyme but that have no context or relationship with one topic to the next in the conversation. This client's behavior and thought processes are consistent with which syndrome?

Schizophrenia

A person who is hospitalized for alcoholism becomes boisterous and belligerent and verbally threatens the nurse. What is the most appropriate response by the nurse?

Sedating the client and placing her in a controlled environment

In addition to hallucinating, a client yells and curses throughout the day. The nurse should:

Seek to understand what the behavior means to the client.

After caring for a terminally ill client for several weeks, a nurse becomes increasingly aware of a need for a respite from this assignment. What is the best initial action by the nurse?

Seeking support from colleagues on the unit

A nurse is caring for a client who has abruptly stopped taking a barbiturate. What should the nurse anticipate that the client may experience?

Seizures

A 30-year-old sales manager tells the nurse, "I am thinking about a job change. I don't feel like I amliving up to my potential." Which of Maslow's developmental stages is the sales manager attempting toachieve?

Self-Actualization

A 30-year-old sales manager tells the nurse, "I am thinking about a job change. I don't feel like I am living up to my potential." Which of Maslow's developmental stages is the sales manager attempting to achieve?

Self-Actualization.

21. A 30-year-old sales manager tells the nurse, "I am thinking about a job change. I don't feel like I am living up to my potential." Which of Maslow's developmental stages is the sales manager attempting to achieve? A) Self-Actualization. B) Loving and Belonging. C) Basic Needs. D) Safety and Security.

Self-actualization is the highest level of Maslow's development stages, which is an attempt to fulfill one's full potential (C). (B) is identifying support systems. (C) is the first level of Maslow's developmental stages and is the foundation upon which higher needs rest. Individuals who feel safe and secure (D) in their environment perceive themselves as having physical safety and lack fear of harm. Correct Answer(s): A

21. A 30-year-old sales manager tells the nurse, "I am thinking about a job change. I don't feel like I am living up to my potential." Which of Maslow's developmental stages is the sales manager attempting to achieve? A) Self-Actualization. B) Loving and Belonging. C) Basic Needs. D) Safety and Security.

Self-actualization is the highest level of Maslow's development stages, which is an attempt to fulfill one's full potential (C). (B) is identifying support systems. (C) is the first level of Maslow's developmental stages and is the foundation upon which higher needs rest. Individuals who feel safe and secure (D) in their environment perceive themselves as having physical safety and lack fear of harm. Correct Answer(s): A

SSRI med names

Sertraline Citalopram Escitalopram Paroxetine Fluoxetine

The nurse is providing information about codependent behaviors to the wife of a client with alcoholism. The nurse is correct to identify and point out to the wife which behavior as an example of codependency? She calls in sick for her husband when he is too hung over to work. She prepares dinner for her husband every night. She shops for all her husband's clothes. She frequently drives her husband to work.

She calls in sick for her husband when he is too hung over to work.

A nurse is caring for several extremely depressed clients. The nurse determines that these clients seem to do best in settings where they have:

Simple daily routines

A 44-year-old single woman loses her job and has been unable to find a job for 8 months. She has exhausted her savings and is overwhelmed. She comes to the crisis intervention center because she is despondent and feels hopeless. What type of crisis does the nurse identify?

Situational

54. A woman brings her 48-year-old husband to the outpatient psychiatric unit and describes his behavior to the admitting nurse. She states that he has been sleepwalking, cannot remember who he is, and exhibits multiple personalities. The nurse knows that these behaviors are often associated with A) dissociative disorder. B) obsessive-compulsive disorder. C) panic disorder. D) post-traumatic stress syndrome.

Sleepwalking, amnesia, and multiple personalities are examples of detaching emotional conflict from one's consciousness, which is the definition of a dissociative disorder (A). (B) is characterized by persistent, recurrent intrusive thoughts or urges (obsessions) that are unwilled and cannot be ignored, and provoke impulsive acts (compulsions) such as handwashing. (C) is an acute attack of anxiety characterized by personality disorganization. (D) is re-experiencing a psychologically terrifying or distressing event that is outside the usual range of human experience, such as war, rape, etc. Correct Answer(s): A

54. A woman brings her 48-year-old husband to the outpatient psychiatric unit and describes his behavior to the admitting nurse. She states that he has been sleepwalking, cannot remember who he is, and exhibits multiple personalities. The nurse knows that these behaviors are often associated with A) dissociative disorder. B) obsessive-compulsive disorder. C) panic disorder. D) post-traumatic stress syndrome.

Sleepwalking, amnesia, and multiple personalities are examples of detaching emotional conflict from one's consciousness, which is the definition of a dissociative disorder (A). (B) is characterized by persistent, recurrent intrusive thoughts or urges (obsessions) that are unwilled and cannot be ignored, and provoke impulsive acts (compulsions) such as handwashing. (C) is an acute attack of anxiety characterized by personality disorganization. (D) is re-experiencing a psychologically terrifying or distressing event that is outside the usual range of human experience, such as war, rape, etc. Correct Answer(s): A

15. The nurse is assessing a client's intelligence. Which factor should the nurse remember during this part of the mental status exam? A) Acute psychiatric illnesses impair intelligence. B) Intelligence is influenced by social and cultural beliefs. C) Poor concentration skills suggests limited intelligence. D) The inability to think abstractly indicates limited intelligence.

Social and cultural beliefs (B) have significant impact on intelligence. Chronic psychiatric illness may impair intelligence (A), especially if it remains untreated. Limited concentration does not suggest limited intelligence (C). Difficulties with abstractions are suggestive of psychotic thinking (D), not limited intelligence. Correct Answer(s): B

15. The nurse is assessing a client's intelligence. Which factor should the nurse remember during this part of the mental status exam? A) Acute psychiatric illnesses impair intelligence. B) Intelligence is influenced by social and cultural beliefs. C) Poor concentration skills suggests limited intelligence. D) The inability to think abstractly indicates limited intelligence.

Social and cultural beliefs (B) have significant impact on intelligence. Chronic psychiatric illness may impair intelligence (A), especially if it remains untreated. Limited concentration does not suggest limited intelligence (C). Difficulties with abstractions are suggestive of psychotic thinking (D), not limited intelligence. Correct Answer(s): B

65. A 72-year-old female client is admitted to the psychiatric unit with a diagnosis of major depression. Which statement by the client should be of greatest concern to the nurse and require further assessment? A) I will die if my cat dies. B) I don't feel like eating this morning. C) I just went to my friend's funeral. D) Don't you have more important things to do?

Sometimes a client will use an analogy to describe themselves, and (A) would be an indication for conducting a suicide assessment. (B) could have a variety of etiologies, and while further assessment is indicated, this statement does not indicate potential suicide. Normal grief process differs from depression, and at this client's age peer/cohort deaths are more frequent, so (C) would be within normal limits. (D) is an expression of low self-esteem typical of depression. (B, C, and D) are examples of decreased energy and mood levels which would negate suicide ideation at this time. Correct Answer(s): A

65. A 72-year-old female client is admitted to the psychiatric unit with a diagnosis of major depression. Which statement by the client should be of greatest concern to the nurse and require further assessment? A) I will die if my cat dies. B) I don't feel like eating this morning. C) I just went to my friend's funeral. D) Don't you have more important things to do?

Sometimes a client will use an analogy to describe themselves, and (A) would be an indication for conducting a suicide assessment. (B) could have a variety of etiologies, and while further assessment is indicated, this statement does not indicate potential suicide. Normal grief process differs from depression, and at this client's age peer/cohort deaths are more frequent, so (C) would be within normal limits. (D) is an expression of low self-esteem typical of depression. (B, C, and D) are examples of decreased energy and mood levels which would negate suicide ideation at this time. Correct Answer(s): A

49. The nurse is planning care for a 32-year-old male client diagnosed with HIV infection who has a history of chronic depression. Recently, the client's viral load has begun to increase rather than decrease despite his adherence to the HIV drug regimen. What should the nurse do first while taking the client's history upon admission to the hospital? A) Determine if the client attends a support group weekly. B) Hold all antidepressant medications until further notice. C) Ask the client if he takes St. John's Wort routinely. D) Have the client describe any recent changes in mood.

St. John's Wort, an herbal preparation, is an alternative (nonconventional) therapy for depression, but it may adversely interact with medications used to treat HIV infection (C). The nurse's top priority upon admission is to determine if the client has been taking this herb concurrently with HIV antiviral drugs, which may explain the rise in the viral load. Asking about (A or D) may be helpful in gathering more data about the client's depressive state, but these issues do not have the priority of (C). (B) may be harmful to the client. Correct Answer(s): C

49. The nurse is planning care for a 32-year-old male client diagnosed with HIV infection who has a history of chronic depression. Recently, the client's viral load has begun to increase rather than decrease despite his adherence to the HIV drug regimen. What should the nurse do first while taking the client's history upon admission to the hospital? A) Determine if the client attends a support group weekly. B) Hold all antidepressant medications until further notice. C) Ask the client if he takes St. John's Wort routinely. D) Have the client describe any recent changes in mood.

St. John's Wort, an herbal preparation, is an alternative (nonconventional) therapy for depression, but it may adversely interact with medications used to treat HIV infection (C). The nurse's top priority upon admission is to determine if the client has been taking this herb concurrently with HIV antiviral drugs, which may explain the rise in the viral load. Asking about (A or D) may be helpful in gathering more data about the client's depressive state, but these issues do not have the priority of (C). (B) may be harmful to the client. Correct Answer(s): C

The nurse manager of a mental health unit regularly includes effective boundary-setting as a topic for the monthly staff in-service education sessions. What is the primary principle behind this decision?

Staff members are at risk for problems with boundary-setting.

A 40-year-old male client diagnosed with schizophrenia and alcohol dependence has not had any visitors or phone calls since admission. He reports he has no family that cares about him and was living on the streets prior to this admission. According to Erikson's theory of psychosocial development, which stage is the client in at this time?

Stagnation

The nurse documents the nursing diagnosis "Disturbed sensory perception" for a client with schizophrenia. What did the nurse observe?

Standing in the corner, mumbling and gesticulating

The emergency department nurse is conducting an interview and assisting with the physical examination of a female sexual assault victim. What is most important for the nurse to document on this client's record?

Statements by the client about the sexual assault and the rapist

During the admission process, a client with symptoms of manic behavior has pressured speech punctuated with profanity. What is the most therapeutic approach for the nurse to use to manage this client's behavior?

Stating that the use of profanity should stop because it is inappropriate

A client with the diagnosis of panic disorder jumps when spoken to, complains of feeling uneasy, and says, "It's as though something bad is going to happen." It is most therapeutic at this time for the nurse to:

Stay with the client to be a calming presence.

69. A client is receiving substitution therapy during withdrawal from benzodiazepines. Which expected outcome statement has the highest priority when planning nursing care? A) Client will not demonstrate cross-addiction. B) Co-dependent behaviors will be decreased. C) Excessive CNS stimulation will be reduced. D) Client's level of consciousness will increase.

Substitution therapy with another CNS depressant is intended to decrease the excessive CNS stimulation that can occur during benzodiazepine withdrawal (C). (A, B, and D) are all appropriate outcome statements for the client described, but do not have the priority of (C). Correct Answer(s): C

69. A client is receiving substitution therapy during withdrawal from benzodiazepines. Which expected outcome statement has the highest priority when planning nursing care? A) Client will not demonstrate cross-addiction. B) Co-dependent behaviors will be decreased. C) Excessive CNS stimulation will be reduced. D) Client's level of consciousness will increase.

Substitution therapy with another CNS depressant is intended to decrease the excessive CNS stimulation that can occur during benzodiazepine withdrawal (C). (A, B, and D) are all appropriate outcome statements for the client described, but do not have the priority of (C). Correct Answer(s): C

Considering the anticholinergic-like side effects of many of the psychotropic drugs, the nurse should encourage clients taking these drugs to:

Suck on sugar-free hard candies.

A client on a medical unit refuse to eat and says, "The food is poisoned." The nurse should:

Suggest going to the cafeteria and selecting foods that the client feels safe eating.

A nurse encourages a client to join a self-help group after being discharged from a mental health facility. What is the purpose of having people work in a group?

Support

A nurse is teaching a group of recently hired staff members about conscious and unconscious defense mechanisms that are used to defend the self against anxiety. What is an example of a conscious defense mechanism that the nurse should include?

Suppression

The nurse is performing a pre-op assessment on an 18-month-old client who is scheduled for minor surgery. The nurse observes numerous bruises of different stages over the client's back and buttocks. The mother states that the child must have fallen down while playing alone outside, but she does not provide any specific information. Which statement regarding the assessment data should the nurse enter into the electronic medical record?

Suspected child abuse and neglect.

The nurse is performing a pre-op assessment on an 18-month-old client who is scheduled for minor surgery. The nurse observes numerous bruises of different stages over the client's back and buttocks. The mother states that the child must have fallen down while playing alone outside, but she does not provide any specific information. Which statement regarding the assessment data should the nurse enter into the electronic medical record? Suspected child abuse and neglect. Immature parenting. Normal findings in an 18-month-old. Indications of tissue fragility.

Suspected child abuse and neglect.

The nurse is performing a pre-op assessment on an 18-month-old client who is scheduled for minor surgery. The nurse observes numerous bruises of different stages over the client's back and buttocks. The mother states that the child must have fallen down while playing alone outside, but she does not provide any specific information. Which statement regarding the assessment data should the nurse enter into the electronic medical record? Suspected child abuse and neglect. Immature parenting. Normal findings in an 18-month-old. Indications of tissue fragility.

Suspected child abuse and neglect.

A client has been taking prescribed risperidone (Risperdal) 3 mg twice a day for the past 8 days. A friend brings the client to the outpatient clinic. The client reports tremors, shortness of breath, a fever, and sweating. What should the nurse do?

Take the client's vital signs and arrange for immediate transfer to a hospital.

What is an initial client objective in relation to anger management?

Taking responsibility for the hostile behavior

nurse discusses the plan of care with a depressed client whose husband has recently died. The nurse determines that it will be most helpful to:

Talk with the client about her husband and the details of his death.

A 5-year-old cries and screams continuously from the time their mother drops them off at kindergarten until she picks him up 4 hours later. He is calm and relaxed when he is with his mother. The mother seeks advice from a friend who is a nurse. Which response by the nurse is best?

Talk with your healthcare provider about referring him to a mental health clinic."

A 5-year-old cries and screams continuously from the time their mother drops them off at kindergarten until she picks him up 4 hours later. He is calm and relaxed when he is with his mother. The mother seeks advice from a friend who is a nurse. Which response by the nurse is best? "Talk with your healthcare provider about referring him to a mental health clinic." "Talk with the school principal about withdrawing him until he is more mature." "Arrange with the teacher to let him call home during play time." "Send a picture of yourself to school to keep with him."

Talk with your healthcare provider about referring him to a mental health clinic." Rationale Separation anxiety disorder becomes apparent when the child is separated from the attachment figure. The first evidence of this disorder often occurs when the child begins going to school, and it may be based of the child's fear that something will happen to the attachment figure. Professional help is needed to learn how to cope with anxiety effectively. Separation anxiety is common childhood condition that usually becomes evident around seven months of age, but should subside around three years of age.

Physical examination of a 6-year-old reveals several bite marks in various locations on his body. X-ray examination reveals healed fractures of the ribs. The mother tells the nurse that her child is always having accidents. Which initial response by the nurse would be most appropriate?

Tell me more specifically about your child's accidents.

An elderly female client with advanced dementia is admitted to the hospital with a fractured hip. The client repeatedly tells the staff, "Take me home. I want my Mommy." Which response is best for the nurse to provide?

Tell the client that the nurse is there and will help her

Ten minutes before lunch, a client with obsessive-compulsive behavior begins the ritual of changing clothes for the fourth time. How should the nurse respond to this behavior?

Tell the client to finish changing clothes and say that lunch can be eaten afterward.

A male client with schizophrenia tells the nurse that the voices he hears are saying, "You must kill yourself." To assist the client in coping with these thoughts, which response is best for the nurse to provide?

Tell yourself that the voices are unreasonable.

How can a nurse in the mental health clinic best prepare a client for termination of their therapeutic relationship?

Telling the client during their first meeting how long their entire therapeutic relationship will last

12. The nurse is planning discharge for a male client with schizophrenia. The client insists that he is returning to his apartment, although the healthcare provider informed him that he will be moving to a boarding home. What is the most important nursing diagnosis for discharge planning? A) Ineffective denial related to situational anxiety. B) Ineffective coping related to inadequate support. C) Social isolation related to difficult interactions. D) Self-care deficit related to cognitive impairment.

The best nursing diagnosis is (A) because the client is unable to acknowledge the move to a boarding home. (B, C, and D) are potential nursing diagnoses, but denial is most important because it is a defense mechanism that keeps the client from dealing with his feelings about living arrangements. Correct Answer(s): A

12. The nurse is planning discharge for a male client with schizophrenia. The client insists that he is returning to his apartment, although the healthcare provider informed him that he will be moving to a boarding home. What is the most important nursing diagnosis for discharge planning? A) Ineffective denial related to situational anxiety. B) Ineffective coping related to inadequate support. C) Social isolation related to difficult interactions. D) Self-care deficit related to cognitive impairment.

The best nursing diagnosis is (A) because the client is unable to acknowledge the move to a boarding home. (B, C, and D) are potential nursing diagnoses, but denial is most important because it is a defense mechanism that keeps the client from dealing with his feelings about living arrangements. Correct Answer(s): A

31. A female client with depression attends group and states that she sometimes misses her medication appointments because she feels very anxious about riding the bus. Which statement is the nurse's best response? A) Can your case manager take you to your appointments? B) Take your medication for anxiety before you ride the bus. C) Let's talk about what happens when you feel very anxious. D) What are some ways that you can cope with your anxiety?

The best response is to explore ways for the client to cope with anxiety (D). The nurse should encourage problem-solving rather than dependence on the case manager (A) for transportation. Strategies for coping with anxiety should be encouraged before suggesting (B). (C) is therapeutic, but the best response is an open-ended question to explore ways to cope with the anxiety. Correct Answer(s): D

31. A female client with depression attends group and states that she sometimes misses her medication appointments because she feels very anxious about riding the bus. Which statement is the nurse's best response? A) Can your case manager take you to your appointments? B) Take your medication for anxiety before you ride the bus. C) Let's talk about what happens when you feel very anxious. D) What are some ways that you can cope with your anxiety?

The best response is to explore ways for the client to cope with anxiety (D). The nurse should encourage problem-solving rather than dependence on the case manager (A) for transportation. Strategies for coping with anxiety should be encouraged before suggesting (B). (C) is therapeutic, but the best response is an open-ended question to explore ways to cope with the anxiety. Correct Answer(s): D

A client with depression is to be given fluoxetine (Prozac). What precaution should the nurse consider when initiating treatment with this drug?

The blood level may not be sufficient to cause noticeable improvement for 2 to 4 weeks.

22. A male client with mental illness and substance dependency tells the mental health nurse that he has started using illegal drugs again and wants to seek treatment. Since he has a dual diagnosis, which person is best for the nurse to refer this client to first? A) The emergency room nurse. B) His case manager. C) The clinic healthcare provider. D) His support group sponsor.

The case manager (B) is responsible for coordinating community services, and since this client has a dual diagnosis, this is the best person to describe available treatment options. (A) is unnecessary, unless the client experiences behaviors that threaten his safety or the safety of others. (C and D) might also be useful, but it is most important at this time that a treatment program be coordinated to meet this client's needs. Correct Answer(s): B

22. A male client with mental illness and substance dependency tells the mental health nurse that he has started using illegal drugs again and wants to seek treatment. Since he has a dual diagnosis, which person is best for the nurse to refer this client to first? A) The emergency room nurse. B) His case manager. C) The clinic healthcare provider. D) His support group sponsor.

The case manager (B) is responsible for coordinating community services, and since this client has a dual diagnosis, this is the best person to describe available treatment options. (A) is unnecessary, unless the client experiences behaviors that threaten his safety or the safety of others. (C and D) might also be useful, but it is most important at this time that a treatment program be coordinated to meet this client's needs. Correct Answer(s): B

A nurse is working with clients with a variety of eating disorders. Which characteristic unique to bulimia nervosa differentiates this disorder from anorexia nervosa?

The client behaves appropriately and looks normal.

26. The nurse observes a client who is admitted to the mental health unit and identifies that the client is talking continuously, using words that rhyme but that have no context or relationship with one topic to the next in the conversation. This client's behavior and thought processes are consistent with which syndrome? A) Dementia. B) Depression. C) Schizophrenia. D) Chronic brain syndrome.

The client is demonstrating symptoms of schizophrenia (C), such as disorganized speech that may include word salad (communication that includes both real and imaginary words in no logical order), incoherent speech, and clanging (rhyming). Dementia (A) is a global impairment of intellectual (cognitive) functions that may be progressive, such as Alzheimer's or organic brain syndrome (D). Depression (C) is typified by psychomotor retardation, and the client appears to be slowed down in movement, in speech, and would appear listless and disheveled. Correct Answer(s): C

26. The nurse observes a client who is admitted to the mental health unit and identifies that the client is talking continuously, using words that rhyme but that have no context or relationship with one topic to the next in the conversation. This client's behavior and thought processes are consistent with which syndrome? A) Dementia. B) Depression. C) Schizophrenia. D) Chronic brain syndrome.

The client is demonstrating symptoms of schizophrenia (C), such as disorganized speech that may include word salad (communication that includes both real and imaginary words in no logical order), incoherent speech, and clanging (rhyming). Dementia (A) is a global impairment of intellectual (cognitive) functions that may be progressive, such as Alzheimer's or organic brain syndrome (D). Depression (C) is typified by psychomotor retardation, and the client appears to be slowed down in movement, in speech, and would appear listless and disheveled. Correct Answer(s): C

34. A 40-year-old male client diagnosed with schizophrenia and alcohol dependence has not had any visitors or phone calls since admission. He reports he has no family that cares about him and was living on the streets prior to this admission. According to Erikson's theory of psychosocial development, which stage is the client in at this time? A) Isolation. B) Stagnation. C) Despair. D) Role confusion.

The client is in Erikson's "Generativity vs. Stagnation" stage (age 24 to 45), and meeting the task includes maintaining intimate relationships and moving toward developing a family (B). (A) occurs in young adulthood (age 18 to 25), (C) occurs in maturity (age 45 to death), and (D) occurs in adolescence (age 12 to 20). These are all stages that occur if individuals are not successfully coping with their psychosocial developmental stage. Correct Answer(s): B

34. A 40-year-old male client diagnosed with schizophrenia and alcohol dependence has not had any visitors or phone calls since admission. He reports he has no family that cares about him and was living on the streets prior to this admission. According to Erikson's theory of psychosocial development, which stage is the client in at this time? A) Isolation. B) Stagnation. C) Despair. D) Role confusion.

The client is in Erikson's "Generativity vs. Stagnation" stage (age 24 to 45), and meeting the task includes maintaining intimate relationships and moving toward developing a family (B). (A) occurs in young adulthood (age 18 to 25), (C) occurs in maturity (age 45 to death), and (D) occurs in adolescence (age 12 to 20). These are all stages that occur if individuals are not successfully coping with their psychosocial developmental stage. Correct Answer(s): B

The nurse documents that a client diagnosed with stage 3 Alzheimer's disease presents with aphasia. Which client behavior supports this finding?

The client is no longer able to speak.

The nurse documents that a client diagnosed with stage 3 Alzheimer's disease presents with aphasia. Which client behavior supports this finding? The client is no longer able to speak. The client cannot recognize pictures of family members. The client has lost the ability to walk. The client cannot recall where personal items are located.

The client is no longer able to speak.

The visiting nurse is assessing a client with early-stage dementia who has recently been prescribed donepezil (Aricept). Which information obtained from the client's family member demonstrates that the medication is effective?

The client is showering and getting dressed with minimal assistance.

The visiting nurse is assessing a client with early-stage dementia who has recently been prescribed donepezil (Aricept). Which information obtained from the client's family member demonstrates that the medication is effective? The client is showering and getting dressed with minimal assistance. The client uses confabulation to cover up memory loss. The client did not attend the family reunion last weekend. The client arranges to have groceries delivered.

The client is showering and getting dressed with minimal assistance.

The visiting nurse is assessing a client with early-stage dementia who has recently been prescribed donepezil (Aricept). Which information obtained from the client's family member demonstrates that the medication is effective? The client is showering and getting dressed with minimal assistance. The client uses confabulation to cover up memory loss. The client did not attend the family reunion last weekend. The client arranges to have groceries delivered.

The client is showering and getting dressed with minimal assistance.

25. A client, who is on a 30-day commitment to a drug rehabilitation unit, asks the nurse if he can go for a walk on the grounds of the treatment center. When he is told that his privileges do not include walking on the grounds, the client becomes verbally abusive. Which approach should the nurse use? A) Call a staff member to escort the client to his room. B) Tell the client to talk to his healthcare provider about his privileges. C) Remind the client of the unit rules. D) Ignore the client's inappropriate behavior.

The client is trying to engage the nurse in a dispute. Ignoring the behavior (D) provides no reinforcement for the inappropriate behavior. (A) is not necessary unless the client becomes a physical threat to the nurse. (B) would be inappropriate, because it is referring the situation to the healthcare provider and is not in keeping with good health team management. Consistent limits must be established and enforced. (C) would subject the nurse to more verbal abuse because the client could use any response as an excuse to attack the nurse once again. Correct Answer(s): D

25. A client, who is on a 30-day commitment to a drug rehabilitation unit, asks the nurse if he can go for a walk on the grounds of the treatment center. When he is told that his privileges do not include walking on the grounds, the client becomes verbally abusive. Which approach should the nurse use? A) Call a staff member to escort the client to his room. B) Tell the client to talk to his healthcare provider about his privileges. C) Remind the client of the unit rules. D) Ignore the client's inappropriate behavior.

The client is trying to engage the nurse in a dispute. Ignoring the behavior (D) provides no reinforcement for the inappropriate behavior. (A) is not necessary unless the client becomes a physical threat to the nurse. (B) would be inappropriate, because it is referring the situation to the healthcare provider and is not in keeping with good health team management. Consistent limits must be established and enforced. (C) would subject the nurse to more verbal abuse because the client could use any response as an excuse to attack the nurse once again. Correct Answer(s): D

A client with schizophrenia is attending group therapy sessions. Which actions by the client demonstrate that he is progressing to the stabilizing phase of illness?

The client participates and states that his delusions are not real, but that he needs them to feel better.

A client with schizophrenia is attending group therapy sessions. Which actions by the client demonstrate that he is progressing to the stabilizing phase of illness? The client participates and attempts to convince others that his delusions are real. The client admits to past delusions but states that he is a rock star who no longer needs group therapy. The client participates and states that his delusions are not real, but that he needs them to feel better. The client denies having delusions but states that he writes to government officials to report his spy activities.

The client participates and states that his delusions are not real, but that he needs them to feel better.

The nurse is counseling a client who is dealing with complicated grief over the death of a spouse. Which statement reflects the most desirable outcome for the client?

The client will attend a surviving spousal support groups.

The nurse is counseling a client who is dealing with complicated grief over the death of a spouse. Which statement reflects the most desirable outcome for the client? The client will attend a surviving spousal support groups. The client will plan a memorial tribute for the spouse. The client will stop expressing feelings of loss every day. The client will plan a vacation after finalizing the burial arrangements.

The client will attend a surviving spousal support groups.

A client with delirium keeps attempting to get out of bed and has fallen twice, despite being under close observation. The charge nurse calls the health care provider to obtain an order for mechanical restraints. Which statement is correct regarding the mechanical restraint policy?

The client will need to remain on close-observation with a documentation note every 15 minutes.

A client with delirium keeps attempting to get out of bed and has fallen twice, despite being under close-observation. The charge nurse calls the health care provider to obtain an order for mechanical restraints. Which statement is correct regarding the mechanical restraint policy? The client will need to remain on close-observation with a documentation note every 15 minutes. The client no longer require close-observation, but must be checked at regular intervals. The client may be sedated and left alone as long as the restraints remain in place. Restraints may be applied based on verbal orders received over the phone.

The client will need to remain on close-observation with a documentation note every 15 minutes.

A client with a long history of alcohol abuse who has been hospitalized for 1 week tells the nurse, "I feel much better and probably won't need any more treatment." What does the nurse conclude when evaluating the client's progress?

The client's lack of insight into the emotional aspects of the illness indicates the need for continued supervision.

Which is the most important assessment data for a nurse to gather from the client in crisis?

The client's perception of the circumstances surrounding the crisis

73. A 52-year-old male client in the intensive care unit who has been oriented suddenly becomes disoriented and fearful. Assessment of vital signs and other physical parameters reveal no significant change and the nurse formulates the diagnosis, "Confusion related to ICU psychosis." Which intervention is best to implement? A) Move all machines away from the client's immediate area. B) Attempt to allay the client's fears by explaining the etiology of his condition. C) Cluster care so that brief periods of rest can be scheduled during the day. D) Extend visitation times for family and friends.

The critical care environment confronts clients with an environment which provides stressors heightened by treatment modalities that may prove to be lifesaving. These stressors can result in isolation and confusion. The best intervention is to provide the client with rest periods (C). (A) is not practical--the machinery is often lifesaving. The client is not ready for (B). Although family and friends (D) can provide a support system to the client, visits should be limited because of the critical care that must be provided. Correct Answer(s): C

73. A 52-year-old male client in the intensive care unit who has been oriented suddenly becomes disoriented and fearful. Assessment of vital signs and other physical parameters reveal no significant change and the nurse formulates the diagnosis, "Confusion related to ICU psychosis." Which intervention is best to implement? A) Move all machines away from the client's immediate area. B) Attempt to allay the client's fears by explaining the etiology of his condition. C) Cluster care so that brief periods of rest can be scheduled during the day. D) Extend visitation times for family and friends.

The critical care environment confronts clients with an environment which provides stressors heightened by treatment modalities that may prove to be lifesaving. These stressors can result in isolation and confusion. The best intervention is to provide the client with rest periods (C). (A) is not practical--the machinery is often lifesaving. The client is not ready for (B). Although family and friends (D) can provide a support system to the client, visits should be limited because of the critical care that must be provided. Correct Answer(s): C

Based on non-compliance with the medication regimen, an adult client with a medical diagnosis of substance abuse and schizophrenia was recently switched from oral fluphenazine HCl (Prolixin) to IM fluphenazine decanoate (Prolixin Decanoate). What is most important to teach the client and family about this change in medication regimen?

The effects of alcohol and drug interaction.

Patient with schizophrenia, drug and alcohol abuse in hospital for hepatitis, contact healthcare provider before giving

acetaminophen

A female client is brought to the emergency department after police officers found her disoriented, disorganized and confused. The RN also determines that the client is homeless and is exhibiting suspiciousness. The client's plan of care should include what priority problem?

acute confusion

inability to recognize objects

agnosia

66. The nurse is conducting discharge teaching for a client with schizophrenia who plans to live in a group home. Which statement is most indicative of the need for careful follow-up after discharge? A) Crickets are a good source of protein. B) I have not heard any voices for a week. C) Only my belief in God can help me. D) Sometimes I have a hard time sitting still.

The most frequent cause of increased symptoms in psychotic clients is non-compliance with the medication regimen. If clients believe that "God alone" is going to heal them (C), then they may discontinue their medication, so (C) would pose the greatest threat to this client's prognosis. (A) would require further teaching, but is not as significant a statement as (C). (B) indicates an improvement in the client's condition. (D) may be a sign of anxiety that could improve with treatment, but does not have the priority of (C). Correct Answer(s): C

66. The nurse is conducting discharge teaching for a client with schizophrenia who plans to live in a group home. Which statement is most indicative of the need for careful follow-up after discharge? A) Crickets are a good source of protein. B) I have not heard any voices for a week. C) Only my belief in God can help me. D) Sometimes I have a hard time sitting still.

The most frequent cause of increased symptoms in psychotic clients is non-compliance with the medication regimen. If clients believe that "God alone" is going to heal them (C), then they may discontinue their medication, so (C) would pose the greatest threat to this client's prognosis. (A) would require further teaching, but is not as significant a statement as (C). (B) indicates an improvement in the client's condition. (D) may be a sign of anxiety that could improve with treatment, but does not have the priority of (C). Correct Answer(s): C

41. A female client refuses to take an oral hypoglycemic agent because she believes that the drug is being administered as part of an elaborate plan by the Mafia to harm her. Which nursing intervention is most important to include in this client's plan of care? A) Reassure the client that no one will harm her while she is in the hospital. B) Ask the healthcare provider to give the client the medication. C) Explain that the diabetic medication is important to take. D) Reassess client's mental status for thought processes and content.

The most important intervention is to reassess the client's mental status (D) and to take further action based on the findings of this assessment. Attempting to reassure the client (A) is in effect arguing with the client's delusions and could escalate an already anxious situation. Collaborating about diabetic care (B and C) is not likely to help change the client's false beliefs. Correct Answer(s): D

41. A female client refuses to take an oral hypoglycemic agent because she believes that the drug is being administered as part of an elaborate plan by the Mafia to harm her. Which nursing intervention is most important to include in this client's plan of care? A) Reassure the client that no one will harm her while she is in the hospital. B) Ask the healthcare provider to give the client the medication. C) Explain that the diabetic medication is important to take. D) Reassess client's mental status for thought processes and content.

The most important intervention is to reassess the client's mental status (D) and to take further action based on the findings of this assessment. Attempting to reassure the client (A) is in effect arguing with the client's delusions and could escalate an already anxious situation. Collaborating about diabetic care (B and C) is not likely to help change the client's false beliefs. Correct Answer(s): D

2. A male client is admitted to the mental health unit because he was feeling depressed about the loss of his wife and job. The client has a history of alcohol dependency and admits that he was drinking alcohol 12 hours ago. Vital signs are: temperature, 100° F, pulse 100, and BP 142/100. The nurse plans to give the client lorazepam (Ativan) based on which priority nursing diagnosis? A) Risk for injury related to suicidal ideation. B) Risk for injury related to alcohol detoxification. C) Knowledge deficit related to ineffective coping. D) Health seeking behaviors related to personal crisis.

The most important nursing diagnosis is related to alcohol detoxification (B) because the client has elevated vital signs, a sign of alcohol detoxification. Maintaining client safety related to (A) should be addressed after giving the client Ativan for elevated vital signs secondary to alcohol withdrawal. (C and D) can be addressed when immediate needs for safety are met. Correct Answer(s): B

fear of leaving the home environment

agoraphobia

2. A male client is admitted to the mental health unit because he was feeling depressed about the loss of his wife and job. The client has a history of alcohol dependency and admits that he was drinking alcohol 12 hours ago. Vital signs are: temperature, 100° F, pulse 100, and BP 142/100. The nurse plans to give the client lorazepam (Ativan) based on which priority nursing diagnosis? A) Risk for injury related to suicidal ideation. B) Risk for injury related to alcohol detoxification. C) Knowledge deficit related to ineffective coping. D) Health seeking behaviors related to personal crisis.

The most important nursing diagnosis is related to alcohol detoxification (B) because the client has elevated vital signs, a sign of alcohol detoxification. Maintaining client safety related to (A) should be addressed after giving the client Ativan for elevated vital signs secondary to alcohol withdrawal. (C and D) can be addressed when immediate needs for safety are met. Correct Answer(s): B

8. The community health nurse talks to a male client who has bipolar disorder. The client explains that he sleeps 4 to 5 hours a night and is working with his partner to start two new businesses and build an empire. The client stopped taking his medications several days ago. What nursing problem has the highest priority? A) Excessive work activity. B) Decreased need for sleep. C) Medication management. D) Inflated self-esteem.

The most important nursing problem is medication management (C) because compliance with the medication regimen will help prevent hospitalization. The client is also exhibiting signs of (A, B, and C); however, these problems do not have the priority of medication management. Correct Answer(s): C

8. The community health nurse talks to a male client who has bipolar disorder. The client explains that he sleeps 4 to 5 hours a night and is working with his partner to start two new businesses and build an empire. The client stopped taking his medications several days ago. What nursing problem has the highest priority? A) Excessive work activity. B) Decreased need for sleep. C) Medication management. D) Inflated self-esteem.

The most important nursing problem is medication management (C) because compliance with the medication regimen will help prevent hospitalization. The client is also exhibiting signs of (A, B, and C); however, these problems do not have the priority of medication management. Correct Answer(s): C

3. The charge nurse is collaborating with the nursing staff about the plan of care for a client who is very depressed. What is the most important intervention to implement during the first 48 hours after the client's admission to the unit? A) Monitor appetite and observe intake at meals. B) Maintain safety in the client's milieu. C) Provide ongoing, supportive contact. D) Encourage participation in activities.

The most important reason for closely observing a depressed client immediately after admission is to maintain safety (B), since suicide is a risk with depression. (A, C, and D) are all important interventions, but safety is the priority. Correct Answer(s): B

3. The charge nurse is collaborating with the nursing staff about the plan of care for a client who is very depressed. What is the most important intervention to implement during the first 48 hours after the client's admission to the unit? A) Monitor appetite and observe intake at meals. B) Maintain safety in the client's milieu. C) Provide ongoing, supportive contact. D) Encourage participation in activities.

The most important reason for closely observing a depressed client immediately after admission is to maintain safety (B), since suicide is a risk with depression. (A, C, and D) are all important interventions, but safety is the priority. Correct Answer(s): B

A client with a dissociative identity disorder is to be discharged after a 2-week hospitalization. The nurse, evaluating the effectiveness of the short-term therapy, expects the client to verbalize:

The need for long-term outpatient psychotherapy.

A recently hired nurse is caring for several clients on a mental health unit at a local community hospital. The nurse manager is evaluating the nurse's performance. What situation indicates that the nurse-client boundaries of the recently hired nurse are appropriate?

The nurse shares with the entire treatment team vital information the client disclosed in a private session.

7. The wife of a male client recently diagnosed with schizophrenia asks the nurse, "What exactly is schizophrenia? Is my husband all right?" Which response is best for the nurse to provide to this family member? A) It sounds like you're worried about your husband. Let's sit down and talk. B) It is a chemical imbalance in the brain that causes disorganized thinking. C) Your husband will be just fine if he takes his medications regularly. D) I think you should talk to your husband's psychologist about this question.

The nurse should answer the client's question with factual information and explain that schizophrenia is a chemical imbalance in the brain (B). (A) is a therapeutic response but does not answer the question, and may be an appropriate response after the nurse answers the question asked. Although (C) is likely true to some degree, it is also true that some clients continue to have disorganized thinking even with antipsychotic medications. Referring the spouse to the psychologist (D) is avoiding the issue; the nurse can and should answer the question. Correct Answer(s): B

7. The wife of a male client recently diagnosed with schizophrenia asks the nurse, "What exactly is schizophrenia? Is my husband all right?" Which response is best for the nurse to provide to this family member? A) It sounds like you're worried about your husband. Let's sit down and talk. B) It is a chemical imbalance in the brain that causes disorganized thinking. C) Your husband will be just fine if he takes his medications regularly. D) I think you should talk to your husband's psychologist about this question.

The nurse should answer the client's question with factual information and explain that schizophrenia is a chemical imbalance in the brain (B). (A) is a therapeutic response but does not answer the question, and may be an appropriate response after the nurse answers the question asked. Although (C) is likely true to some degree, it is also true that some clients continue to have disorganized thinking even with antipsychotic medications. Referring the spouse to the psychologist (D) is avoiding the issue; the nurse can and should answer the question. Correct Answer(s): B

45. A nurse working on a mental health unit receives a community call from a person who is tearful and states, "I just feel so nervous all of the time. I don't know what to do about my problems. I haven't been able to sleep at night and have hardly eaten for the past 3 or 4 days." The nurse should initiate a referral based on which assessment? A) Altered thought processes. B) Moderate levels of anxiety. C) Inadequate social support. D) Altered health maintenance.

The nurse should initiate a referral based on anxiety levels (B) and feelings of nervousness that interfere with sleep, appetite, and the inability to solve problems. The client does not report symptoms of (A) or evidence of (C). There is not enough information to initiate a referral based on (D). Correct Answer(s): B

45. A nurse working on a mental health unit receives a community call from a person who is tearful and states, "I just feel so nervous all of the time. I don't know what to do about my problems. I haven't been able to sleep at night and have hardly eaten for the past 3 or 4 days." The nurse should initiate a referral based on which assessment? A) Altered thought processes. B) Moderate levels of anxiety. C) Inadequate social support. D) Altered health maintenance.

The nurse should initiate a referral based on anxiety levels (B) and feelings of nervousness that interfere with sleep, appetite, and the inability to solve problems. The client does not report symptoms of (A) or evidence of (C). There is not enough information to initiate a referral based on (D). Correct Answer(s): B

20. A male client with schizophrenia tells the nurse that the voices he hears are saying, "You must kill yourself." To assist the client in coping with these thoughts, which response is best for the nurse to provide? A) Tell yourself that the voices are unreasonable. B) Exercise when you hear the voices. C) Talk to someone when you hear the voices. D) The voices aren't real, so ignore them.

The nurse should teach the client to use self-talk to disprove the voices (A). Although (B) may be helpful, the client's concrete thinking may make it difficult to understand this suggestion. Clients with schizophrenia have difficulty initiating interaction with others (C). Auditory hallucinations are often relentless, so it is difficult to ignore them (D). Correct Answer(s): A

20. A male client with schizophrenia tells the nurse that the voices he hears are saying, "You must kill yourself." To assist the client in coping with these thoughts, which response is best for the nurse to provide? A) Tell yourself that the voices are unreasonable. B) Exercise when you hear the voices. C) Talk to someone when you hear the voices. D) The voices aren't real, so ignore them.

The nurse should teach the client to use self-talk to disprove the voices (A). Although (B) may be helpful, the client's concrete thinking may make it difficult to understand this suggestion. Clients with schizophrenia have difficulty initiating interaction with others (C). Auditory hallucinations are often relentless, so it is difficult to ignore them (D). Correct Answer(s): A

The nurse suspects child abuse when assessing a 3-year-old boy and noticing several small, round burns on his legs and trunk that might be the result of cigarette burns. Which parental behavior provides the greatest validation for such suspicions?

The parents tell the nurse that the child was burned in a house fire which is incompatible with the nurse's observation of the type of burn

A nurse educator is leading a class on supporting middle-aged adults who are experiencing midlife crises. What should the nurse include as the most significant factor in the development of this type of crisis?

The perception of their life situation

9. At a support meeting of parents of a teenager with polysubstance dependency, a parent states, "Each time my son tries to quit taking drugs, he gets so depressed that I'm afraid he will commit suicide." The nurse's response should be based on which information? A) Addiction is a chronic, incurable disease. B) Tolerance to the effects of drugs causes feelings of depression. C) Feelings of depression frequently lead to drug abuse and addiction. D) Careful monitoring should be provided during withdrawal from the drugs.

The priority is to teach the parents that their son will need monitoring and support during withdrawal (D) to ensure that he does not attempt suicide. Although (A and C) are true, they are not as relevant to the parent's expressed concern. There is no information to support (B). Correct Answer(s): D

9. At a support meeting of parents of a teenager with polysubstance dependency, a parent states, "Each time my son tries to quit taking drugs, he gets so depressed that I'm afraid he will commit suicide." The nurse's response should be based on which information? A) Addiction is a chronic, incurable disease. B) Tolerance to the effects of drugs causes feelings of depression. C) Feelings of depression frequently lead to drug abuse and addiction. D) Careful monitoring should be provided during withdrawal from the drugs.

The priority is to teach the parents that their son will need monitoring and support during withdrawal (D) to ensure that he does not attempt suicide. Although (A and C) are true, they are not as relevant to the parent's expressed concern. There is no information to support (B). Correct Answer(s): D

47. An anxious client expressing a fear of people and open places is admitted to the psychiatric unit. What is the most effective way for the nurse to assist this client? A) Plan an outing within the first week of admission. B) Distract her whenever she expresses her discomfort about being with others. C) Confront her fears and discuss the possible causes of these fears. D) Accompany her outside for an increasing amount of time each day.

The process of gradual desensitization by controlled exposure to the situation which is feared (D), is the treatment of choice in phobic reactions. (A and C) are far too aggressive for the initial treatment period and could even be considered hostile. (B) promotes denial of the problem, and gives the client the message that discussion of the phobia is not permitted. Correct Answer(s): D

47. An anxious client expressing a fear of people and open places is admitted to the psychiatric unit. What is the most effective way for the nurse to assist this client? A) Plan an outing within the first week of admission. B) Distract her whenever she expresses her discomfort about being with others. C) Confront her fears and discuss the possible causes of these fears. D) Accompany her outside for an increasing amount of time each day.

The process of gradual desensitization by controlled exposure to the situation which is feared (D), is the treatment of choice in phobic reactions. (A and C) are far too aggressive for the initial treatment period and could even be considered hostile. (B) promotes denial of the problem, and gives the client the message that discussion of the phobia is not permitted. Correct Answer(s): D

A client with an antisocial personality disorder is being admitted to a mental health unit. What information should the nurse include in this initial interview?

The unit's usual routines and rules

61. A 46-year-old female client has been on antipsychotic neuroleptics for the past three days. She has had a decrease in psychotic behavior and appears to be responding well to the medication. On the fourth day, the client's blood pressure increases, she becomes pale and febrile, and demonstrates muscular rigidity. Which action should the nurse initiate? A) Place the client on seizure precautions and monitor carefully. B) Immediately transfer the client to ICU. C) Describe the symptoms to the charge nurse and record on the client's chart. D) No action is required at this time as these are known side effects of such drugs.

These symptoms are descriptive of neuroleptic malignant syndrome (NMS) which is an extremely serious/life threatening reaction to neuroleptic drugs (B). The major symptoms of this syndrome are fever, rigidity, autonomic instability, and encephalopathy. Respiratory failure, cardiovascular collapse, arrhythmias, and/or renal failure can result in death. This is an EMERGENCY situation, and the client requires immediate critical care. Seizure precautions (A) are not indicated in this situation. (C and D) do not consider the seriousness of the situation. Correct Answer(s): B

While sitting in the day room of the mental health unit, a male adolescent avoids eye contact, looks at the floor and talks softly when interacting verbally with the RN. the two trade places, and the RN demonstrates the client's behaviors. what is the main goal of this therapeutic technique?

allow the client to identify the way he interacts

illusion is a misinterpretation of

an existing external stimulus

61. A 46-year-old female client has been on antipsychotic neuroleptics for the past three days. She has had a decrease in psychotic behavior and appears to be responding well to the medication. On the fourth day, the client's blood pressure increases, she becomes pale and febrile, and demonstrates muscular rigidity. Which action should the nurse initiate? A) Place the client on seizure precautions and monitor carefully. B) Immediately transfer the client to ICU. C) Describe the symptoms to the charge nurse and record on the client's chart. D) No action is required at this time as these are known side effects of such drugs.

These symptoms are descriptive of neuroleptic malignant syndrome (NMS) which is an extremely serious/life threatening reaction to neuroleptic drugs (B). The major symptoms of this syndrome are fever, rigidity, autonomic instability, and encephalopathy. Respiratory failure, cardiovascular collapse, arrhythmias, and/or renal failure can result in death. This is an EMERGENCY situation, and the client requires immediate critical care. Seizure precautions (A) are not indicated in this situation. (C and D) do not consider the seriousness of the situation. Correct Answer(s): B

43. A male adolescent is admitted with bipolar disorder after being released from jail for assault with a deadly weapon. When the nurse asks the teen to identify his reason for the assault, he replies, "Because he made me mad!" Which goal is best for the nurse to include in the client's plan of care? The client will A) outline methods for managing anger. B) control impulsive actions toward self and others. C) verbalize feelings when anger occurs. D) recognize consequences for behaviors exhibited.

Those with bipolar disorder often exhibit poor impulse control, and the most important goal for this client at this time is to learn to control impulsive behavior (B) so that he can avert the social consequences related to such behaviors. (A, C, and D) are important goals, but they do not address the acute issue of impulse control, which is necessary to reduce the likelihood of harming self or others. Correct Answer(s): B

43. A male adolescent is admitted with bipolar disorder after being released from jail for assault with a deadly weapon. When the nurse asks the teen to identify his reason for the assault, he replies, "Because he made me mad!" Which goal is best for the nurse to include in the client's plan of care? The client will A) outline methods for managing anger. B) control impulsive actions toward self and others. C) verbalize feelings when anger occurs. D) recognize consequences for behaviors exhibited.

Those with bipolar disorder often exhibit poor impulse control, and the most important goal for this client at this time is to learn to control impulsive behavior (B) so that he can avert the social consequences related to such behaviors. (A, C, and D) are important goals, but they do not address the acute issue of impulse control, which is necessary to reduce the likelihood of harming self or others. Correct Answer(s): B

14. An elderly female client with advanced dementia is admitted to the hospital with a fractured hip. The client repeatedly tells the staff, "Take me home. I want my Mommy." Which response is best for the nurse to provide? A) Orient the client to the time, place, and person. B) Tell the client that the nurse is there and will help her. C) Remind the client that her mother is no longer living. D) Explain the seriousness of her injury and need for hospitalization.

Those with dementia often refer to home or parents when seeking security and comfort. The nurse should use the techniques of "offering self" and "talking to the feelings" to provide reassurance (B). Clients with advanced dementia have permanent physiological changes in the brain (plaques and tangles) that prevent them from comprehending and retaining new information, so (A, C, and D) are likely to be of little use to this client and do not help the client's emotional needs. Correct Answer(s): B

An older client with dementia is unable to follow a simple command. The nurse would document this client's behavior as being:

apraxia

14. An elderly female client with advanced dementia is admitted to the hospital with a fractured hip. The client repeatedly tells the staff, "Take me home. I want my Mommy." Which response is best for the nurse to provide? A) Orient the client to the time, place, and person. B) Tell the client that the nurse is there and will help her. C) Remind the client that her mother is no longer living. D) Explain the seriousness of her injury and need for hospitalization.

Those with dementia often refer to home or parents when seeking security and comfort. The nurse should use the techniques of "offering self" and "talking to the feelings" to provide reassurance (B). Clients with advanced dementia have permanent physiological changes in the brain (plaques and tangles) that prevent them from comprehending and retaining new information, so (A, C, and D) are likely to be of little use to this client and do not help the client's emotional needs. Correct Answer(s): B

What is the planned effect of naloxone when it is administered for a heroin overdose?

To compete with opioids for receptors that control respiration

A nurse is caring for several clients who have severe psychiatric disorders. What is the major reason that a health care provider prescribes an antipsychotic medication for these clients?

To reduce the positive symptoms of psychosis

The nurse suggests counseling for a 13-year-old whose close friend has just committed suicide. The nurse's intervention is based on the understanding that an adolescent is at risk for copycat suicide mainly because members of this age group:

Typically mimic the behavior of their peers

A secretary in a home health agency gossip about coworkers and then writes them notes to tell them how valuable they are to the organization and how much she likes working with them. What defense mechanism is being used by the secretary?

Undoing

A nurse is evaluating the medication regimens of clients to determine whether the therapeutic levels have been achieved. For which medication should the nurse review the client's serum blood level?

Valproic acid (Depakene)

The interdisciplinary treatment team is designing a plan for a client with antisocial personality. The client's behaviors include lying to other clients, flattering the primary nurse, verbally abusing a client who has Alzheimer's disease, and being argumentative at counseling sessions. Which behavior is the priority for the treatment team to address?

Verbal abuse of another client.

The interdisciplinary treatment team is designing a plan for a client with antisocial personality. The client's behaviors include lying to other clients, flattering the primary nurse, verbally abusing a client who has Alzheimer's disease, and being argumentative at counseling sessions. Which behavior is the priority for the treatment team to address? Verbal abuse of another client. Lying to the other clients. Flattering his primary nurse. Arguing during counseling sessions.

Verbal abuse of another client.

The serum lithium blood level of a client with a mood disorder, manic episode, is 2.3 mEq/L. What should the nurse expect when assessing this client?

Vomiting, diarrhea, and decreased coordination

A client was recently given a diagnosis of a manic episode of a bipolar I disorder. What activity is most therapeutic for this client at this time?

Walking around the unit with a nurse

While watching television in the dayroom a client who has demonstrated withdrawn, regressed behavior suddenly screams, bursts into tears, and runs from the room to the far end of the hallway. What is the most therapeutic intervention by the nurse?

Walking to the end of the hallway where the client is standing

The nurse observes a female client with schizophrenia watching the news on TV. She begins to laugh softly and says, "Yes, my love, I'll do it." When the nurse questions the client about her comment she states, "The news commentator is my lover, and he speaks to me each evening. Only I can understand what he says." What is the best response for the nurse to make?

What do you believe the news commentator said to you?

The nurse observes a schizophrenic female client watching the news on TV. She begins to laugh softlyand says, "Yes, my love, I'll do it." When the nurse questions the client about her comment shestates, "The news commentator is my lover and he speaks to me each evening. Only I can understandwhat he says." What is the best response for the nurse to make?

What do you believe the news commentator said to you?

The nurse is assessing a client who expresses feeling overwhelmed with the care of an elderly parent. Which question best uncovers the client's perception of this event as a stressor? Have you started drinking or smoking as a result of this stressor? What impact does this stressor have on your life? Are you having trouble getting to sleep or staying asleep? What do you believe is causing you stress right now? Who else is helping with the caregiving and household chores?

What impact does this stressor have on your life? What do you believe is causing you stress right now? Rationale A change in someone's financial status, other stressors present, and an unanticipated stress are factors that have an affect on individuals regardless of their social cultural background or age or gender.

nurse is assessing a client who has been emotionally immobilized since her husband requested a divorce and moved out of their home. What should be determined first by the nurse in the crisis intervention center?

What the divorce means to the client

A male client with schizophrenia who is taking fluphenazine decanoate (Prolixin decanoate) is being discharged in the morning. A repeat dose of medication is scheduled for 20 days after discharge. The client tells the nurse that he is going on vacation in the Bahamas and will return in 18 days. Which statement by the client indicates a need for health teaching?

When I return from my tropical island vacation, I will go to the clinic to get my Prolixin injection.

A nurse is caring for a newly admitted client with obsessive-compulsive disorder. When should the nurse anticipate that the client's anxiety level will increase?

When limits are set on the performance of a ritual

A male client is preparing to leave the hospital and return to college. When saying goodbye, he hugs the nurse and kisses her on the cheek. What is the most appropriate response by the nurse?

Wish him well with his future studies

A client in the psychiatric hospital is attempting to communicate by stating, "Sky, flower, angry, green, opposite, blanket." The nurse recognizes this type of communication as:

Word salad

A nurse is working with a client who has emotional problems. During what stage of the therapeutic nurse-client relationship does the nurse anticipate that most of the client's problem-solving will occur?

Working stage

For which adverse effect should the nurse continually assess a client who is receiving valproic acid (Depakene)?

Yellow sclerae

Which assessment finding alerts the nurse to stop administering haloperidol (Haldol) to a client until further laboratory work is done?

Yellow sclerae

At the first meeting of a group of older adults at a daycare center for the elderly, the nurse asks one of the members what kinds of things she would like to do with the group. The older woman shrugs her shoulders and says, "You tell me, you're the leader." What is the best response for the nurse to make?

Yes, I will be leading this group. What would you like to accomplish during this time?

The mother of an 8-month-old infant with profound mental and physical disabilities tells the RN how depressed she is because she realized that her child will never achieve normal growth and development milestones. how should the RN respond to the mother?

ask the mother if she has ever thought about harming herself or the child

The RN is working with a male client at a community mental health center when the client reports hearing voices that tell him to get a knife from the kitchen and hurt himself. What intervention is most important or the RN to implement?

assign the UAP to remain with the client at all times

The nurse is planning care for a client with bullimia nervosa. What interventions should the nurse plan for this client?

avoid finger foods

During a high school class on substance abuse, a student tells the group "If I tried cocaine, I know I could handle it. I know when to stop". What response is best for the nurse to provide?

denial of an addiction problem is often the first response to the behavior

A female client is admitted to the hospital after attempting suicide. She reveals that her desire for sex has diminished since her child's birth 3 years ago. What is most directly related to the client's loss of interest in sex?

depression

If Lithium is at a therapeutic dose (0.6-1.2)

continue at the current dose and give the med

The nurse is planning care for a client who is experiencing a major depressive disorder. What intervention would be appropriate to address the client's low self-esteem?

discuss perfectionistic behavior

A husband states to the nurse that his wife is not sleeping, buying impulsively, taking last minute trips, and has lost 22 pounds in one month. What is an appropriate nursing diagnosis?

disturbed thought processes

Someone said they want to take disulfiram to limit their alcohol consumption. What do you teach?

disulfiram is for wanting to stop drinking alcohol completely and that's the only way it works

A patient comes in the ED and is drinking and mumbles something about shooting himself? What is biggest priority?

dont let him leave

Pt teaching for TCAs

drink fluids, wear sunglasses and use eye drops for dry eyes, chew gum or candy for dry mouth, eat lots of fiber for constipation.

A male client with schizophrenia is demonstrating echolalia, which is becoming annoying to other clients on the unit. what intervention is best for the RN to implement

escort the client to his room

Someone has recently been prescribed duloxatine, what does the physician need to gather additional data on?

family hx of mental illness any other antidepressants taken recently weight gain liver function

One intervention for bulimia nervosa is the avoidance of

finger food to prevent binge eating

A patient is developing akastheisia, what drug is she most likely taking?

haldol

a young adult with wooded tooth enamel presents to the clinic with multiple complaints including severe pain in the chest and upper abdomen that occurred when the client induced committing after eating a large breakfast. the client also reports severe heartburn in the last and describes a history of taking laxatives and eating prunes whenever overeating. what client problem should the nurse address first?

heart burn

There is a patient with heartburn, tooth decay, abdominal pain, and laxative use. What do you treat first?

heartburn

A client is dressed in a low-cut blouse, no underwear, red lips. What do you do?

help her back to her room to change

A college student who is a victim of a carjacking presents to the community health center and report increased anxiety. During the interview, what nursing intervention should take the highest priority?

help the client feel safe and decrease anxiety

The RN is leading a group on the inpatient psychiatric unit. which approach should the RN use during the working phase of group development?

helping clients identify areas of problem in their lives

While awaiting the results of a urine drug screen a client is experiencing intense cravings, muscle aches, abdominal cramps, watery eyes, and a runny nose. The nurse suspect this client is experiencing withdrawal symptoms associated with

heroin

A client with schizophrenia is crying because of being personally responsible for an earthquake in Chile as reported on the evening news. What should the nurse realize this client is experiencing?

idea of reference

A client with bipolar disorder is admitted with a manic episode and started on lithium therapy 2 weeks ago. Which finding is of most concern to the nurse?

increased thirst and polyuria

A mental health worker (MHW) is caring for a Clint with escalating aggressive behavior. which action by the MHW warrants immediate intervention by the nurse?

is attempting to physically restrain the client

A middle-aged female client with no previous psychiatric history is seen in the mental health clinic because her family describes her as having paranoid thoughts. On assessment, she tells the nurse "I want to find out why these people are stalking me". Which response should the nurse provide?

it sounds like this experience is frightening for you

A male client with known auditory hallucinations begins talking loudly and gesturing wildly while in the unit's day room. What action should the nurse implement first?

listen to what the client is saying

A woman shows codependency when she brings her husband into the ED after a car wreck he caused by drunk driving.

she makes excuses for him and says he was stressed

A client diagnosed with schizophrenia tells the nurse that the FBI is shining a flashlight in his room so that they can follow his activity. Which action would be the best for the nurse to make?

look around the environment and try to identify the cause for the shining light

A male client with long history of alcohol dependency arrives in the emergency department describing the feelings of bugs crawling on his body. His blood pressure is 170/102, his pulse rate is 110 bpm, and is blood alcohol level is 0. What should the nurse administer?

lorazepam (Ativan)

A 35-year-old male client on the psychiatric ward of a general hospital believes that someone is trying to poison him. The nurse understands that a client's delusions are most likely related to his

low self-esteem

A client who is admitted to the mental health unit reports shortness of breath and dizziness. the client tells the nurse "I feel like I'm going to die" which nursing problem should the nurse include in this client's plan of care?

moderate anxiety

The nurse is caring for a client with endocarditis. After reviewing the client's chart, what is the priority nursing assessment? Gentamicin 75 mg IV q8 hours, Penicillin 2 million units IV 14 hours

monitor for tinnitus, imbalance, or visual changes

An older client with severe confusion and pneumonia is prescribed antibiotic therapy and Haldol. For which symptom should the nurse monitor this client during the rest of the day?

neck stiffness

Lithium is the most common cause of

nephrogenic diabetic insipidus

Narcan was administered to an adult client following a suicide attempt with an overdose of hydrocodone bitrate (Vicodin). within 15 min, the client is alert and oriented. in planning nursing care which intervention has the highest priority at this time?

observe the client for further narcotic effects

A client with bipolar disorder is demonstrating sings of mania. What kind of activity is most suitable for this client?

painting

Barbiturates drug names

pentobarbital, phenobarbital

The nurse completes an assessment of a client who is experiencing intimate partner violence (IPV) which finding of the injuries should the nurse include in the documentation?

photographs

A female client engages in repeated checks of door and window locks. Behavior that prevents her from arriving on time and interferes with her ability to function effectively. What action should the nurse take?

plan a list of activities to be carried out daily

A client tells the nurse that she spends most of her days worrying about her family, her health and the possibility of natural disasters. The best action for the nurse to take at this time would be:

point out that time spent worrying often has no impact on actual outcomes

The nurse is completing the admission assessment of an underweight adolescent who is admitted to a psychiatric unit with a diagnosis of depression. which finding requires notification to the healthcare provider?

potassium level 2.9

A client with schizophrenia who persistently experiences auditory hallucinations is being discharged. What intervention should the nurse recommend for the client to practice while experiencing hallucinations?

practice saying "stop" until the hallucination goes away

A client with a fear of heights is seeking help because the phobia is interfering with work expectations. Which strategy should the nurse prepare to instruct to help treat this clients's disorder?

practicing with a kitchen ladder

The nurse is caring for a client who was widowed eight months ago and has two young children. Based on the client's care plan, which intervention should the nurse include for this client?

provide opening for the client to verbalize feelings of hopelessness and loss

A client with schizophrenia is admitted to the psych care unit for aggressive behavior, auditory hallucinations and potential for safe harm. The client has not been taking medications as prescribed and insists that the food has been poisoned and refuses to eat. What intervention should the RN implement?

provide the client with food in unopened containers

The nurse lading a group session of adolescent client gives the members a handout about anger management. one of the male clients is fidgety, interrupts peers when they try to talk and walks about his pets at home. what nursing action is best for the nurse to take?

redirect him by encouraging him to read from the handout

A client in group is talking about her prostitution, the nurse asks her if she was abused by her parents. She states, "my mother ran my father out when I was young". What defense mechanism was used?

repression

What is an example of uncomplicated alcohol abuse withdrawal in a client with chronic alcohol use?

restless and irritated diaphoresis nausea and vomiting grand mal seizures

The nurse interacts with a client who is very depressed and slow to respond to questions. The nurse acts the client to describe current feelings, but the client looks down at the table. What action by the nurse is best to implement?

return at a later time to talk

A guy going through alcohol withdrawal comes in with diarrhea and bloody stool and refuses to take oral meds. What do you do first?

start an IV

A client on the mental health unit is becoming more agitated, shouting at the staff, and pacing in the hallway. When the PRN medication is offered, the client refuses the medication and defiantly sits on the floor in the middle of the unit hallway. What nursing intervention should the RN implement first?

take other clients in the area to the client lounge

A middle-aged adult with major depressive disorder suffers from psychomotor retardation, hypersomnia, and motivation. which intervention is likely to be most effective in returning this client to a normal level of functioning?

teach the client to develop a plan for daily structured activities

During a one-to-one session, the nurse begins to become angry with the client. what action should the nurse take?

terminate the session before the feelings escalate

idea of reference is a false idea which causes

the client to believe that "the media" are referring to her

Based on non-compliance with the medication regimen, an adult client with a medical diagnosis of substance abuse and schizophrenia was recently switched from oral fluphenazine HCl (Prolixin) to IMfluphenazine decanoate (Prolixin Decanoate). What is most important to teach the client and familyabout this change in medication regimen?

the effects of alcohol and drug interaction

During the admission assessment to the mental health unit, a client reports that the people at the office, where the client works, are antagonistic and the client is thinking of shooting the supervisor. The client asks the nurse not to reveal this to anyone else. The nurse immediately notifies the client's therapist and other team members of the client's thought. The therapist then calls the client's supervisor. What outcome is appropriate based on the action of the nurse?

the nurse and therapist will be asked to educate other team members on appropriate sharing of client information

Patients taking selective serotonin reuptake inhibitors (Fluoxetine - Prozac) can have serotonin syndrome which includes

tremors hyperreflexia clonus alternation in mental status

An older client repeatedly talks about taking his own life What should the nurse understand as being the major reason for the client's suicidal wish?

unsuccessful in resolving integrity verse despair issues

Valproic Acid

used to treat bipolar; nursing intervention include monitoring labs- liver enzymes: hepatotoxic Think of Valllll: L - Liver toxic - so we monitor for jaundice and liver labs, ALT's and ASTs L - LOW platelets aka thrombocytopenia - high risk for bleeding -Not pregnancy safe -DO NOT quit abruptly


Kaugnay na mga set ng pag-aaral

Peds--Exam 2---Evovle Chapter 31 - The Infant and Family

View Set

Chapter 11: Exercise for Health and Fitness

View Set

Parental Medications Essential of Med

View Set

Medication Safety and Quality Improvement

View Set

FINISHED Ethics Chapter 5 (ABSOLUTISM VERSUS RELATIVISM)

View Set

Income Statement: Format, Formulas, Etc.

View Set

POLI 130G Indian Prime Ministers and Chief Ministers

View Set